othersss everything

Pataasin ang iyong marka sa homework at exams ngayon gamit ang Quizwiz!

Which statements are true for basal cell carcinoma? Please find the correct combination. 1) It affects mainly elderly patients. 2) It occurs commonly as a plaque with telangiectasias and glistening edges. 3) It is commonly ulcerated. 4) It destroys neighboring tissues. 5) Metastatic spreading is common. A) Answers 1, 2, 3 and 4 are correct B) All answers are correct C) Answers 1, 2 and 4 are correct D) Answers 2, 3 and 5 are correct E) Answers 1, 3 and 5 are correct

A) Answers 1, 2, 3 and 4 are correct

Which signs or symptoms are characteristic for a postthrombotic leg ulcer? 1) White atrophy 2) A varicose vein draining the region of the ulcer 3) Localization just above the ankle on the inside of the leg 4) Intermittent claudication A) Answers 1, 2 and 3 are correct. B) Answers 1 and 3 are correct. C) Answers 2 and 4 are correct. D) Only answer 4 is correct. E) All answers are correct.

A) Answers 1, 2 and 3 are correct.

Which signs or symptoms are characteristic for arteriosclerosis obliterans of the lower extremities? 1) Intermittent claudication 2) Pain at rest 3) Gangrene 4) Reduced pain upon lifting the leg A) Answers 1, 2 and 3 are correct. B) Answers 1 and 3 are correct. C) Answers 2 and 4 are correct. D) Only answer 4 is correct. E) All answers are correct.

A) Answers 1, 2 and 3 are correct.

Which substances do have a keratolytic effect in adequate concentrations? 1) Salicylic acid 2) Urea 3) Lactic acid 4) Boric acid A) Answers 1, 2 and 3 are correct. B) Answers 1 and 3 are correct. C) Answers 2 and 4 are correct. D) Only answer 4 is correct. E) All answers are correct.

A) Answers 1, 2 and 3 are correct.

The biological effect of vitamin-A on the oral mucosa : A) inflammation B) increased keratinization C) atrophy D) hyperplasa E) angular cheilitis

B) increased keratinization In vitamin-A deficiency -the is considered as an epithelial protective vitamin increased mucous membrane keratinization occurs. The other lesions listed are not directly associated with the vitamin-A deficiency.

Etiology of median cervical cyst or fistule is a A) remaining thoracic duct B) remaining thyroglossal duct C) laryngocele D) struma

B) remaining thyroglossal duct The median cervical cyst or fistula is a remnant of the thyroglossal duct, and so during surgery the removal of the middle part of the hyoid bone is suggested in order to avoid remnants.

By the help of Valsalva maneuver we can A) diagnose otosclerosis B) ventilate the middle ear C) detect nystagmus D) exclude conductive hearing loss E) evaluate the respiratory function of the nose

B) ventilate the middle ear By applying the Valsalva maneuver, we can ventilate the ear. The air goes from the nostrils to the middle ear.

Nevi ... do not have to be removed surgically. A) growing fast B) which do not change over many years C) changing their color (such as darkening) D) which ulcerate or bleed E) which often itch

B) which do not change over many years

Please associate following pathogens with the statements below. A) Trichophyton schönleini B) Microsporum canis C) Trichophyton rubrum D) Trichophyton verrucosum BOR - 40 - It results in usually a superficial mycosis of the scalp and trunk. If the scalp is affected, systemic antimycotic treatment is indicated. On the trunk, it is characterized by red, annular, oval or round plaques with scaling edges. BOR - 41 - It leads to a chronic dermatitis of the scalp which typically heals with scarring alopecia. BOR - 42 - It affects typically animals such as cows; in humans, it causes usually a deep tinea of the scalp (also called kerion Celsi) and tinea barbae. BOR - 43 - This is the most common pathogenic dermatophyte in Central Europe including Hungary.

BOR - 40 - It results in usually a superficial mycosis of the scalp and trunk. If the scalp is affected, systemic antimycotic treatment is indicated. On the trunk, it is characterized by red, annular, oval or round plaques with scaling edges. - B) BOR - 41 - It leads to a chronic dermatitis of the scalp which typically heals with scarring alopecia. - A) BOR - 42 - It affects typically animals such as cows; in humans, it causes usually a deep tinea of the scalp (also called kerion Celsi) and tinea barbae. - D) BOR - 43 - This is the most common pathogenic dermatophyte in Central Europe including Hungary. - C)

Which therapeutic option(s) is (are) appropriate for the treatment of a postthrombotic leg ulcer? 1) Systemic antibiotics 2) Deep venous thrombosis prophylaxis 3) Systemic vasodilation by calcium channel blockers 4) Compression stockings A) Answers 1, 2 and 3 are correct. B) Answers 1 and 3 are correct. C) Answers 2 and 4 are correct. D) Only answer 4 is correct. E) All answers are correct.

C) Answers 2 and 4 are correct.

Charrière = A) 0,5 mm B) 3 mm C) 1 mm D) 0,33 mm

D) 0,33 mm Charrière (Ch) is the outer diameter of the catheters; 1 Ch = 0.33 mm.

The recommended maximum fat intake as a percentage of total energy intake according to WHO is approximately A) 5% B) 10% C) 15% D) 30%

D) 30% It has long been assumed that it does not make any particular difference what part of our energy needs is covered by proteins, fats and carbohydrates. Epidemiological studies of recent decades have clearly confirmed not only the importance of individual nutrients but also their correct proportions. According to the WHO recommendations, 15-30% of the energy intake should come from fat consumption. The increase of fat intake is mainly due to the consumption of food from animal origins (meat, milk), which is at the expense of the complex carbohydrate consumption. These so-called high-energy, western-type diets increase the risk of many chronic non-infectious diseases.

What is the most important immunological marker of mixed connective tissue disease (MCTD)? A) Anti-CCP B) Antinuclear factor C) Anti-SSA D) Anti-U1RNP E) Anti-SSB

D) Anti-U1RNP For MCTD, the anti-U1RNP antibody is the most specific, for RA, the anti-CCP, for SLE, the antinuclear factor, and for Sjögren's syndrome anti-SSA and -SSB are the most specific.

Which is the most common food poisoning in Hungary nowadays? A) Staphylococcus aureus intoxication B) Salmonellosis C) Bacillus cereus intoxication D) Campylobacteriosis

D) Campylobacteriosis Campylobacterosis has been the most frequently reported gastrointestinal disease in the EU Member States and in Hungary since 2005. In recent years, the number of reported salmonellosis cases is declining in many countries, including Hungary, although it is still one of the most common causes of gastrointestinal infections, similarly to Campylobacter infections, which involves the most commonly infants (0- 4 years old).

What is the most common service type within the activity of the pulmonary care institutions in Hungary? A) X-ray screening ("lung screening") B) complex screening C) Other primary and secondary prevention services D) Chronic care and treatment of pulmonologic patients

D) Chronic care and treatment of pulmonologic patients Pulmonary care institutions are dealing manily with providing care for pulmonological patients. The volume of X-ray screening has been decreased as the method is not effective enough for detecting lung cancers, and, on the other hand, the prevalence of TBC does not justify the regalur screening at populational level. On the basis of its experience and infrastructure the pulmonary care network performs complex screening services to enhance prevention and to utilize the capacities, but this is not its main activity area..

Which of the following factors plays a certain role in the epidemiology of prostate cancer? A) age B) genetic factors C) hormone levels D) most likely all three

D) most likely all three Although there are no clear pathogenic factors in the epidemiology of prostate cancer, it is likely that age (older age), genetic factors (more commonly occurring in the family) and hormonal factors (androgen-dependent) are likely to be involved in the development of the disease.

In which disease(s) should the patient be separated/isolated/quarantined at the joint Szent István - Szent László Hospital? 1) Hepatitis A 2) AIDS 3) typhoid abdominalis 4) acute anterior polyomyelitis A) Answers 1, 2 and 3 are correct B) Answer 1 and 3 are correct C) Answer 2 and 4 are correct D) only 4 is right E) all 4 answers are correct

D) only 4 is right

It is typical for renal colic, except: A) kidney sensitivity B) nausea, vomiting C) pain in the labias or in the testicles D) sensitivity at the McBurney's point

D) sensitivity at the McBurney's point Typical renal colic is associated with intense pain, which spreads into the testicles of the same side or in the labias and often comes with nausea and vomiting. The pressure sensitivity experienced at the McBurney point is typical for acute appendicitis.

Osteoma of a paranasal sinus A) metastatizes to the bones B) is a congenital lesion C) most commonly affects swimmers D) should be surgically removed E) can be treated with radiotherapy

D) should be surgically removed Paranasal sinus osteoma is removed after a surgical intervention. Pain, obstruction of the sinus openings and the aesthetic deformation is resolved.

Which is a non-muscarine symptom? A) Bradycardia B) bronchospasm C) increased sweating D) spasm of the respiratory muscles E) needle pupil

D) spasm of the respiratory muscles The spasms of the respiratory muscles is a nicotine-like symptom due to organic phosphates, while bradycardia, bronchospasm, increased sweating and needle pupil are muscarinic alterations.

The most common malignant tumor of the larynx is: A) adenocarcinoma B) chondrosarcoma C) adenocystic carcinoma D) squamous cell carcinoma

D) squamous cell carcinoma The most common malignant tumor of the larynx is the squamous cell carcinoma.

Adequate therapy of a confirmed otosclerosis is A) mastoidectomy B) paracentesis C) transnasal Eustachian tube inflation D) stapedectomy or stapedotomy

D) stapedectomy or stapedotomy A choice of treatment in otosclerosis is removal of the sclerotic stapes and replacement by a prosthetic one: stapedectomy. Another type of operation is stapedotomy, when only the stapes superstructure is removed and replaced by a prosthesis.

The best radiographic modality for the diagnosis of an esophageal foreign body is A) chest X-ray B) barium swallow test C) swallow radiography without contrast media D) swallow radiography with absorbable contrast media

D) swallow radiography with absorbable contrast media The esophageal foreign body is best detectable by swallow radiography with absorbable contrast media.

At the contraction of the ciliary muscle: 1) decreasing the pulling effect of the lens zonules 2) the crystalline lens thickens 3) the refractive power increased 4) the pupil dilates A) 1st, 2nd and 3rd answers are correct B) 1st and 3rd answers are correct C) 2nd and 4th answers are correct D) only 4th answer is correct E) all of the answers are correct

A) 1st, 2nd and 3rd answers are correct During the constriction of the ciliary muscle the pupillary reaction does not change, but on the other hand the refraction increases, because the lens zonules relax and the crystalline lens thickens (becomes more convex).

Which formula is used to calculate relative excess mortality rate (REMR)? (expected number of deaths: E; observed number of deaths: O) A) (O - E)/ O × 100 B) (O - E) / E × 100 C) (E - O) / O × 100 D) (E - O) / E × 100 E) none of them

A) (O - E)/ O × 100 The relative excess mortality rate and the excess mortality rate differ in their denominator only. The denominator of the relative excess death rate is the observed number of deaths instead of expected number of deaths. REMR=(O-E)/O ×100

It is characteristic to the central midface bone fracture: 1) diplopia 2) swollen face 3) nasal bleeding 4) liquour discharge A) 1st, 2nd and 3rd answers are correct B) 1st and 3rd answers are correct C) 2nd and 4th answers are correct D) only the 4th answer is correct E) all of the answers are correct

A) 1st, 2nd and 3rd answers are correct The Le Fore II mid-facial fracture usually involves the bones of the orbital basis , the bones move caudally and therefore diplopia develops. The soft tissue injury leads to sudden swelling of the face, because the mimic muscle have no fascia sheets. The nasal bleeding is also a common sing. This fracture does not involves the cranial bones. The liquour discharge is not common-

Function of the vitreous: 1) to transmit the light 2) to support the crystalline lens and retina 3) to deliver metabolic products to the retina 4) tear production A) 1st, 2nd and 3rd answers are correct B) 1st and 3rd answers are correct C) 2nd and 4th answers are correct D) only 4th answer is correct E) all of the answers are correct

A) 1st, 2nd and 3rd answers are correct The vitreous is a gel like material filling the inside of the eye, has nothing to do with tear production, but it transmits the light, give a support to the crystalline lens and transports metabolic products to the retina.

The conversion of testosterone into dihydrotestosterone is done by: A) 5α-reductase B) 7-β-citosterine C) aromatase D) acid phosphatase

A) 5α-reductase In the development of BPH an important role is played by the dihydrotestosterone converted from testosterone by the 5-α reductase.

How much time is spent on average by general practitioners with one patient? A) 6-7 minutes B) 10-12 minutes C) 16-18 minutes D) 25-27 minutes

A) 6-7 minutes Based on Hungarian statistics, general practitioners serve 8-10 patients per hour in general, i.e. 6-7 minutes fall on one patient.

The normal serum creatinine level is: A) 60-120 micromol/liter B) 6-12 micromol/ liter C) 6-120 microgramm/ liter D) 60-120 milligram / liter

A) 60-120 micromol/liter The normal creatinine level is 60-120 μmol / l.

Which medicine is NOT effective for treating an acute gout attack? A) Allopurinol B) Corticosteroid C) Colchicine D) Non-steroidal anti-inflammatory drugs E) TNF-α inhibiting biological agents

A) Allopurinol In gout, allopurinol is responsible for long-term reduction of elevated uric acid levels but it isn't effective in the treatment of seizures.

Which of the listed diseases is more common in men? A) Ankylosing spondylitis B) Rheumatoid arthritis C) Systemic lupus erythematosus D) Polymyalgia rheumatica E) Sjögren syndrome

A) Ankylosing spondylitis Only ankylosing spondylitis dominates in men, the other diseases listed are more common in women.

Condyloma acuminatum occurs commonly ... 1) in HIV positive patients 2) in patients with chronic vaginal discharge 3) in promiscuous individuals 4) associated with menorrhagia 5) in pregnancy A) Answers 1, 2 and 3 are correct B) Answers 1, 2 and 4 are correct C) Answers 1, 4 and 5 are correct D) Answers 1, 3 and 5 are correct E) Answers 2, 3 and 4 are correct F) Answers 2, 3 and 5 are correct G) Answers 2, 4 and 5 are correct H) Answers 3, 4 and 5 are correct

A) Answers 1, 2 and 3 are correct

Which statements are true for pyogenic granuloma? Please find the correct combination. 1) It is intense red. 2) It bleeds easily. 3) It develops usually after an injury. 4) It can transform into malignancy. 5) It has a firm consistence. A) Answers 1, 2 and 3 are correct B) Answers 1, 2 and 4 are correct C) Answers 1, 4 and 5 are correct D) Answers 1, 3 and 5 are correct E) Answers 2, 3 and 4 are correct F) Answers 2, 3 and 5 are correct G) Answers 2, 4 and 5 are correct H) Answers 3, 4 and 5 are correct

A) Answers 1, 2 and 3 are correct

What are the characteristics of PCBs? 1) they accumulate in the adipose tissue 2) they are excreted in breast milk 3) they induce microsomal enzymes in the liver 4) the population is mainly exposed to them by inhalation A) Answers 1, 2 and 3 are correct B) Answers 1 and 3 are correct C) Answer 2 and 4 are correct D) only answer 4 is correct E) all 4 answers are correct

A) Answers 1, 2 and 3 are correct PCBs accumulate in the food chain due to their accumulation in animal and human adipose tissue, and are excreted in breast milk. They induce the microsomal enzymes of the liver. The population is mainly exposed to PCBs orally by means of food.

Risk factor(s) of osteoporosis is (are): 1) decrease in oestrogen production 2) physical inactivity 3) smoking 4) D-hypervitaminosis A) Answers 1, 2 and 3 are correct B) Answers 1 and 3 are correct C) Answers 2 and 4 are correct D) Only answer 4 is correct E) All of the answers are correct

A) Answers 1, 2 and 3 are correct The main risk factors of osteoporosis are: decrease in estrogen production, physical inactivity and smoking. D-hypervitamonis causes bone alterations characterized by increased calcification.

Furuncles occur commonly ... 1) in diabetes mellitus 2) due to prolonged therapy with systemic or topical glucocorticoids 3) in immunodeficiency 4) in malignant melanoma A) Answers 1, 2 and 3 are correct. B) Answers 1 and 3 are correct. C) Answers 2 and 4 are correct. D) Only answer 4 is correct. E) All answers are correct.

A) Answers 1, 2 and 3 are correct.

Tuberculin test is anergic ... 1) in patients who had not been immunized against M. tuberculosis 2) under immunosuppressive therapy 3) in elderly 4) after tuberculotic infections A) Answers 1, 2 and 3 are correct. B) Answers 1 and 3 are correct. C) Answers 2 and 4 are correct. D) Only answer 4 is correct. E) All answers are correct.

A) Answers 1, 2 and 3 are correct.

Which manifestation is typical for psoriasis? 1) Arthropathy 2) Pustules 3) Onychodystrophy 4) Scaling of the lips (cheilitis) A) Answers 1, 2 and 3 are correct. B) Answers 1 and 3 are correct. C) Answers 2 and 4 are correct. D) Only answer 4 is correct. E) All answers are correct.

A) Answers 1, 2 and 3 are correct.

First diagnostic procedure: A 65-year-old man comes to the clinic with a complaint of frequent urination. He also complains about low back and leg pain. There is no blood in the urine, urination is not painful, comes in weak yellow stream. A) DRE (digital rectal examination) B) PSA (prostate specific antigen) C) Transrectal ultrasound (TRUS) D) prostate biopsy

A) DRE (digital rectal examination) When examining male patients with urinary complaints, the first test to be performed is the urinalysis and then the rectal digital examination, which is the simplest and the cheapest and is always mandatory.

What is the absolute classification criteria for gout diagnosis? A) Detection of synovial monosodium monohydrate crystals B) Uric acid nephrolithiasis C) Occurrence of acute flares of gouty arthritis D) The presence of tophuses E) Hyperuricemia

A) Detection of synovial monosodium monohydrate crystals During the diagnosis of gout, the detection of urinary crystals is an absolute marker, the other symptoms are characteristic, but not absolute criterions.

In which joints does Heberden's osteoarthritis develop? A) In distal interphalangeal joints B) In proximal interphalangeal joints C) In knee joints D) In the small joints of the leg E) Anywhere

A) In distal interphalangeal joints Heberden's osteoarthritis is a degenerative lesion of the DIP joints.

What changes could be seen related to the average life expectancy of men over the age of 65 in the different social strata/categories between 1972 and 2001 in England? A) Increased in all social categories, but the increase was the highest among those in the highest category. B) Increased in all social categories at similar rates C) Increased in all social categories, but the change was higher in the lowest category compared to change affecting those in the highest social category D) Increased in the highest category and decreased in the lowest category

A) Increased in all social categories, but the increase was the highest among those in the highest category. The socio-economic situation is a factor that significantly affects health status and its various indicators, including life expectancy.Life expectancy was significantly greater among those in better position compared to those in lower socioeconomical groups in England.

Which bone disease is NOT characterized by elevated serum alkaline phosphatase? A) Osteoporosis B) Osteomalacia C) Primary hyperparathyroidism D) Paget's disease of the bones E) Secondary hyperparathyroidism

A) Osteoporosis Serum alkaline phosphatase enzyme levels are elevated in bone diseases except in osteoporosis.

By performing the TRUS procedure an increased, non homogenic prostate is found. The capsule is ruptured. The PSA level is 20 ng/ml. The DRE shows a large, not sensitive prostate with hard, uneven surface. What is the most likely diagnosis? A 65-year-old man comes to the clinic with a complaint of frequent urination. He also complains about low back and leg pain. There is no blood in the urine, urination is not painful, comes in weak yellow stream. A) PCA (prostate cancer) B) BPH (Benign prostatic hyperplasia) C) chronic prostatitis D) acute prostatitis

A) PCA (prostate cancer) The hard, uneven prostate, the inhomogeneous prostate structure detected with elevated PSA level and with TRUS, and the broken capsule, refer to prostate cancer. BPH: palpation found: glandular or muscle like prostate. US test: the capsule is intact although the PSA may have elevated but is below 10 ng / ml. Prostatitis chronica: sensitive, stuffed founds in the prostate, US may exhibit inhomogeneity, possibly echodense masses in the prostate, PSA may be moderately elevated. Acute prostatitis: patient has high fever, a very sensitive, dough-sensitive, enlarged prostate.

It does NOT belong to the characteristics of adult-onset Still's disease: A) Pigmented villonodular synovitis B) High fever C) Maculopapular rash on the upper arms D) Arthralgia E) Visceral symptoms

A) Pigmented villonodular synovitis Pigmented villonodular synovitis is a benign synovial proliferation and an independent disease, the other four symptoms are characteristic of the adult-onset Still's disease.

What subgroup of juvenile idiopathic arthritis is characterized primarily by rheumatoid factor positivity? A) Polyarthritis B) Systemic form C) Oligoarthritis D) Enthesopathic arthritis E) Psoriatic arthritis

A) Polyarthritis The polyarthritis subgroup may show seropositivity, the rest mostly doesn't.

In which field do medical doctors work as entrepreneurs with the highest proportion? A) Primary Care B) Outpatient care - Dentists C) Outpatient care - Specialists (except for dentists) D) Active inpatient care

A) Primary Care There is a relatively high proportion of physicians working in primary and outpatient care as enterpreneurs, while this proportion is below 10% in inpatient care and is mainly restricted to specific occupations and duties. Proportion of enterpreneurs working in outpatient care is between 50% and 70% (with a higher proportion in the case of dentists), while the majority of primary care physicians (around 85%) are entrepreneurs.

The salivary flow will not enhanced : A) Sjögren's syndrome B) acute heavy metal poisoning C) during dental treatment D) trigeminal neuralgia E) in epileptic attack

A) Sjögren's syndrome The mechanical, chemical and neurogenic stimuli can enhance the salivation. In this way the B,C.D are associated with elevated flow rate. The Sjogren's disease , the autoimmune disease of the major salivary glands is associated with severe xresotomia. In this disease the mouth is totally dry and salivation cannot be stimulated.

What is the most appropriate procedure? A 50-year-old male presents with a bleeding, pruritic, exophytic, ulcerated, brownish black, nodular tumor of 1 cm diameter over the left scapula which has grown quickly in the last half a year. A) Surgical excision of the primary tumor B) Irradiation of the primary tumor followed by surgical excision of the tumor remnants C) Chemotherapy of the primary tumor followed by surgical excision of the tumor remnants D) Surgical excision of the primary tumor followed by postoperative irradiation

A) Surgical excision of the primary tumor

Which of the following statements is not true related to blood provision in Hungary? A) The tasks related to blood provision is managed by the Hungarian Red Cross B) blood donation is voluntary C) blood donation is free of charge D) human blood can not be marketed

A) The tasks related to blood provision is managed by the Hungarian Red Cross The main principles of blood provision are the following in Hungary. The tasks related to blood provision are managed by the Hungarian National Blood Transfusion Service (HNBTS). Hungarian Red Cross participates in the organization of blood donations. Blood donation is voluntary, unlike in some countries where a fee is payed for donatng blood. Human blood and blood products could not be marketed.

Which of the following definitions describes clinical effectiveness? A) To achieve the greatest possible health gain compared to the available resources B) Effective use of care resources at the given level C) A higher level of health care than the care provided by family doctors and outpatient institutions D) Minimizing the risk of side effects and complications

A) To achieve the greatest possible health gain compared to the available resources The definition of clinical efficacy is to maximize health gain compared to available resources. This can only be achieved through the efficient use of resources and the minimization of complications, but these are the indicators of clinical effectiveness not the definition of clinical effectiveness itself.

What is the most common cause of deaths among 10-18 year olds? A) accidents B) suicides C) upper respiratory tract infections D) gastrointestinal infections

A) accidents Approximately 37% of deaths are caused by accidents among individuals aged 10-18. The number of deaths related to accidents is more then two times bigger among boys compared to girls.

Act LXXXIII (1997) (about the mandatory health insurance in Hungary) applies which principles to the regulation of health insurance from the followings? 1) Financial health insurance services are proportional to the amount of the mandatory health insurance contribution 2) Healthcare services have the same professional content for all people who are entitled to health care 3) The State ensures that the services specified in this Act be fulfilled even if the necessary costs can not be covered by the health insurance 4) In the structure of the health care system, progressivity has the utmost importance A) answers 1, 2 and 3 are correct B) Answer 1 and 3 are correct C) 2 and 4 are correct D) only 4 is correct answer E) all 4 answers are correct

A) answers 1, 2 and 3 are correct Act LXXXIII of 1997 lays down five principles for the regulation of health insurance in Hungary: health services can be used to an extent justified by the health conditions under the law (the service is independent of the amount of the contribution); Cash benefits may be used in proportion to the health insurance contribution (eg sickness benefit); Healthcare services offer the same professional services to all people entitled to healthcare (equal opportunities); the state ensures the fulfillment of the statutory services from the state budget, even if the necessary expenses cannot be covered by the health insurance (state commitment); the authorities carrying out the health insurance tasks inform the insured persons of their rights and obligations and assist in enforcing their claims (right to information). The principle of progressivity does not appear here, it can be found among the principles of the act CLIV (1997) regarding health care.

The definition of health defined by Stokes et al. combines the following: 1) anatomical integrity 2) ability to be "valuable" for the family, in the workplace and in society 3) ability to overcome physical, biological and social burdens/stress 4) the absence of polymorphism in the gene pool A) answers 1, 2 and 3 are correct B) answers 1 and 3 are correct C) answers 2 and 4 are correct D) only answer 4 is correct E) all of the answers are correct

A) answers 1, 2 and 3 are correct All components were mentioned in the definition created by Stokes et al., except the absence of polymorphism, which does not exist at individual level.

Which of the listed pathogens play(s) a role in the development of tumors? 1) HPV 2) HBV 3) HCV 4) HAV A) answers 1, 2 and 3 are correct B) answers 1 and 3 are correct C) answers 2 and 4 are correct D) only answer 4 is correct E) all of the answers are correct

A) answers 1, 2 and 3 are correct Among the listed pathogens, HPV participates in the development of cervical cancer, HBV and HCV could cause liver cancers; while HAV is not a carcinogenic virus.

Select which of the listed diseases are mandatory to be screened for in certain work-related activities (in child-care, educational and health institutions, food industry, etc.). 1) syphilis 2) gonorrhea 3) HIV 4) genitourinary chlamydial infection A) answers 1, 2 and 3 are correct B) answers 1 and 3 are correct C) answers 2 and 4 are correct D) only answer 4 is correct E) all 4 answers are correct

A) answers 1, 2 and 3 are correct As specified in various acts, screening tests should be performed in certain jobs in order to detect syphilis, gonorrhea and HIV infection to prevent the transmissions of these diseases.

3 months after the TUR surgery of bladder tumor cystoscopy should be performed because this is the best way to detect the possible recurrence. A) both the statement and the explanation are true and a causal relationship exists between them; B) both the statement and the explanation are true but there is no causal relationship between them; C) the statement is true, but the explanation is false; D) the statement is false, but the explanation itself is true E) both the statement and the explanation are false

A) both the statement and the explanation are true and a causal relationship exists between them

Clearance investigations are required in case of infection with the following pathogen(s): 1) Salmonella enteritidis 2) Salmonella typhi 3) Vibrio cholerae 4) Yersinia enterocolitica A) answers 1, 2 and 3 are correct B) answers 1 and 3 are correct C) answers 2 and 4 are correct D) only answer 4 is correct E) all 4 answers are correct

A) answers 1, 2 and 3 are correct Clearing examinations are needed in the following infections: 1. Salmonellosis. After recovering, a fecal bacteriological control (release) test should be carried out only in persons who attend communities of children of 0 to 3 years, who are receiving care in social or health care institutions and who are working with breast milk or are giving breast milk. These persons are allowed to continue their activity in case of having two normal stool tests on two consecutive days started two days after the clinical recovery. 2. Typhus abdominalis. The patient should be isolated until tests conducted on fecal and urine samples on 3 consecutive days starting 48 hours after the cease of the clinical symptoms and the end of the antibiotic therapy give negative results. If only one of the tests gives positive result then the isolation may be discontinued, but the patient should undergo a clinical observation until the pathogen shedding/carrying is ceased. 3. Cholera. The patient cured from cholera have to be isoleted until bacteriological stool tests on two consecutive days starting 48 hours after clinical recovery and end of the antibiotic therapy give negative result. 4. After the recovery of infection with Yersiniosis no mandatory release tests are required.

Findings leading to the establishment of modern epidemiology: 1) microbiological identification of pathogens 2) discovering the immune system 3) invention of antimicrobial chemotherapeutics 4) vaccination against smallpox A) answers 1, 2 and 3 are correct B) answers 1 and 3 are correct C) answers 2 and 4 are correct D) only answer 4 is correct E) all 4 answers are correct

A) answers 1, 2 and 3 are correct Development in the field of microbiology, immunology and pharmacology bring forth the qualitative changes (improvements) of "new age"/modern epidemiology. These findings include the identification of pathogens, discoveryof the body's reactions against them and the discovery ofchemotherapeutics (antimicrobial drugs).

The main objectives of public health are: 1) continuous monitoring of the given population's health status, identification of groups/individuals with increased risk, identification of health problems and public health priorities 2) development of health promotion and disease prevention programs 3) an investigation related to the availability of health services 4) establishing the healthcare framework A) answers 1, 2 and 3 are correct B) answers 1 and 3 are correct C) answers 2 and 4 are correct D) only answer 4 is correct E) all of the answers are correct

A) answers 1, 2 and 3 are correct Development of organizational framework of healthcare is not a public health objective; since in this field only supporting the planning is a task of the public health to ensure the general availability.

According to the recommendation based on the cooperation of the World Health Organization and the American Society of Diabetes, diabetes screening is characterized by the following: 1) it is reasonable above the age of 45 2) it is based on determining fasting blood sugar levels 3) In case of normal test results, the repetition of the test on a three yearly basis seems to be appropiate 4) it is based on the oral glucose tolerance test A) answers 1, 2 and 3 are correct B) Answer 1 and 3 are correct C) Answer 2 and 4 are correct D) only answer 4 is correct E) all 4 answers are correct

A) answers 1, 2 and 3 are correct Earlier start and more freuqent screening is suggested for individuals whose BMI reaches or exceeds 27 kg/m2, who has straight-line relatives with diabetes, who have offspring(s) with a birth weight over 4 kg, who have have 2.9 mmol or higher triglyceride levels and/or have decreased HDL cholesterol level (0.9 mmol/l or lower). According to the recommendation regular diabetes screening based on assesing fasting glucose is suggested for persons above the age of 45. In case of a normal test result, repetition of the test on a three yearly basis seems to be appropiate

Nutritional recommendation for cancer prevention: 1) Cutdown of alcohol consumption 2) Breastfeeding 3) increased consumption of fresh fruits and vegetables 4) Avoiding milk consumption A) answers 1, 2 and 3 are correct B) Answer 1 and 3 are correct C) 2 and 4 are correct D) only 4 is correct answer E) all 4 answers are correct

A) answers 1, 2 and 3 are correct Excessive alcohol consumption increases the risk of oral, pharyngeal, laryngeal, esophagus, liver, colon, rectum and breast cancer, therefore cutdown of alcohol consumption is recommended. Breastfeeding has a positive effect on the mother as it reduces breast cancer risk and is beneficial to the baby as it reduces the risk of adult obesity and therefore the development of many cancers. The beneficial effect of increased fruit and vegetable consumption is due to the fact that they contain high quantities of dietary fibers, vitamins, minerals and other phytochemicals. The protective effect of milk consumption was observed against the development of colorectal cancer.

Patients infected with the following disease(s) should be isolated: 1) meningitis epidemica 2) parotitis epidemica 3) scarlet fever 4) legionellosis A) answers 1, 2 and 3 are correct B) answers 1 and 3 are correct C) answers 2 and 4 are correct D) only answer 4 is correct E) all of the answers are correct

A) answers 1, 2 and 3 are correct Patients infected with meningitis epidemica or parotis epidemica must be isolated in rooms for infectious diseases. Patients infected with scarlat fever must be isolated at their own home or at rooms for ifectious diseases. The reason behind the isolation is that all three types of diseases are transmitted by humans. Since the transmission of the two forms of Legionellosis from humans to humans does not play a role, the isolation is not necessary in these cases.

Amongst the etiological factors of malignancies, in case of men the size of which risk factor(s) reaches or exceeds 10% ? 1) lifestyle factors 2) smoking 3) exposure to sunlight 4) occupational exposure A) answers 1, 2 and 3 are correct B) answers 1 and 3 are correct C) answers 2 and 4 are correct D) only answer 4 is correct E) all of the answers are correct

A) answers 1, 2 and 3 are correct Sizes of risk factors among the aetiological factors for men: lifestyle factors 35%, smoking 30%, sunlight 10%, occupational exposure 5%, alcohol 5%.

The census 1) involves all members of the population 2) examines the population for a certain time (date?) 3) in Hungary census was first conducted between 1784 and 1778 4) examines a representative sample of the population A) answers 1, 2 and 3 are correct B) Answers 1 and 3 are correct C) Answers 2 and 4 are correct D) only answer 4 is correct E) all 4 answers are correct

A) answers 1, 2 and 3 are correct The basic data of structural demography is provided by the census that counts all individuals on the basis of well-defined structural characteristics for a given time. Censuses are usually conducted in every 10 years in developed countries. Investigations of the ongoing changes between two censuses are based on the data obtained from calculations using extra- and interpolational methods. The ongoing changes can also be examined by analysises conducted on representative samples (1-5% of the population; microcensuses). The traces of censuses can be detected in the ancient times. Modern censuses are conducted since the 17th-18th century. The first census was implemented in Hungary due to the will of József the 2nd between 1784 and 1787 while the next census was. only in conducted in 1869. Since then, censuses are usually conducted in every 10 years.

Which muscarinic symptoms can be alleviated by the addition of atropine in organic phosphate ester poisoning? 1) bronchospasm 2) bradycardia 3) narrowed pupils (pinpoint pupils) 4) spasms of the respiratory muscles A) answers 1, 2 and 3 are correct B) answers 1 and 3 are correct C) answers 2 and 4 are correct D) only answer 4 is correct E) all of the answers are correct

A) answers 1, 2 and 3 are correct The first three symptoms (bronchospasm, bradycardia, pinpoint pupils) are muscarine-like symptoms, therefore they can be mitigated by atropine, while the fourth symptom, the spasm of the respiratory muscles, as being a nicotinic answer, could not be treated with atropine.

Strategic groups of suicide prevention are the followings: 1) Training of professionals who are able to identify individuals with increased risk 2) Elimination of the identified risk factors as much as possible 3) Providing direct help through the use of crisis ambulances and helplines 4) Employment program aiming at the mentally ill people A) answers 1, 2 and 3 are correct B) Answer 1 and 3 are correct C) Answer 2 and 4 are correct D) only answer 4 is correct E) all 4 answers are correct

A) answers 1, 2 and 3 are correct The literature distinguishes five strategic groups of interventions related to suicide prevention. These include all methods except the employmet of persons with mental disorders.

Factors that increase the risk of obesity: 1) sedentary lifestyle 2) poor socio-economic status (in developed countries) 3) high consumption of soft drinks 4) foods with low glycemic index A) answers 1, 2 and 3 are correct B) answers 1 and 3 are correct C) answers 2 and 4 are correct D) only answer 4 is correct E) all of the answers are correct

A) answers 1, 2 and 3 are correct The main factors increasing the risk of obesity are the sedentary lifestyle, the increased consumption of high-energy foods, the increased consumption of fast-food menus and sweetened soft drinks. Regular exercises and a home/school environment supporting healthy alimentation help to maintain optimal body weight. Regular consumption of foods with low glycemic index (legumes, vegetables, whole grains) are considered as protective factor in the aspect of obesity. Especially high prevalence of obesity could be seen among women with poor social status in developed countries.

The revisions of the International Classification of Diseases (ICD) are characterized by the followings: 1) Revision 10 is currently valid 2) Revision 11 is currently in progress 3) professional reviews are supervised by the WHO 4) Class boundaries will be ereased in the 11th revision A) answers 1, 2 and 3 are correct B) Answer 1 and 3 are correct C) Answer 2 and 4 are correct D) only answer 4 is correct E) all 4 answers are correct

A) answers 1, 2 and 3 are correct The tenth, currently valid revision of the ICD was implemented on 1 Jan 1996. The 11th revision of ICD is currently in progress and the expected year of its completion is 2014. The revisions have been supervised by the WHO since 1948. One of the most important elements of revision is to specify better the class boundaries rather than to erase them.

The following reported infectious diseases should be unreported: 1) in case of long lasting changes affecting any organ 2) in case of complications 3) in case of lethal outcome 4) in case of severe diseases A) answers 1, 2 and 3 are correct B) Answer 1 and 3 are correct C) Answer 2 and 4 are correct D) only answer 4 is correct E) all 4 answers are correct

A) answers 1, 2 and 3 are correct Unreporting must be done in case of long lasting changes effecting any organ, complications or fatal outcome.

For which of the following areas and institutions, respectively lies the responsibilities on State Secretariat for Healthcare of the Ministry of Human Capacities 1) National Public Health and Medical Office Service 2) Patient care at universities 3) The authorities of the health care with national power 4) WHO Regional Office A) answers 1, 2 and 3 are correct B) Answer 1 and 3 are correct C) Answer 2 and 4 are correct D) only answer 4 is correct E) all 4 answers are correct

A) answers 1, 2 and 3 are correct While the first three areas / institutions are the responsibility of the State Secretariat for Healthcare, the Regional Office of WHO is not supervised by them, but they coordinate the cooperation with it.

Which part of the tympanic membrane should be preferred when doing a paracentesis? A) antero-inferior quadrant B) antero-superior quadrant C) postero-superior quadrant D) postero-inferior quadrant

A) antero-inferior quadrant In paracentesis the preferred quadrant is the anterior inferior since there is no structure behind it that we can damage, and the fluid easily flows out from there.

Renal colic is a relative contraindication to IVP, because no filtration appears in the kidneys during the colic period. A) both the statement and the explanation are true and a causal relationship exists between them; B) both the statement and the explanation are true but there is no causal relationship between them; C) the statement is true, but the explanation is false; D) the statement is false, but the explanation itself is true E) both the statement and the explanation are false

A) both the statement and the explanation are true and a causal relationship exists between them During renal colic, the affected kidneys are obstructed, so a reduced degree of contrast agent will be visible. Thus, the IVP test performed at that time will not be valuable.

Before radical prostatectomy (PSA under 10 ng/ml) regional lymph node dissection should be performed, because the result influences the possibly occurring adjuvant therapy. A) both the statement and the explanation are true and a causal relationship exists between them; B) both the statement and the explanation are true but there is no causal relationship between them; C) the statement is true, but the explanation is false; D) the statement is false, but the explanation itself is true E) both the statement and the explanation are false

A) both the statement and the explanation are true and a causal relationship exists between them Radical prostatectomy means the „en bloc" removal of the prostate and the seminal vesicles. This surgery is indicated only at an early stage (prostate-restricted tumor), so it can not be performed in lymph node positivity. The lymph nodes are removed before surgery or laparoscopically in the first part of the operation and if the frozen section is positive, prostate removal can not be performed.

The cholinesterase inhibitors enhance the salivation, BECAUSE it will cause acetylcholine accumulation at the parasympathetic nerve endings A) both the statement and the reason are true, and the reason verifies the statement B) both the statement and the reason are true ,but there is no relation between them C) the statement is true the reason is false D) the statement is false the reason is true E) both the statement and the reason are false

A) both the statement and the reason are true, and the reason verifies the statement The parasympathetic response enhances the salivary secretion. The sympatholythic drugs, such as the cholinestherase inhibitors enhances the acetylcholine accumulation and in this way can stimulate the salivation.

Even the extraction of a single tooth can lead to severe discrepancy in the articulation BECAUSE the contact point system is disrupted A) both the statement and the reason are true, and the reason verifies the statement. B) both the statement and the reason are true ,but there is no relation between them C) the statement is true the reason is false D) the statement is false the reason is true E) both the statement and the reason are false

A) both the statement and the reason are true, and the reason verifies the statement. Excepting the extraction of the wisdom teeth (no 8) even a single missing tooth can upset the harmonious occlusion leading to a disturbed articulation. One of the reason of this phenomena is the disruption of the contact point system (the contacts between the interproximal tooth surfaces) and the concomitant tooth migration, tilting and over eruption of the involved tooth.

During the mix dentition period even the symptom less non vital deciduous teeth are maintained BECAUSE its space maintenance role is important in the development of the permanent dental arch . A) both the statement and the reason are true, and the reason verifies the statement. B) both the statement and the reason are true ,but there is no relation between them C) the statement is true the reason is false D) the statement is false the reason is true E) both the statement and the reason are false

A) both the statement and the reason are true, and the reason verifies the statement. In case of the deciduous tooth's preterm extraction due to the migration and shifting of the neighbouring teeth the erupting permanent teeth cannot fit properly in the dental arch. Gamed crowding dentition is formed and the tooth erupts either in extra-or intraocclusal position. Therefore in the period of mixed dentition space maintenance appliance should be used.

The leukoplakia is a precancerous lesion on the oral mucosa THERFORE it treatment is mandatory in any cases A) both the statement and the reason are true, and the reason verifies the statement. B) both the statement and the reason are true ,but there is no relation between them C) the statement is true the reason is false D) the statement is false the reason is true E) both the statement and the reason are false

A) both the statement and the reason are true, and the reason verifies the statement. One of the white lesions of the oral cavity , the leukoplakia -depending of its clinical type and histology can lead to cancer formation (3-36%) (the average incidence is 2-6%). Therefore they should be treated by specialists .

The incipient carious lesion can heal by" restitution ad integrum" after topical fluorid therapies BECAUSE the initial demineralization is reversible A) both the statement and the reason are true, and the reason verifies the statement. B) both the statement and the reason are true ,but there is no relation between them C) the statement is true the reason is false D) the statement is false the reason is true E) both the statement and the reason are false

A) both the statement and the reason are true, and the reason verifies the statement. The incipient caries is a superficial demineralisation on the surface of enamel. If the saliva contains sufficient substrates ( calcium, phosphate and fluoride) and the pH in the local environment is beneficial for remineralisation the lesion can heal with "restitution ad integrum"

Which disease is the leading cause of death among 65-74-year olds in Hungary? A) circulatory diseases B) malignant tumors C) accidents D) respiratory tract infections

A) circulatory diseases according to mortality rates, deaths from circulatory diseases are more common among the Hungarian 65 to 74 year-olds compared to individuals over the age of 85 in the EU-15 countries (according to 2009 data: men: 44.5 %, Women: 44.6%).

Rinne's test is informative in: A) conductive hearing loss B) sensorineural hearing loss C) perforation of the tympanic membrane D) sudden hearing loss

A) conductive hearing loss In Rinne test the patient hears the transmitted sound better on the bone due to bone conductivity, rather than in the air. This indicates a conductive hearing loss.

In which of the below listed diseases you would suspect retinal microaneurysm regarding differential diagnosis? A) diabetes mellitus B) retinal branch (venous) occlusion C) hypertensive retinopathy D) hereditary retinal degeneration E) occlusion of retinal artery

A) diabetes mellitus Microaneurysms are most characteristic in retinopathy due to diabetes mellitus. In the capillaries pericytes die and the vessel walls thicken unevently. This process is the first symptom of the microangiopathy in diabetes.

Disorders of voice formation A) dysphonia B) lisping C) sputtering D) stuttering E) aphasia

A) dysphonia Disorders of voice information are collectively called dysphonia.

Symptoms of acut laryngitis do not include A) haemoptoe B) hoarseness C) laryngeal pain D) coughing E) hiperaemic vocal chords

A) haemoptoe Acute laryngitis symptoms does not usually include haemoptoe.

The characteristics of Pierre-Robin-syndrome 1) glossoptosis 2) micrognathia 3) palatoschisis 4) coarctatio aortae A) if 1., 2. and 3. are correct B) if 1. and 3. are correct C) if 2. and 4. are correct D) only the 4th answer is correct E) if all the answers are correct

A) if 1., 2. and 3. are correct The Pierre Robin syndrome (palatoschisis, micrognathia and glossoptosis) is a congenital hereditary condition. Its typical sign is the gothic palate, which frequently associated with palatourano-staphyloschisis . The lower jaw is underdeveloped and the tongue is in a retro position .

The characteristic sign of amelogenesis imperfecta 1) a disturbed enamel development 2) many times the total enamel coverage is missing on the crown of the tooth 3) this affects both the disturbed matrix formation and the calcification 4) it occurs only in the permanent teeth A) if 1., 2. and 3. are correct B) if 1. and 3. are correct C) if 2. and 4. are correct D) only the 4th answer is correct E) if all the answers are correct

A) if 1., 2. and 3. are correct The amelogenesis imperfecta is a developmental disease the affects only the structure of the enamel and not the form of the crown. It is characteristic that the developing enamel is porous, soft, matt. It warns down easily and turns to brown.

The symptoms of dental caries at a vital tooth : 1) sensitive to cold 2) softened dentine 3) it is sensitive to osmotic attack 4) sensitive to percussion A) if 1., 2. and 3. are correct B) if 1. and 3. are correct C) if 2. and 4. are correct D) only the 4th answer is correct E) if all the answers are correct

A) if 1., 2. and 3. are correct The caries depending of its progression shows different symptoms . The early sign of the enamel caries is the sensitivity to osmotic and thermal (cold, hot) stimuli. The dentin caries due to the further demineralisation affects the decalcified structures. Although the pathology involving only the hart tissues of the teeth will not lead to sensitivity to percussion.

What can cause chronic glossitis ? 1) candidasis 2) diabetes mellitus 3) avitaminosis 4) hemophilia A) if 1., 2. and 3. are correct B) if 1. and 3. are correct C) if 2. and 4. are correct D) only the 4th answer is correct E) if all the answers are correct

A) if 1., 2. and 3. are correct The chronic glossitis is much more common than the acute one. Associated with systemic diseases the glossitis cannot be cured without controlling the background disease. The most common causes are: pernicious anemia, vitamin-B deficiency, gastrointestinal ailments candidiasis and diabetes mellitus.

Drugs decreasing salivary flow rate 1) homatropine 2) scopolamine 3) atropine 4) noscapine A) if 1., 2. and 3. are correct B) if 1. and 3. are correct C) if 2. and 4. are correct D) only the 4th answer is correct E) if all the answers are correct

A) if 1., 2. and 3. are correct The tropeins ( a structure made up by the condensation of a pyridine and pirolidine ring) even in therapeutic dose cause xerostomia, dray through, thirst and mydriasis. This sometimes can be used for the transitory inhibition of the salivation. This is very useful during impression taking or special oral exams.

Treatment of tonsillogenic sepsis requires A) immediate tonsillectomy performed with antibiotic protection B) application of broad-spectrum antibiotics C) cryotherapy of the tonsils performed with antibiotic protection D) therapeutic punction of cerebrospinal liquor E) splenectomy

A) immediate tonsillectomy performed with antibiotic protection Tonsillogenic sepsis requires an immediate tonsillectomy and antibiotic treatment.

The prevalence of iodine deficiency is the lowest A) in America B) in Europe C) in Africa D) in Southeast Asia

A) in America Iinadequate iodine supplement is the lowest in America (10%) and the highest in Europe (50%) in the general population. Although the effective anti-iodine preventive intervention, iodising the salt was estabilished in the 1920s, in Europe, even today less than half of the households use iodised salt. Despite the efforts of recent decades, the inadequate intake of iodine remains a major public health problem in Europe, althought the severe mental impairment related to iodine deficiency is very rare.

The importance of community interventions in health promotion is that A) individual behavior is strongly influenced by family and friends B) individual behaviour is determined by family and friends C) designing community level interventions is easier than designing individual level interventions D) collaboration with communities is less expensive than interventions targeting individuals

A) individual behavior is strongly influenced by family and friends Human communities have a significant concious and non-concious impact on the behavior of their members. Individual behaviours which differ from the expected/acted behaviour of the community are not sustainable and, if maintained, often lead to social isolation. Thus, behaviour having a positive effect on health status can be most efficiently developed at community level by helping communities to increase their health awarness.

What is the cause of the gingival enlargement in leukaemia ? A) infiltration by malignant cells B) reactive fibrosis C) haemangioma D) capillary rigidity E) neither of them

A) infiltration by malignant cells The early sign of acute leukemia is the swelling of the interdental gingiva, the spontaneously bleeds ,and finally necrotized creating ulcers. It is caused by the infiltrating malignant hematological cells.

Polychlorinated biphenyls induce (activate?): A) microsomal enzymes of the liver B) glutathione peroxidase C) superoxide dismutase D) catalase

A) microsomal enzymes of the liver Absorbed PCBs induce changes in steroid metabolism and metabolic processes due to the induction of microsomal enzymes in the liver.

For the treatment of a simple rhinitis we recommend A) nasal drops for 5 to 7 days B) antibiotics C) NSAIDs D) corticosteroids

A) nasal drops for 5 to 7 days The simple rhinitis is treatable with 5 to 7 days of nasal drop usage. Other treatment is unnecessary.

Which tobacco ingredient is responsible for the development of smoking-related vascular diseases? A) nicotine B) polycyclic aromatic hydrocarbons C) carbon monoxide D) ammonia

A) nicotine The cardiovascular complications due to smoking are related to the activity of stimulant nicotine, which acts as an acetylcholine agonist.

The venous pulsation within the papilla: A) normal sign B) means enlarged intracranial pressure C) can be observed following retinal vein occlusion D) glaucoma should be suspected / anticipated E) sign of papillitis

A) normal sign The retinal venous pulsation is a normal finding. The pressure in central retinal vein is approximately the same with the intraocular pressure. During systole the intraocular pressure increasing by 1-2 mmHg, by this way the soft vein wall is compressed. This changing with the increasing minimal pressure rise is enough to cause venous pulsation. If venous outflow is blocked, spontaneous vein pulsation stops.

The patient ate fish and has a stiching sensation upon swallowing. First you'll search for the fishbone A) on the tonsils and root of the tongue B) in the piriform sinus C) in the epipharynx D) in the laryngeal vestibule

A) on the tonsils and root of the tongue Fish bone usually gets trapped on the tonsils and the root of the tongue.

Follicular tonsillitis should be treated with A) penicillin for 10 days B) doxycycline for 1 week C) prompt removal of the tonsils D) local administration of medicines (with a swab) on the tonsils and systemic painkillers

A) penicillin for 10 days Follicular tonsillitis is treated with the usage of penicillin for 10 days, to prevent secondary infections.

Adequate measures in case of an esophageal foreign body A) radiography with absorbable contrast media and esophagoscopy B) introduction of an esophageal tube C) dilation of the cardia D) administration of painkillers, spasmolitics

A) radiography with absorbable contrast media and esophagoscopy In case of esophageal foreign body a radiography with absorbable contrast media and esophagoscope must be done.

Which primary prevention strategy involves the early risk/ risk factor detection/identification? A) risk group strategy B) Populational strategy C) Both D) None

A) risk group strategy Population strategy of primary prevention includes preventive interventions for the whole population. There are no target groups here, information or interventions should reach everyone. The principle of the risk factor strategy is to identify individuals or groups with increased risk, and then aim these groups specifically with preventive services. Obviously, this strategy requires risk factor identification in order to select the target group properly.

The leading sign of the chronic osteomyelitis is: A) sequestration B) leukopenia C) fibrinous exsudate D) high fever E) mostly associated with rheumatic complains

A) sequestration The leading sign of the chronic osteomyelitis is the sequestration, the body's chronic response. The other symptoms are characteristic to acute inflammation and or the inflammation of the cortical bone.

The following factor has an important role in the etiology of urothelial cancer: A) smoking B) alcohol C) bensidin D) bensin

A) smoking Smoking plays a role in the development of urothelial carcinomas. Other factors, such as certain polycyclic hydrocarbons also increase the risk of bladder cancer, but the relationship is not as clearly demonstrated as with smoking.

Characteristics of juvenile idiopathic arthritis 1) Begins before the age of 16 2) Growth restriction of the child 3) Reduced mental development of the child 4) Joint pain is generally mild 5) Biological therapy not applicable A) the 1st, 2.nd and 4th answers are correct B) the 1st, 3rd and 4th answers are correct C) the 1st, 4th and 5th answers are correct D) the 1st, 2nd and 3rd answers are correct E) the 1st, 3rd and 5th answers are correct

A) the 1st, 2.nd and 4th answers are correct JIA, featured by mild joint pain, could affect growth, but not mental development, and the use of biological therapy in its treatment could led to revolutionary breakthrough.

Cystography is indicated in: 1) bladder fistula 2) detecting or excluding bladder trauma 3) examine bladder diverticulum 4) diagnosing cystitis A) the 1st, 2nd and 3rd answers are correct B) the 1st and 3rd answers are correct C) the 2nd and 4th answers are correct D) only the 4th answer is correct E) all of the answers are correct

A) the 1st, 2nd and 3rd answers are correct Contrast agent exit can be seen during cystography in bladder injury or in cases of fistulas. Bladder diverticulum is represented by increased contrast agent uptake during cystography. In the diagnosis of cystitis, anamnesis and urine sediment controll (pyuria) is mostly sufficient, and if necessary (mostly in specific types) cystoscopy can be performed.

Cause of acute anuria: 1) fungal infections 2) methyl alcohol 3) ethylene glycol 4) sodium hydroxide A) the 1st, 2nd and 3rd answers are correct B) the 1st and 3rd answers are correct C) the 2nd and 4th answers are correct D) only the 4th answer is correct E) all of the answers are correct

A) the 1st, 2nd and 3rd answers are correct Ethylene glycol (antifreeze liquid), certain mushroom poisoning and methyl alcohol may cause acute renal anuria based on nephrotoxicity.

Inflammations with fever: 1) acute pyelonephritis 2) acute prostatitis 3) acute epididymitis 4) acute cystitis A) the 1st, 2nd and 3rd answers are correct B) the 1st and 3rd answers are correct C) the 2nd and 4th answers are correct D) only the 4th answer is correct E) all of the answers are correct

A) the 1st, 2nd and 3rd answers are correct Only urological organs having parenchyma can cause fever, so acute cystitis does not.

What tests can detect prostate cancer? 1) PSA 2) DRE (digital rectal examination) 3) biopsy 4) abdominal CT A) the 1st, 2nd and 3rd answers are correct B) the 1st and 3rd answers are correct C) the 2nd and 4th answers are correct D) only the 4th answer is correct E) all of the answers are correct

A) the 1st, 2nd and 3rd answers are correct PSA (prostate-specific antigen) is prostate and non-prostatic cancer specific. Its normal level is 0 to 4 ng / ml, it is uncertain at 4-10 ng / ml, it is more likely tumor if it is more than 10 ng / ml. With the RDE examination the cartilage- hard nodule raises the suspicion of malignant prostate disease, but a certain diagnosis can only be established based on histology. In advanced case abdominal CT only shows the larger lymph nodes, but for the finer structure TRUS (transrectal ultrasound) is needed, but it is not as specific as RDV and PSA together. So the first step is therefore a rectal examination and after palpating a suspecious finding PSA level should be measured. (2 days intermission should be between the blood test and the RDE, because palpation affects the PSA level). Prostate biopsy is always mandatory in case of suspected prostate cancer.

In the andrological examination the following hormone levels are measured: 1) testosterone 2) FSH 3) LH 4) Parathormone A) the 1st, 2nd and 3rd answers are correct B) the 1st and 3rd answers are correct C) the 2nd and 4th answers are correct D) only the 4th answer is correct E) all of the answers are correct

A) the 1st, 2nd and 3rd answers are correct Testosterone is the male sex hormone, LH is its trophormon, and FSH stimulates spermatogenesis (they are measured in case of hypogonadism). We are measuring parathormone in potential hyperthyroidism.

Which drugs are used to treat osteoporosis from the following? 1) Bisphosphonates 2) Teriparatide 3) Hyaluronic acid 4) Denosumab against RANKL 5) Chondroitin sulphate A) the 1st, 2nd and 4th answers are correct B) the 1st, 3rd and 5th answers are correct C) the 1st., 2nd and 5th answers are correct D) the 1st, 3rd and 4th answers are correct E) the 2nd, 4th and 5th answers are correct

A) the 1st, 2nd and 4th answers are correct Bisphosphonates, teriparatide and denosumab are innovative therapeutic agents in osteoporosis; hyaluronic acid and chondroitin sulphate are used to treat osteoarthritis.

Complications of a middle ear choleoksteatoma may be 1) labyrinthitis 2) cavernous sinus thrombosis 3) facial nerve palsy 4) recurrent laryngeal nerve palsy 5) subperiosteal abscess A) the 1st, 2nd, 3rd and 5th answers are correct B) the 2nd, 3rd and 4th answers are correct C) the 1st, 2nd and 5th answers are correct D) the 1st and 3rd answers are correct E) the 4th and 5th answers are correct

A) the 1st, 2nd, 3rd and 5th answers are correct Middle ear cholesteatoma complications are labyrinthitis, cavernous sinus thrombosis, facial nerve palsy, and subperiosteal abscess.

Slow-growing lump on the neck may be the sign of 1) cervical metastasis 2) non-Hodgkin-lymphoma 3) struma 4) pulsion diverticulum 5) cervical cyst A) the 1st, 2nd, 3rd and 5th answers are correct B) the 2nd, 3rd and 4th answers are correct C) the 3rd, 4th and 5th answers are correct D) the 1st and 4th answers are correct

A) the 1st, 2nd, 3rd and 5th answers are correct Slow growing lump on the neck may be a sign of cervical metastasis, non-Hodgkin lymphoma, struma or cervical cyst.

Which of the following can be used to separate kidney cyst from tumor? 1) ultrasound 2) IVP (Intravenous pyelogram) 3) CT 4) retrograde pyelography 5) angiography A) the 1st, 3rd and 5th answers are correct B) the 1st, 4th and 5th answers are correct C) the 2nd, 3rd and 4th answers are correct D) the 2nd, 4th and 5th answers are correct

A) the 1st, 3rd and 5th answers are correct US and CT can separate the solid surface from the fluid-containing cystic formula according to their echogenicity and density. Angiography shows abnormal vascular strucure in the tumor, but in the cyst is not to be seen. Retrograde pyelography and excretory urography only shows the volumen, but no differential diagnosis can be made.

What is the most important determinant factor in the financing of general practices in Hungary? A) the number of patients registered at the GP B) Number of actual doctor-patient encounters C) quality of the provided primarycare D) Preventive services (and their quality) performed by the general practitioner

A) the number of patients registered at the GP GP practices are financed by a per capita basis in Hungary, so the financing is based on the number of registered persons. Nevertheless, a degression can be found in financing if the registered patients number exceeds a certain limit.

Why do we not see pus cells in the urine which was proved to be pus positive with KOH test? A) the urine is alkaline B) besides the pus cells, there are also bacterial cells too C) the urine is acidic D) there are crystals in the urine

A) the urine is alkaline In strongly alkaline urine, cellular elements fall apart, so no white blood cells can be seen under a microscope.

The most important urgent thing to do in case caustic (chemical) damage of the surface of the eye: A) to remove the caustic materials and to wash out the eye with ample water B) antibiotic eye drops hourly C) subconjunctival corticosteroid injection D) to pad the eye E) referral to hospital (inpatient)

A) to remove the caustic materials and to wash out the eye with ample water In case of caustic (chemical) injury of the ocular surface, the most important is to remove the caustic agent from the eye with ample water irrigation. It is very important to examine the upper fornix. To evert the upper eyelid is possible only with the Desmarres equipment. With only a glass stick, the fornix cannot be visualized.

What can develop in the middle ear due to short-term obstruction of the Eustachian tube? A) transsudate B) haematoma C) ympanosclerosis D) acute inflammation E) exudate

A) transsudate Obstruction of the Eustachian tube frequently causes transudation in the tympanic cavity.

Stroboscopy is used in otorhinolaryngology for the evaluation of A) vocal fold vibration B) paranasal sinuses C) esophageal motions D) nasal breathing

A) vocal fold vibration Stroboscope is used for the evaluation of the vocal fold vibration.

The symptoms of sinus apertus: 1) when the patient rinses the mouth the water flows out through the nose 2) strong arterial bleeding 3) when blowing the patient's nose the air flows through the perforation into the oral cavity creating a whistling noise 4) an extensive hematoma A) 1st, 2nd and 3rd answers are correct B) 1st and 3rd answers are correct C) 2nd and 4th answers are correct D) only the 4th answer is correct E) all of the answers are correct

B) 1st and 3rd answers are correct The characteristic feature of the sinus apertus is the antro-oral communication - the water/air pathological communication between the to cavities. The air can enter into the oral cavity from the nose, and the water from the oral cavity into the nose. Hematoma is not a common characteristic sign.

The optimal fluoride content of table salt A) 1,25 mg/kg B) 250 mg/kg C) 25 mg/kg D) 50 mg/kg E) 150 mg/kg

B) 250 mg/kg In the Hungarian cuisine 250mg/kg fluoridated table salt has an equivalent protective effect like a 1mg/l fluoridated water. The 150mg/kg content is only a supplement and the lower doses have no caries protective effects.

In a patient with prolonged hoarseness (dysphonia) you have to perform a laryngoscopy, or even a histological test after A) 8 days B) 3 weeks C) 8 weeks D) 3 months E) 1 year

B) 3 weeks Persistent hoarseness of more than 3 weeks suggest a visit to the ENT for a laryngoscopy, whereupon small alteration on the area can be manageable.

Intratracheal intubation should be replaced by tracheostomy after A) 6 hours B) 48 hours C) 8 days D) 4 weeks E) it should not be replaced at all

B) 48 hours Tracheostomy is indicated 48 hours after intratracheal intubation. Longer waiting then the suggested can cause laryngeal damage.

How many active family doctors work in adult basic care in Hungary approximately? A) 1000 B) 5000 C) 10000 D) 20000

B) 5000 Approximately 5,000 general practitioners (adult family doctors?) (and about 1,500 children family doctor) work in Hungary. Their number is almost identical to the number of practice districts, as the number of vacant practices in recent years has been around 150-200. In the near future, primary care will be severely challenged by the high rate of retirement of elderly GPs. The situation may be worsened further by the , increasing number of young doctors leaving the country (i.e. by the decrease of refill).

Which of the following substances is/are responsible for the carcinogenic effect of smoking? 1) N'-nitrosonornicotine 2) nicotine 3) benzo(a)pyrene 4) carbon monoxide A) Answers 1, 2 and 3 are correct B) Answers 1 and 3 are correct C) Answers 2 and 4 are correct D) Only answer 4 is correct E) All of the answers are correct

B) Answers 1 and 3 are correct Chemical components of tar are responsible for the carcinogenic effects of smoking, including nitrosamines (eg nitrosonornicotine) and benzpyrene, which have been declared carcinogens based on data obtained from International Agency for Research on Cancer (IARC).

Which metabolite detected in the urine refers to benzene exposure during biomonitoring? 1) phenolic acid 2) hippuric acid 3) muconic acid 4) ortho-cresol A) Answers 1, 2 and 3 are correct B) Answers 1 and 3 are correct C) Answers 2 and 4 are correct D) only answer 4 is correct E) all 4 answers are correct

B) Answers 1 and 3 are correct Periodic medical examination of those working under benzene exposure involves biomonitoring based on metabolites that includes measuring the urinary concentrations of phenol and muconic acid.

Which pesticides inhibit/block the activity of acetylcholinesterase? 1) methylcarbamates 2) chlorinated hydrocarbons 3) organic phosphate esters 4) phenol derivatives A) Answers 1, 2 and 3 are correct B) Answers 1 and 3 are correct C) Answer 2 and 4 are correct D) only answer 4 is correct E) all 4 answers are correct

B) Answers 1 and 3 are correct The carbamate pesticides inhibit of acetylcholinesterase reversibly, while the organic phosphate esters inhibit the enzyme irreversibly. Therefore, carbamate and thiocarbamate poisoning are generally less severe than poisoning caused by organic phosphate esters, but both are dose-dependent.

Epidemiological studies proving the relationship between being overweight/obese and the onset of coronary heart disease: 1) North Karelia project 2) Pritkin-Ornish 3) Framingham Heart Study 4) National Cholesterol Education Committee A) Answers 1, 2 and 3 are correct B) Answers 1 and 3 are correct C) Answers 2 and 4 are correct D) only answer 4 is correct E) all 4 answers are correct

B) Answers 1 and 3 are correct The relationship between being overweight/obese and the onset of coronary heart disease was confirmed by the North Karelia project and the Framingham Heart study. The National Cholesterol Education Committee deals with the risk-increasing effects of cholesterol in the aspect of causing coronary heart disease, whereas the name of the most restricted fat intake reducing diet is Pritkin-Ornish.

Which chemotherapeutic agent is used most commonly for treatment of a malignant melanoma? A 50-year-old male presents with a bleeding, pruritic, exophytic, ulcerated, brownish black, nodular tumor of 1 cm diameter over the left scapula which has grown quickly in the last half a year. A) Azathioprine B) Dacarbazine C) Bleomycin D) Cyclophosphamide

B) Dacarbazine

Which of the following cells belongs to the professional antigen presenting cells of the cutaneous adaptive immune system? A) Keratinocyte B) Langerhans cell C) Merkel cell D) Melanocyte

B) Langerhans cell

NOT characteristic of ankylosing spondylitis: A) Elevated erythrocyte sedimentation rate B) Low back pain worsens after activity, improves after rest C) Low back pain worsens during early morning rest, improves with activity D) Early bilateral sacroiliitis E) Syndesmophytes on spinal X-ray

B) Low back pain worsens after activity, improves after rest Inflammatory spinal pain typical to Bechterew's disease increases in rest, relieves with motion, oppositely to degenerative back pain which increased after motion.

All following disorders are manifestations of cutaneous tuberculosis, except: A) Lupus vulgaris B) Lupus erythematosus C) Erythema induratum Bazin D) Scrofuloderma E) Lichen scrofulosorum

B) Lupus erythematosus

What is the most likely differential diagnosis? A 50-year-old male presents with a bleeding, pruritic, exophytic, ulcerated, brownish black, nodular tumor of 1 cm diameter over the left scapula which has grown quickly in the last half a year. A) Kaposi sarcoma B) Nodular malignant melanoma C) Squamous cell carcinoma D) Mycosis fungoides

B) Nodular malignant melanoma

The four-player model of healthcare services is extended with one of the following participants compared to the two-player everyday consumer model: A) Provider B) Politics (state) C) Both D) None

B) Politics (state) Consumers and service providers are the two main participants of common consumer models. Obviusly these two participants are present in the field of health services as well. However, the health care consumer models are complemented with two other participants, namely with the financier (usually the health insurer, as the services are usually not obtained by the consumers directly) and wih the politics. Politics is the expression of state's will related to healthcare.

Which is the most common autoimmune disease among the following? A) Systemic lupus erythematosus B) Rheumatoid arthritis C) Systemic sclerosis D) Sjögren syndrome E) Dermatomyositis

B) Rheumatoid arthritis Prevalence of RA (0.5-1%) far exceeds other autoimmune disorders' prevalence.

In which of the below mentioned diseases causing dry eye syndromes? A) ankylopoietic spondylarthrosis B) Sjögren syndrome C) paresis of the facial nerve D) Horner syndrome

B) Sjögren syndrome Dry eye is a symptom in Sjögren syndrome.

Who disposesthe alpha fetoprotein (AFP) test of a pregnant woman, when and for what purpose? A) The pysician will dispose the test on the 12th week for the screening of the closure dosorders of the neural tube B) The pysician will dispose the test on the 16th week for the screening of the closure dosorders of the neural tube C) The health visitor will dispose the test on the 16th week for the screening of the closure dosorders of the neural tube. D) The health visitor will dispose the test on the 18th week for the screening of the closure dosorders of the neural tube.

B) The pysician will dispose the test on the 16th week for the screening of the closure dosorders of the neural tube According to the decree 33/1992 regarding the prenatal care of pregnants, serum alpha-fetoprotein test is performed during the 16th week of pregnancy. The test is ordered by the physician who is responsible for carrying the prenatal care.

In which case are the interests of the provider and the insurer contrary in the field of healthcare? A) the amount of contribution paid by the insured person B) The spectrum of services offered by the service provider facilites C) Both D) None

B) The spectrum of services offered by the service provider facilites The healthcare provider is generally not interested in the question who is paying for its service. Furthermore, it not interested in the amount of self payment of the customer for receiving adequate service (of course, a too high self payment is not in the interest of the provider as it decreses the requisition of the service). Rather, the provider and the insurer have contrary interest in the spectrum of the services offered. Interest of the service provider is to offer as many types of service as possible (because it sells more and gets more reimbursed), while the insurer prefers a narrower range of the services because it has to pay less for less service.

Which drug's characteristic side effect is the dry mouth syndrome (Xersotomia) A) Lidocain (lidocainium chloratum anhydricum) B) Troparin (papaverinium sulfuricum + methylhomatropinum bromatum) C) Papaverinum (papaverinium chloratum) D) Eunoctin (nitrazepamum) E) Tarivid (ofloxacinum)

B) Troparin (papaverinium sulfuricum + methylhomatropinum bromatum) The Troparine's side effect is the xerostomia. The other listed drugs have no effect on the salivation

Sialolithiasis A) is most common in the parotid gland B) affects most frequently the submandibular gland C) is detected mostly in the sublingual gland D) can be effectively treated with a special diet

B) affects most frequently the submandibular gland Sialolithiasis affects most commonly the submandibular gland, and the resolution is the removal of the gland in most of the cases.

Holzknecht-Jacobson sign indicates that A) both sides of the lung functions equally B) an unilateral bronchial foreign body causes the mediastinum move towards the affected side C) a pulmonary emphysema is present D) a pulmonary tumor giving radiography signs is detected E) radiographic contour of the mediastinum is widened

B) an unilateral bronchial foreign body causes the mediastinum move towards the affected side A unilateral bronchial foreign body causes the mediastinum move towards the affected side, whereas this sign is called Holzknecht-Jacobson.

Active immunization is used to prevent the following diseases: 1) meningitis epidemica 2) scarlet fever 3) pneumococcal meningitis 4) legionellosis A) answers 1, 2 and 3 are correct B) answers 1 and 3 are correct C) answers 2 and 4 are correct D) only answer 4 is correct E) all 4 answers are correct

B) answers 1 and 3 are correct Active immunization is used against meningitis epidemica, which is mandatory in certain occupations (eg laboratory technicians), otherwise it is recommended, especially for those who are more susceptible. Active immunization is available against pneumococcus meningitis (free of charge for those younger than 2 years old) which is highly recommended for those with increased risk.There is no active immunization against scarlet fever and legionellosis.

Analytical epidemiological studies 1) most common types are the case/control and the cohort studies 2) Testhypothetical relationships using aggregated or individual data 3) evaluate the role of significance of etiological factors 4) Analyze data obtained from observation or generated by experimental procedures. A) Answers 1, 2 and 3 are correct B) answers 1 and 3 are correct C) Answers 2 and 4 are correct D) only answer 4 is correct E) all 4 answers are correct

B) answers 1 and 3 are correct Analytical studies mainly focus on "why" type questions: they examine and test hypothetical relationships, identify the influencing (risk or preventive) factors, measure the role (significance) of the risk factors in the development of the phenomenon (risk). The two basic types of analytical epidemiological studies are the case/control and cohort studies, which primarily aim is to elucidate the etiological background of the diseases (occurring events)

Which of the following four heavy metals has/have human mutagenic effect? 1) cadmium 2) cobalt 3) chrome 4) lead A) answers 1, 2 and 3 are correct B) answers 1 and 3 are correct C) answers 2 and 4 are correct D) only answer 4 is correct E) all of the answers are correct

B) answers 1 and 3 are correct Cadmium and chromium have mutagenic effect in humans, while cobalt and lead do not have any mutagenic effect in humans.

Benzene: 1) damages the bone marrow and causes aplastic anaemia 2) its vapors mainly damage the lungs 3) due to its chronic effect acute myeloid leukemia may develop 4) causes skin burns A) answers 1, 2 and 3 are correct B) answers 1 and 3 are correct C) answers 2 and 4 are correct D) only answer 4 is correct E) all of the answers are correct

B) answers 1 and 3 are correct Chronic benzene exposure damages the hematopoietic system, causes aplastic anemia and reversible pancytopenia. Although the production of different cell types is damaged at different level, all hematopoietic changes may indicate benzene toxication. The most serious complications of chronic benzene exposure are the acute or possibly chronic myeloid leukemia.

The most specific palpation sign of prostate cancer is: A) enlarged, pasta like B) cartilage hard C) enlarged, painful D) enlarged, painless

B) cartilage hard A typical palpation found in prostate cancer is a insensitive hard noudule, or a an insensitive, stony uneven surfaced prostate. Acute prostatitis is associated with high fever, a highly sensitive, enlarged prostate. In the case of BPH, we can palpate a glandular or muscular, painless, enlarged prostate.

The IPV is characterized by the followings: 1) it contains inactivated viruses 2) it is an oral vaccine 3) the last vaccine is being given to children of 6 years old in Hungary 4) the last vaccine is being given to children of 15 months old in Hungary A) Answers 1, 2 and 3 are correct B) answers 1 and 3 are correct C) Answer 2 and 4 are correct D) only answer 4 is correct E) all 4 answers are correct

B) answers 1 and 3 are correct IPV is a vaccine containing inactivated polyoviruses which should be given intramuscularly. The administration of the vaccine (containing also other components) takes place in age-related manner according to the vaccination regulation is performed that is effective in Hungary since 1 January 2010. The vaccine contains other components as well. The vaccine containing the components of DTPa (diphtheria, tetanus, acellularis pertussis), IPV (inactivated polyovirus vaccine) and Hib (haemophilus influenzae b) together must be given to childrem of 2, 3, 4 and 18 months afe, while DTpa and IPV vaccination must be given at the age of 6 years (this is the last IPV vaccination).

Carcinogenic nutritional ingredients / factors: 1) N-nitroso compounds 2) flavonoids 3) low-fibre diet 4) beta-carotene A) answers 1, 2 and 3 are correct B) answers 1 and 3 are correct C) answers 2 and 4 are correct D) only answer 4 is correct E) all of the answers are correct

B) answers 1 and 3 are correct N-nitroso compounds, which are formed in the intestinal tract from nitrites and nitrates, may be involved in the development of esophageal, gastrointestinal, bowel and bladder cancers. Low-fibre diet is a factor that increases the risk of developing colon cancer. Flavonoids and beta-carotene have a protective effect.

Which are the characteristics of London-type smog? 1) It is reducing type smog. 2) Nitrogen oxide plays a role in its mechanism. 3) Sulfur dioxide plays a role in its mechanism. 4) UV radiation is required to its formation. A) answers 1, 2 and 3 are correct B) answers 1 and 3 are correct C) answers 2 and 4 are correct D) only answer 4 is correct E) all 4 answers are correct

B) answers 1 and 3 are correct The London-type smog is considered as reducing smog, the following chemical components play roles in its mechanism of action: carbon dioxide, carbon monoxide, sulfur dioxide. Nitrogen oxide and UV radiation are not involved in thise mechanism, these component are the preconditions of the Los Angeles-type oxidative smog.

Which of the following statements applies to the solidarity-type insurance system? 1) Ensures equal opportunities better compared to the risk system 2) It is profit-oriented system 3) It does not encourage participants to maintain disease-preventive lifestyles 4) This system provides health services of the highest quality A) answers 1, 2 and 3 are correct B) answers 1 and 3 are correct C) answers 2 and 4 are correct D) only answer 4 is correct E) all 4 answers are correct

B) answers 1 and 3 are correct The essence of an insurance system based on solidarity (also called Bismarck's insurance system), is that everyone pays contribution in proportion to their income, and when they are sick, they receive care that meet their needs. The system is therefore not for-profit and basically offers equal possibilities to the contributors, as the benefit does not depend on the amount of the contribution made. At the same time, contributors have no interest in health preserving lifestyle because they have no financial benefits in regard to neither contributions paid nor services used. The system is unable to provide the most up-to-date (and most expensive) care options for all, as this can not be financed by the contributions.

What is the relationship between the level of education and the health status? 1) The health status of higher educated persons is better than the health status of lower educated persons. 2) Higher educated people are less likely to visit their doctor in severe complaints than those with lower education level. 3) Higher education means a higher comprehensive capacity, therefore it facilitates the access to disease prevention information. 4) The proportion of smokers is higher among higher educated people. A) answers 1, 2 and 3 are correct B) answers 1 and 3 are correct C) answers 2 and 4 are correct D) only answer 4 is correct E) all of the answers are correct

B) answers 1 and 3 are correct The level of education, like the social status, is an important influencing factor of health status, as it has a positive effect on most indicators describing health status. The positive effects of education on health status are achieved through various mechanisms: higher educated people acquired more (background) knowledge during their studies; by the help of their comprehensive skills they can find and understand better health-related knowledge; because of their wider access to resources, they can reduce their health risks better and, if necessary, have access to health services relatively easier.

Swallowed foreign bodies most commonly get stuck A) in the hypopharynx B) at the first isthmus of the esophagus C) at the second isthmus of the esophagus D) where the esophagus opens into the stomach E) at the pylorus

B) at the first isthmus of the esophagus Swallowed foreign bodies usually get stuck at the first isthmus of the esophagus. These are usually easy to remove.

In narrow (closed) angle glaucoma which of the below mentioned medication is contraindicated? A) betoptic B) atropine C) acetazolamide D) pilocarpin

B) atropine In case of angle closure glaucoma pupillary dilation with atropine is contraindicated.

Too much iodine intake: A) causes hypothyroidism B) can induce hyperthyroidism C) causes cretenism D) causes obesity

B) can induce hyperthyroidism Too much iodine intake can cause hyperthyroidism; especially in the long-defective areas, it was found that at the beginning of iodine supplementation thyrotoxicosis rarely occured (so-called iodo-Basedow disease) in individuals over 40. Iodine is an essential component of thyroid hormones. In the absence of iodine growth- and developmental disorders, deafness, damage to the nervous system, slowing of mental activity, apathy could develop. Severe iodine deficiency during pregnancy and during early childhood causes developmental disorders and severe psychomotor disoerders. Cretens are characterized by severe mental retardation. Increasing metabolism due to hyperthyroidism can lead to weight loss

The patient suffers from hearing loss on the right side. Weber test is lateralized to the right side. Probable cause of the hearing loss is: A) otosclerosis on the left side B) cerumen on the right side C) Ménière's disease on the right side D) sensorineural hearing loss on the right side

B) cerumen on the right side Weber test is heard louder to the affected side due to conductive hearing loss in the defective ear. The reason of conductive hearing loss most of the times is the existence of earwax.

The testicular torsion: A) is always on both sides B) children most likely wake up because of scrotal pain C) immediately causes a high fever D) is more common on the right side

B) children most likely wake up because of scrotal pain The testicular torsion mostly occurs in young boys. It usually starts with a suddenly appearing, very severe testicular pain radiating towards the same side flank. High fever can only be caused if the testicles are already necrotized. It appears with the same frequency on both sides.

The most important early characteristics of hypertensive retinopathy: A) arteriovenous crossing B) constriction of arterioles C) widening of the light reflex of the arterioles D) copper wire arterioles E) silver wire arterioles

B) constriction of arterioles Hypertension starts with the constriction of the smallest vessels, this sign is specific. The other signs can be found not exclusively in hypertension and occurs in the later phase of hypertension.

What are the main points of Bismarck's health insurance system? A) Health-related costs/expenditure are covered by taxes; the care is based on citizenship and belongs to everyone (civil right) B) contribution is in relation to income, care is based on needs; the care is granted for those who are paying the contribuition C) contribution and chosen service are defined in a contract; care is granted to the insured person D) None of them

B) contribution is in relation to income, care is based on needs; the care is granted for those who are paying the contribuition In Bismarck's health insurance system, insured persons pay a contribution (in proportion related to their income) and therefore receive care according to their needs. The system is based on the principles of solidarity (for the insured), because the care does not depended on the amount of contribution paid but the entitlement is granted by the fact of paying contribution). The Beveridge system is characterized by the theorem that health care costs are financed by taxes and care is granted for everyone. In the case of risk insurances, the contribution payment is related to the risk and the services chosen.

In which typical disease complains the patient pain when touching his scrotum? A) testicular tumor B) epididymitis C) hydrocele D) varicocele

B) epididymitis In the case of epididymitis, the patient's epididymides and testicles are enlarged, and feels painful to touch. In case of testicular cancer, a painless nodule or the painless enlargement of the testis can be detected. Varicocele can cause dragging-like or aching pain within the scrotum, enlarged veins are palpable or to be seen, and does not feel painful when touched. The hydrocele is a transparent, painless, and fluctuating fluid can be detected.

Tenderness with pressure on the tragus or traction on the auricle is typical in: A) othematoma B) external otitis C) mastoiditis D) otosclerosis

B) external otitis Tenderness with pressure on the tragus or traction on the auricle is characteristic in cases of external ear infections.

Surgical solution of a median cervical cyst or fistule is A) extirpation of the fistule together with the hyoid bone B) extirpation of the fistule and the cyst together with the middle section of the hyoid bone C) extirpation of the fistule up to the root of the tongue

B) extirpation of the fistule and the cyst together with the middle section of the hyoid bone Surgical solution of the median cervical cyst or fistula is the extirpation of it together with the middle section of the hyoid bone, to prevent the recurrence of it.

It is a characteristic disease located on the soft palate: A) aphthous stomatitis B) herpangina C) flue D) morbilli ( measles) E) pertussis (whooping cough )

B) herpangina Only the vesicles of herpangina are located solely on the soft palate. The oral lesion associated with Morbilli (see FOG-1). The common flue and pertussis (whooping cough) have no characteristic oral symptoms. The aphthous stomatitis does not cause vesicle, it is an ulcer located on the lip, buccal mucosa the tongue or soft palate.

What is the most common cardiovascular disease among 65+ in Hungary? A) unstable angina B) high blood pressure C) stroke D) heart attack

B) high blood pressure According to data obtained from the Hungarian General Practitioners' Morbidity Sentinel Stations Program (GPMSSP), a very high incidence o high blood pressure (60-70%) can be seen among Hungarians aged 65+.

The characteristic signs of Down-syndrome 1) hyper salivation 2) microglossia 3) small teeth 4) very wide upper dental arch A) if 1., 2. and 3. are correct B) if 1. and 3. are correct C) if 2. and 4. are correct D) only the 4th answer is correct E) if all the answers are correct

B) if 1. and 3. are correct Down's syndrome is a chromosomal aberration. Its general syndromes associate with characteristic facial features. Macroglossia, hypopastic teeth, and at least in 90% orthodontic abnormalities -like open bite, cross bite occur.

The symptoms of Arthrosis temporomandibularis 1) compromised opening movement 2) swollen joints 3) terminal crepitation 4) deviation towards the diseased joint A) if 1., 2. and 3. are correct B) if 1. and 3. are correct C) if 2. and 4. are correct D) only the 4th answer is correct E) if all the answers are correct

B) if 1. and 3. are correct The degenerative processes associated with swelling. The opening of the mouth is compromised and the forced opening movement leads to pain. Histologically the destruction of the cartilage, calcification and sever other structural destructions can be detected. The crepitation is the most prominent early morning.

Where is the number of suicide-related deaths the highest among individuals over 65? A) In the capital (Budapest) B) in the Northern and Southern Great Plains C) in Northern Hungary D) in Western Transdanubia

B) in the Northern and Southern Great Plains The highest suidice rates could be observed among men and women over the age of 65 years in the two regions of the Great Plain.

During the treatment of a nasal furuncle always avoid A) hospitalization and bed rest B) incision and pressing (squeezing) of the lesion C) parenteral antibiotic therapy D) application of warm compresses

B) incision and pressing (squeezing) of the lesion Nasal furuncle should not be incised or squeezed. Aggressive treatment on it can cause thrombophlebitis and further infection. Recommended treatment is antibiotics.

In an aphakic state following cataract operation which of the below listed provides the best correction? A) contact lens B) intraocular artificial lens C) spectacles D) refractive surgery (PRK)

B) intraocular artificial lens The intraocular artificial lens gives the most accurate imaging of the objects in the retina. If the refraction of the artificial lens equals to the refraction of the removed crystalline lens, the image produced in the retina will have the same size as the (preoperative) original size of the image.

What is specific for the female urine A) it has higher pH level B) it contains a lot of epithelial cells C) it contains always some red blood cells D) it contains always some pus cells

B) it contains a lot of epithelial cells Epithelial cells are often detected in female urine. These are mostly derived from the vagina.

What is characteristic of the so called cariogenic plaque? A) It can develop on any tooth surface B) its dominant bacterial flora is organized from lactobacilli and acidifying streptococci C) it is bacterium-free D) its pH is always above 6.3 E) in can be recognized by naked eye

B) its dominant bacterial flora is organized from lactobacilli and acidifying streptococci Cariogenic plaque can only be formed on the so called non self-cleaning tooth surfaces. The plaque contains several different microbial species that sometimes can only be detected by plaque disclosing agents.

An elder patient presents with unilateral sanguinous, purulent, foul-smelling nasal discharge. You think of: A) frontal sinusitis B) malignant tumor of the nose or a paranasal sinus C) ozaena D) infected dental cyst E) angiofibroma

B) malignant tumor of the nose or a paranasal sinus Unilateral sanguinous, purulent, foul-smelling nasal discharge indicates us to a possibility of malignant tumor in the nose or paranasal sinuses.

The best way to prevent the development of folic acid deficiency diseases: A) folic acid supplement B) mandatory enrichment of bread flour with folic acid C) voluntary fortification of folic acid for certain cereal products D) varied diet

B) mandatory enrichment of bread flour with folic acid Folic acid is a general methyl donor involved in the synthesis of nucleic acids and amino acids as an enzyme cofactor. In the absence of folic acid, severe disorders in the homocysteine-methionine metabolism and damage of the cell proliferation occur. Folic acid deficiency during pregnancy cause an increased risk of closure abnormalities of the neural tube as well as fetal growth retardation, resulting in low birth weight. To prevent the consequences of hyperhomocysteinemia, it is recommended to consume folic acid fortified foods for the whole population. In order to reduce neural tube closure disorders in recent years fortification of flour with folic acid is mandatory in 60 countries, regarding the fact that almost half of the pregnancies are not anticipated and, consequently, the folic acid supplementation may start late.

Which age and gender has the highest suicidal risk in Hungary? A) men under 65 years B) men over 65 years C) women under 65 years D) women over 65 years

B) men over 65 years The risk of suicide in men over the age of 65 is the highest in Hungary; According to data of 2009, suicide rate was more than two and a half times higher than the EU average of the same age group and more than twice as high as the rate of deaths from suicide among Hungarian men under the age of 65.

Which of the below listed entities not leading to the medial concha? A) medial ethmoideal cells B) nasolacrimal duct C) maxillary sinus D) frontal sinus

B) nasolacrimal duct The nasolacrimal duct entering the lower concha.

If you observe in front of the retina a semicircle, in the upper part a horizontal shape hemorrhage during funduscopy, what do you think? A) superficial intraretinal hemorrhage B) preretinal hemorrhage C) deep intraretinal hemorrhage D) vitreous hemorrhage E) chorioideal hemorrhage

B) preretinal hemorrhage It is preretinal bleeding. The shape is horizontal in only that cases when the bleeding is between the retina and the detached posterior vitreous.

The following reflexes are needed to watch nearby objects, except: A) convergence B) pupillary dilation C) contraction of the ciliary muscle D) thickening of the crystalline lens E) enlarged refraction

B) pupillary dilation To watch nearby object, accommodation is needed, in that case the pupil is not dilating, on the contrary it is narrowing.

The term "unilateral radical neck dissection" means A) removal of the metastatic-suspect lymph nodes on one side of the neck B) removal of all cervical lymph nodes together with sternocleidomastoid muscle and internal jugular vein and nervus accessorius on one side C) removal of jugular lymph nodes together with the larynx D) removal of cervical fat tissues and cervical lymph nodes

B) removal of all cervical lymph nodes together with sternocleidomastoid muscle and internal jugular vein and nervus accessorius on one side Unilateral radical neck dissection indicates removal of all cervical lymph nodes together with SCM muscle and internal jugular vein and nervus accessorius on the same side.

When feeling a painless node in the testis the next step should be: A) observation, making up a next consultation B) send him to urology C) antibiotics and control D) investigation because of potential chronic prostatitis

B) send him to urology The painless palpable nodule in the testicles is suspected of tumor, so urological examination is recommended.

To prevent superficial bladder tumors bladder installation can be performed with: 1) BCG 2) Bleomycin 3) Mitomycin C 4) Vincristin A) the 1st, 2nd and 3rd answers are correct B) the 1st and 3rd answers are correct C) the 2nd and 4th answers are correct D) only the 4th answer is correct E) all of the answers are correct

B) the 1st and 3rd answers are correct Bleomycin and Vincristin are suitable for systemic chemotherapy. For the instillation treatment of superficial bladder tumors only Mitomycin C, BCG, Adriamycin and Epirubicin can be used.

Which factors have negative effect on potency? 1) diabetes 2) hyperthyroidism 3) beta blockers used in hypertension 4) antidiabetics A) the 1st, 2nd and 3rd answers are correct B) the 1st and 3rd answers are correct C) the 2nd and 4th answers are correct D) only the 4th answer is correct E) all of the answers are correct

B) the 1st and 3rd answers are correct Diabetes mellitus causes in a vascular and in a neurogenic way damage to potency. Βeta-blockers may lead to erectile dysfunction due to reduced arterial flow.

Ultrasound 1) is better than intravenous urography to detect kidney tumors 2) is better than intravenous urography to detect urothelial tumor 3) is better than CT to detect prostate diseases 4) in the urological ultrasound examinations the 15000-20000 Mhz frequency is used A) the 1st, 2nd and 3rd answers are correct B) the 1st and 3rd answers are correct C) the 2nd and 4th answers are correct D) only the 4th answer is correct E) all of the answers are correct

B) the 1st and 3rd answers are correct Intravenous urography shows the urinary system, which is more informative than US in urothelial tumors. To detect changes in parenchyma (kidney tumor), US is much better. TRUS is considerably cheaper than CT but provides less information about the prostate. Urological transducers have a frequency of 3 to 7 MHz.

Clinical changes in ankylosing spondylitis: 1) Ankylosing inflammation of the spine 2) Association of acute anterior uveitis 3) More frequent association of lymphomas 4) Peripheral enthesitis 5) Sicca syndrome A) the 1st, 2nd and 5th answers are correct B) the 1st, 2nd and 4th answers are correct C) the 1st, 2nd and 3rd answers are correct D) the 1st, 3rd and 4th answers are correct E) the 1st, 4th and 5th answers are correct

B) the 1st, 2nd and 4th answers are correct Ankylosing inflammation of the spine may be associated with peripheral enthesitis and uveitis, but more frequent association with lymphomas and Sicca's syndrome is not typical.

Cause of pain associated with posture or movement can be: 1) rheumatic disease 2) vesicoureteral reflux (VUR) 3) kidney stone stuck in the ureter 4) nephroptosis 5) spinal disc herniation A) the 1st, 3rd and 5th answers are correct B) the 1st, 4th and 5th answers are correct C) the 2nd, 3rd and 4th answers are correct D) the 2nd, 4th and 5th answers are correct

B) the 1st, 4th and 5th answers are correct Body movement related pain is mainly caused by diseases affecting the parts of the musculoskeletal system, for example rheumatoid arthritis, disc herniation. Renal ptosis can cause a pain observed when standing (because of renal occlusion) . Laying down relieves the pain. Kidney stones stuck in the urinary tract can cause continuous colic. Vesicoureteral reflux causes renal pain while urination.

Cause of macroscopic haematuria can be: 1) renal adenocarcinoma 2) acute pyelonephritis 3) vesicoureteral reflux 4) acute cystitis 5) bladder tumor A) the 1st, 3rd and 5th answers are correct B) the 1st, 4th and 5th answers are correct C) the 2nd, 3rd and 4th answers are correct D) the 2nd, 4th and 5th answers are correct

B) the 1st, 4th and 5th answers are correct The most common symptom of bladder tumor is macroscopic, clumsy, painless, so-called "silent" haematuria. Renal tumors may also cause macroscopic bleeding when they break into the cavity. In one form of acute cystitis namely in haemorrhagic cystitis in addition to pyuria, macroscopic haematuria can be observed.

Symptom of bilateral recurrent laryngeal nerve palsy is 1) dysphagia 2) inspiratory stridor 3) changes in voice A) the 1st, 2nd and 3rd answers are correct B) the 2nd and 3rd answers are correct C) none of the above is correct

B) the 2nd and 3rd answers are correct Inspiratory stridor and changes in voice are symptoms indicating bilateral recurrent nerve palsy.

Tonsillar carcinoma 1) more common in females than in males 2) patient's medical history often includes alcohol and/ or nicotine abuse 3) gives lymphogenic metastases in 60% of the cases 4) appears together with clinical metasases in 25% of the cases 5) results in a 5 year postoperative survival in 35% of the patients A) the 1st, 2nd and 3rd answers are correct B) the 2nd, 3rd and 5th answers are correct C) the 1st and 4th answers are correct D) the 1st, 4th and 5th answers are correct E) all of the answers are correct

B) the 2nd, 3rd and 5th answers are correct Tonsillar carcinoma is seen in patients with medical history of HPV infection, alcohol and/or nicotine abuse. It gives lymphogenic metastasis in 60% of the cases. And results in 5 year survival in 35% of the patients.

Praecancerotic lesions of the larynx include 1) Sjögren's syndrome 2) adult form of laryngeal papilloma 3) Quincke oedema 4) leukoplakia 5) pachydermia A) the 1st and 2nd answers are correct B) the 2nd, 4th and 5th answers are correct C) the 3rd and 4th answers are correct D) the 1st and 4th answers are correct E) all of the answers are correct

B) the 2nd, 4th and 5th answers are correct Paracancerotic lesions of the larynx include adult form of laryngeal papilloma, leukoplakia and pachydermia.

Which illness is associated with increased erythrocyte sedimentation rate of the following? 1) Osteoporosis 2) Psoriatic arthritis 3) Osteoarthritis 4) Rheumatoid arthritis 5) Polymyalgia rheumatica A) the 1st, 4th and 5th answers are correct B) the 2nd, 4th and 5th answers are correct C) the 1st, 3rd and 4th answers are correct D) the 2nd, 3rd and 4th answers are correct E) the 1st, 2nd and 5th answers are correct

B) the 2nd, 4th and 5th answers are correct Psoriatic arthritis, rheumatoid arthritis and polymyalgia rheumatica are inflammatory diseases with elevated erythrocyte sedimentation rates, while osteoporosis and osteoarthritis are not.

Which of the followings could cause vitamin B12 deficiency?: 1) a strict vegetarian diet 2) Folic acid deficiency 3) atrophic gastritis affecting the elderly 4) lack of UV exposure A) 1, 2 and 3 is correct B) the correct answers are 1 and 3 C) 2 and 4 is correct D) only 4 correct answer E) 4 and the answer is correct

B) the correct answers are 1 and 3 In the absence of vitamin B12 (cobalamin), megaloblastic anemia and severe neuropathy develop. Vitamin B12 can only be synthesized by microorganisms. In humans, cobalamin is absorbed in the lower part of the ileum, with the help of gastric intrinsic factor secreted by the stomach. Atrophic gastritis is common among the elders, which causes vitamin B12 deficiency due to decreased secretion of gastric juice. All food of animal origin (liver, meat, fish, egg, milk and dairy products) contain vitamin B12, but plant foods do not contain vitamin B12, therefore a vegetarian diet, especially the rather rigorous vegan diet posesa risk of vitamin B12 deficiency. Folic acid deficiency does not cause the symptoms of vitamin B12 deficiency, but in the absence of vitamin B12, no formation of biologically active tetrahydrofolic acid occurs.

The therapeutic effects of the lasers can be mostly used in: A) refractive surgery B) the fundus changes caused by diabetes mellitus C) conjunctival suffusion D) endogenous uveitis E) malposition of the lids

B) the fundus changes caused by diabetes mellitus The therapeutic effects of lasers can be most efficiently used in the treatment of the fundus changes in diabetes mellitus. The timely performed pancoagulation may stop the progressive neovascular stadium, if the glucose level of the patient is also controlled (the last one is the most important).

The following clinical features are characteristic for herpes zoster, except: A) grouped vesicles B) usually mild symptoms C) linear arrangement of vesicle groups D) contagious through direct lesional skin contact E) severe complications are rare but possible

B) usually mild symptoms

Please associate following definitions with the diagnoses below. A) Hereditary bullous disease B) Drug-induced bullous disease C) Bullous metabolic disorder D) Bullous disease related to celiac disease E) Autoimmune bullous disease with typical oral mucous membrane manifestation BOR - 44 - Pemphigus vulgaris BOR - 45 - Porphyria cutanea tarda BOR - 46 - Dermatitis herpetiformis BOR - 47 - Epidermolysis bullosa simplex BOR - 48 - Toxic epidermal necrolysis

BOR - 44 - Pemphigus vulgaris - E) BOR - 45 - Porphyria cutanea tarda - C) BOR - 46 - Dermatitis herpetiformis - D) BOR - 47 - Epidermolysis bullosa simplex - A) BOR - 48 - Toxic epidermal necrolysis - B)

Please associate following statements with the diagnoses below. A) Red plaques in sun-exposed areas B) It can be verified by patch-tests C) It can be triggered by sport and physical activity D) It can be caused by C1 esterase inhibitor deficiency E) White dermographism is a specific sign for it BOR - 49 - Atopic dermatitis BOR - 50 - SLE BOR - 51 - Allergic contact dermatitis BOR - 52 - Hereditary angioedema BOR - 53 - Cholinergic urticaria

BOR - 49 - Atopic dermatitis - E) BOR - 50 - SLE - A) BOR - 51 - Allergic contact dermatitis - B) BOR - 52 - Hereditary angioedema - D) BOR - 53 - Cholinergic urticaria - C)

Please associate following disorders with the corresponding immunologic pathomechanisms below. A) Allergic contact dermatitis B) Allergic rhinitis C) Sarcoidosis D) Schönlein-Henoch purpura E) Autoimmune hemolytic anemia BOR - 54 - IgE-mediated hypersensitivity reaction BOR - 55 - Non-necrotizing granuloma formation BOR - 56 - Hypersensitivity reaction of delayed type BOR - 57 - Immune complex reaction

BOR - 54 - IgE-mediated hypersensitivity reaction - B) BOR - 55 - Non-necrotizing granuloma formation - C) BOR - 56 - Hypersensitivity reaction of delayed type - A) BOR - 57 - Immune complex reaction - D)

Please associate following germs with the corresponding diagnoses or skin signs below. A) Haemophilus ducreyi B) Mycobacterium tuberculosis C) Rochalimaea henselae D) Mycobacterium marinum E) Bacillus anthracis BOR - 59 - Cat scratch disease BOR - 60 - Pustula maligna BOR - 61 - Ulcus molle BOR - 62 - Swimming pool granuloma BOR - 63 - Lupus vulgaris

BOR - 59 - Cat scratch disease - C) BOR - 60 - Pustula maligna - E) BOR - 61 - Ulcus molle - A) BOR - 62 - Swimming pool granuloma - D) BOR - 63 - Lupus vulgaris - B)

Which clinical features are typical for genital herpes infection? 1) Painless ulcers 2) Painful regional lymphadenopathy 3) No prodromal symptoms 4) Recurrent disease A) Answers 1, 2 and 3 are correct. B) Answers 1 and 3 are correct. C) Answers 2 and 4 are correct. D) Only answer 4 is correct. E) All answers are correct.

C) Answers 2 and 4 are correct.

NOT a characteristic hand change of rheumatoid arthritis: A) Erosions of the small joints B) Subluxations of the small joints C) "Pencil in cup" lesion D) Bone ankylosis E) Swan-neck and boutonniere finger deformity on the hands

C) "Pencil in cup" lesion The "pencil in cup" lesion is a feature of psoriatic arthritis, the other X-ray findings are characteristic of RA.

The optimal fluoride intake at the age of one year : A) should not be given B) 0,05 mg C) 0,3 mg D) 0,1 mg E) 1,0 mg

C) 0,3 mg In a non fluoride supplemented environment the optimal dose that has caries protective effect, and does not damage the enamel and has no systemic side effects or poisoning effects is 0,3 mg. the 0,15-0,2x of the adult dose.

Which of the followings are among the main objectives of the European Health Strategy 2008-2013? 1) Harmonization of the health policies of the member states 2) Protecting the health of the aging population 3) Establishment of a Joint European Health Fund to finance the EU action programs 4) Support of dynamic health systems and new technologies for the sustainable development of health systems A) 1, 2 and 3 are correct B) Answer 1 and 3 are correct C) 2 and 4 are correct D) only 4 correct answer E) all 4 answers are correct

C) 2 and 4 are correct The European Union has implemented a number of joint action programs in the field of health in the context of closer integration. The EU published a White Paper in 2007 estabilishing the health strategy for 2008-2013. The three main objectives of this publication were: protecting health in the aging Europe, protecting citizens from health threats, supporting dynamic health care systems and supporting new technologies.

salivary enzymes 1) proteases 2) maltase 3) lipase 4) amylase A) 1., 2. and 3. answers are correct B) 1. and 3. answers are correct C) 2. and 4. answers are correct D) only the 4. answer is correct E) all the answers are correct

C) 2. and 4. answers are correct In the oral cavity the fermentation of the carbohydrates are started by the salivary enzymes. If a carbohydrate containing foodstuff - like bread stays in the oral cavity for a relatively long time due to the fermentation it will be sweet. The metabolism of fat and proteins starts only in the deeper part of the GI system.

The following diseases frequently associate with diabetes mellitus except A) necrobiosis lipoidica B) candida infection C) granuloma anulare D) erythema multiforme E) cutaneous microangiopathy

D) erythema multiforme

What are the characteristics of acute acoustic trauma? 1) The role of air pressure change is significant in its development 2) Developed due sound of high intensity 3) Hearing loss is bilateral 4) Hearing impairment is perceived A) Answers 1, 2 and 3 are correct B) Answers 1 and 3 are correct C) Answers 2 and 4 are right D) only answer 4 is correct E) all 4 answers are correct

C) Answers 2 and 4 are right Acute acoustic trauma develops due to a single high intensity noise which is not accompanied by significant pressure fluctuation. Therefore, only the inner ear is being damaged and the outer and middle ear will show no abnormal changes. The acoustic damage is perceptual, irreversible, effects the high-frequency sound range as one-sided due to the shielding effect of the head. Mostly it is caused by the firing of firearms.

The incidence of new malignant neoplasms is characterized by the following in Hungary 1) The most frequent malignancy in men is prostate cancer 2) The most frequent malignancy in men is bronchial and lung cancer 3) The most frequent malignancy in women is bronchial and lung cancer 4) The most frequent malignancy in women is breast cancer A) Answers 1, 2 and 3 are correct B) Answers 1 and 3 are correct C) Answers 2 and 4 are right D) only answer 4 is correct E) all 4 answers are correct

C) Answers 2 and 4 are right Based on National Registry on Cancer data, the following malignancies were the most frequently diagnosed in Hungary in 2009: for men, bronchial and lung cancer (19.5%), cancer of the the lips, the oral cavity and the pharynx (10.5%) , colorectal and anal cancer (10%). In case of women, breast cancer (21.5%), bronchial and lung cancer (12%), colorectal and anal cancer (8%), and cervical cancer (4%). The percentage distribution indicates the order in both sexes.

Breast screening is recommended in Hungary as follows: 1) for women aged 50-65 2) for women aged 45-65 3) annually 4) biannually A) Answers 1, 2 and 3 are correct B) Answers 1 and 3 are correct C) Answers 2 and 4 are right D) only answer 4 is correct E) all 4 answers are correct

C) Answers 2 and 4 are right Breast cancer screening (mammography) is performed biannually for the age group 45-65 in Hungary

Cancer disease (s) caused by asbestos exposure 1) lymphoma 2) Mesothelioma 3) pancreatic cancer 4) lung carcinoma A) Answers 1, 2 and 3 are correct B) Answers 1 and 3 are correct C) Answers 2 and 4 are right D) only answer 4 is correct E) all 4 answers are correct

C) Answers 2 and 4 are right The relationship between asbestos exposure and mesothelioma is important because mesothelioma is considered as a rare tumor and almost exclusively develops due to exposure to asbestos. Asbestos can also cause lung cancer, but it is not associated with the development of lymphoma and pancreatic cancer.

Which pathogen causes Lyme disease? A) Campylobacter jejuni B) Klebsiella pneumoniae C) Borrelia burgdorferi D) Coxsackie virus E) Epstein-Barr virus

C) Borrelia burgdorferi Only Borrelia burgdorferi causes Lyme disease..

Which country has the best balanced accessibility/equality/quality relation of the health care in the past three decades? A) Hungary B) USA C) Canada D) Poland

C) Canada Formerly socialist countries have emphasized the accessibility and equality but were unable to sustain the proper quality (due to limited resources). The USA healthcare system was characterized by high quality, but this care was provided for narrower groups only. In Canada, the basic medical care is financed from taxes (a strong economic potential is available for this) and the additional treatments are financed mainly through additional insurances, so the Canadian healthcare system is considered as well organized.

What joints are NOT affected in rheumatoid arthritis? A) Metacarpophalangeal (MCP) joints B) Proximal interphalangeal (PIP) joints C) Distal interphalangeal (DIP) joints D) Knee joints E) Radiocarpal joints

C) Distal interphalangeal (DIP) joints The involvement of DIP joints is not typical for RA at all, opposed to the other typical lesions.

It does NOT belong to the diagnostic criteria for systemic lupus erythematosus: A) Anti-DNA positivity B) Nervous system manifestation C) Erosive arthritis D) Renal involvement E) Butterfly (malar) rash

C) Erosive arthritis The disease could be characterized by non-erosive arthritis.

What side effect is NOT characteristic of the antirheumatic biological treatment? A) Bone marrow depression B) Elevated liver enzymes C) Growing joint complaints D) Allergic skin reaction E) Latent tuberculosis flare-up

C) Growing joint complaints Joint complaints tend to decrease together with the level of inflammation and not increase. Other side effects may occur occasionally.

According to the Miller's caries theory: A) Adenylphosfate is provided by the saliva for the fermentation of carbohydrate in the dental plaque B) The initial caries is caused by the bacterial penetration into the enamel C) Lactic acid is produced in the dental plaque D) A caries is mainly caused by anaerobic microorganisms E) The protein content of the tooth is a decisive factor in the development of dental caries

C) Lactic acid is produced in the dental plaque The father of the modern caries theory is Miller, according to this the cause of disease is the low acidic pH. Int he dental plaque the Streptococcus mutans, sanguis, salivarius ferment the sugar and create lactic acid, as a consequence the plaque become acidic and the acids will solve- demonetize the enamel prism.

What is the base (first-line) therapy for rheumatoid arthritis? A) Sulfasalazine B) Cyclophosphamide C) Methotrexate D) Chloroquine E) Azathioprine

C) Methotrexate In RA, methotrexate is the most effective among the listed drugs, and is therefore compulsorily the first to be selected, prior to biological treatments.

Which advanced disease state could be characterized by subcutaneous rheumatoid nodules? A) Polymyalgia rheumatica B) Systemic lupus erythematosus C) Rheumatoid arthritis D) Ankylosing spondylitis E) Systemic sclerosis

C) Rheumatoid arthritis Subcutaneous rheumatoid nodules occur only in RA and not in the other listed diseases.

In case of dry eye syndrome, which examination should be performed first? A) conjunctival biopsy B) tonometry C) Schirmer test D) fluorescein staining of the cornea E) measuring the lizozyme enzyme activity

C) Schirmer test In "dry eye" syndrome the tear production is diminished, which can be shown the most easiest way by the Schirmer test.

The diabetes is a risk factor in periodontal disease BECAUSE it can cause odontoblast resorption A) Both the statement and the reason are true, and the reason verifies the statement. B) Both the statement and the reason are true ,but there is no relation between them C) The statement is true the reason is false D) The statement is false the reason is true E) Both the statement and the reason are false

C) The statement is true the reason is false The diabetic patients are more susceptibly to infections because of a decreased immune resistance. One of the cause of the different oral lesions and periodontitis is the microvasculopathy and the other is the dehydration of the tissues. The resorption of the odontoblastic cells are not causative factors and this does not occur at all.

Which of the following is an interleukin-6 R-inhibitor biological drug? A) Etanercept B) Adalimumab C) Tocilizumab D) Infliximab E) Rituximab

C) Tocilizumab Tocilizumab is an IL-6R inhibitor; etanercept, adalimumab and infliximab are TNF-α inhibitors; and rituximab is a B cell inhibitor.

Which vitamin plays an essential role in iron absorption? A) Vitamin A B) Vitamin B12 C) Vitamin C D) Vitamin D.

C) Vitamin C The hem-iron absorbtion derived from hemoglobin and myoglobin found in meat is hihgly effective. The absorption of inorganic non-hem iron is considerably worse, and is influenced by nutritional factors. The absorption of non-hem iron is inhibited by phytates, polyphenols and oxalates found in plant foods. High calcium intake also inhibits iron absorption. Vitamin C and fermented foods facilitate iron absorption. The role of vitamin C in iron absorption is so important that it can be considered as the physiological role of this vitamin. Each meal should contain at least 25 mg of Vitamin C, if iron-absorbtion inhibitors are present, even more. There are slight differences in the absorption of hem-iron, but the differences in the absorption of non-hem-iron could be as high as 20 times.. In the case of mixed diet, the absorbtion of the total iron content is approx.. 10%.

In acute upper airway infection of an infant, one-sided swelling of the upper and lower eyelid appears. Which condition would you think of? A) acute rhinitis B) acute maxillary sinusitis C) acute ethmoiditis D) allergic reaction

C) acute ethmoiditis In acute upper respiratory infection of an infant, one sided swelling of the upper and lower eyelid is an indication of an acute ethmoiditis. In this age the rest of the sinuses are either not fully developed or cannot cause eyelid swelling. In case of allergy, there is a symmetric swelling.

The most important cause of the visual deterioration in people over 80 years: A) diabetes mellitus B) myopia C) age-related macular degeneration (AMD) D) glaucoma E) cataract

C) age-related macular degeneration (AMD) The age-related macular degeneration affects approximately 30% of the people above the age of 80 yrs.

Necrotic, coated lesion of the tonsils suggests a(n) A) scarlatina-associated tonsillitis B) mononucleosis C) agranulocytic angina D) tonsillitis follicularis E) herpangina

C) agranulocytic angina Necrotic, coated lesion of the tonsils suggest agranulocytic angina. Treatment of the haematological status is recommended.

ETEC group of E. coli is characterized by the followings: 1) causes dysenteria in infants 2) causes cholera-like symptoms in adults 3) causes dysenteria-like symptoms in adults 4) causes asymptomatic excretion/shedding A) answers 1, 2 and 3 are correct B) answers 1 and 3 are correct C) answers 2 and 4 are correct D) only answer 4 is correct E) all of the answers are correct

C) answers 2 and 4 are correct ETEC = Enterotoxic (or enterotoxigenic) E. coli. Its toxins paralyze the Na + -K + pump in the epithelial cells of the intestinal wall, so the infection cause profuse cholera-like diarrhoea. As with most coliform infections, asymptomatic shedding is also possible. Coli infections may also occur in the form of EPEC (enteropathogenic E. coli, intestinal mucosal degeneration) and EIEC (enteroinvasive E. coli, dysenteria-like symptoms, mainly attacking the colon) in Hungary. Beyond the above forms, enteroadhesive, enterohaemorrhagic and enteroaggregative E. coli groups are also known but these forms appear in Hungary never or on very rare occasions.

Which gas(es) can cause pulmonary oedema when inhaled at high concentration? 1) ethylene 2) phosgene 3) hydrogen sulfide 4) fluorine A) answers 1, 2 and 3 are correct B) answers 1 and 3 are correct C) answers 2 and 4 are correct D) only answer 4 is correct E) all 4 answers are correct

C) answers 2 and 4 are correct High concentrations of phosgene and fluorine could cause irritation in the respiratory tract during their acute toxication and eventually could cause lung oedema. Ethylene as asphyxiant gas at high concentration in the air, could cause hypoxia which could result in suffocation due anoxia. Hydrogen sulfide - although it irritates the airway mucosa - could cause hyperpnoe, spasms, loss of consciousness and eventually coma.

If botulism infection is suspected, then the following samples should be sent to the laboratory: 1) feces 2) food leftovers 3) vomit 4) blood A) answers 1, 2 and 3 are correct B) answers 1 and 3 are correct C) answers 2 and 4 are correct D) only answer 4 is correct E) all 4 answers are correct

C) answers 2 and 4 are correct If botulism infection is suspected, samples of food leftovers and blood must be sent to the laboratory. The bacteria and toxins can be detected from food leftovers while the toxin absorbed and entered the circulation can be detected from blood samples. Sending feces samples does not make sense due to the fact that the symptoms are caused by swallowed and absorbed toxins.

Which of the following public health activities is/are primary preventive intervention(s)? 1) cervical cancer screening 2) vaccination 3) breast tumour screening 4) programs related to the prevention of smoking addiction A) answers 1, 2 and 3 are correct B) answers 1 and 3 are correct C) answers 2 and 4 are correct D) only answer 4 is correct E) all of the answers are correct

C) answers 2 and 4 are correct Primary prevention's aims are to maintain general health, to prevent health loss and diseases. Primary prevention aims at eliminating the potential disease sources (etiological and /or risk factors) completely or partially or forming and increasing the resistance against them. Screening programs for an early detection of the disease are considered as secondary preventive interventions.

Complication(s) of silicosis is/are the followings: 1) Raynaud syndrome 2) tuberculosis 3) pleural mesothelioma 4) cor pulmonale A) answers 1, 2 and 3 are correct B) answers 1 and 3 are correct C) answers 2 and 4 are correct D) only answer 4 is correct E) all of the answers are correct

C) answers 2 and 4 are correct Silicosis has been a well-known and proven predisposition to the development of tuberculosis. Due to the increased load on the heart, the formation of cor pulmonale is also a typical complication of advanced or long term silicosis.

How does the treatment threshold depend on the benefit and risk of the treatment? 1) treatment threshold = 1 / [risk / benefit +1] 2) treatment threshold = 1 / [benefit / risk +1] 3) treatment threshold = benefit / (risk + benefit) 4) treatment threshold = risk / (benefit + risk) A) Answers 1, 2 and 3 are correct B) Answers 1 and 3 are correct C) answers 2 and 4 are correct D) only answer 4 is correct E) all 4 answers are correct

C) answers 2 and 4 are correct The relationship between treatment threshold and benefit: treatment threshold = risk /( benefit + risk) = 1 / [benefit / risk +1]

At what age do teenagers start smoking at the highest rate? A) at the age of 10-11 B) at the age of 11-12 C) at the age of 13-15 D) at the age of 15- 18

C) at the age of 13-15 According to the 2009/2010 date of the HBSC study (Health Behaviour of School-age Children), the frequency of smoking in 11-year-olds seems low, but it's increased more than 10 times among 15+ teenagers in European countries.

Which is the most characteristic visual field defect from the followings in a hypophyseal tumor? A) complete homonym hemianopsia B) lower homonym hemianopsia C) bitemporal hemianopsia D) upper homonym hemianopsia E) monocular blindness

C) bitemporal hemianopsia The hypohysis tumor presses the chiasm centrally. The crossing fibers arriving here from the nasal retina, they transmit visual stimuli from the temporal visual field. The damage of these nerve fibers causing bilateral temporal visual field defect.

The most important risk factor of learned helplessness: A) prematurity B) positive family atmosphere C) child abuse D) chronic disease

C) child abuse The learned helplessness develops through a sustained, prolonged loss of control, it includes the feeling of hopelessness, self-abandonment,and has other consequences, too (eg.decreased immune function, learning deficit.

With the pseudoisochromatic charts can be examined the: A) adaptation B) accommodation C) colour perception disturbances D) anomalic quotiens E) fusion frequency

C) colour perception disturbances With the use of the pseudoisochromatic charts, the colour discrimination disorders can be examined.

The meaning of the chemosis: A) conjunctivitis B) bleeding in the anterior chamber C) conjunctival edema D) scleral bleeding E) periorbital edema

C) conjunctival edema

A middle-aged male patient has a painless, freely mobile mass of about 30 mm in one parotid gland for 5 years. Facial nerve funtion is intact. The probable diagnose is A) osteoma in the external acoustic meatus B) glomus caroticum tumor C) pleomorph adenoma D) salivary stone in the parotid gland E) malignant tumor of the parotid gland

C) pleomorph adenoma Pleiomorphic adenoma is the most frequent benign parotid tumor, accessible to remove surgically and does not give metastasis. The diagnosis is made by fine needle aspiration.

Select the normal BMI range. A) under 25 B) from 20 to 24.9 C) from 18.5 to 24.9 D) over 25

C) from 18.5 to 24.9 The body mass index calculation formula is the following: body weight in kilograms has to be divided by the square of height in meters. According to the classification of the World Health Organization, the normal range is between 18.5-24.9. Those with BMI of 20-25 have the lowest risk of premature death. Under 20, the risk is increased in the industrialized countries. It seems that BMI below 20 does not play a causative role in higher mortality, but in this group the proportion of those who lost weight as a result of several diseases is higher. The BMI of Asian populations tends to be lower, consequently here a BMI value between 18.5-20.0 does not indicate an increased risk.

After tonsillectomy, urgent hospitalization is needed if the patient presents with A) oedema of the soft palate and uvula B) tenderness of cervical lymph nodes C) haemoptoe D) intense pain irradiating into the ears E) swallowing difficulties

C) haemoptoe After tonsillectomy, haemoptoe indicates an urgent hospitalization.

The characteristic sign of mandibular fracture 1) diplopia 2) the dental arch continuity is broken 3) xerostomia 4) traumatic occlusion A) if 1., 2. and 3. are correct B) if 1. and 3. are correct C) if 2. and 4. are correct D) only the 4th is correct E) if all the answers are correct

C) if 2. and 4. are correct In case of mandibular fractures the bones due to the muscle pull will be partially or totally dislocated. Characteristic the traumatic occlusion caused by the vertical displacement of the bones and the deformation of the dental arch due to the horizontal movement of the fractured bones. Diplopia and xersotomia does not occur as a side effect of mandibular fractures.

The characteristic sign of acute periodontitis 1) the pain is exacerbated when the patient goes to bed 2) sensitivity to percussion 3) a sudden swelling around the tooth 4) when closing the mouth a sharp pain evoked when occluding the teeth A) if 1., 2. and 3. are correct B) if 1. and 3. are correct C) if 2. and 4. are correct D) only the 4th answer is correct E) if all the answers are correct

C) if 2. and 4. are correct The acute periapical periodontitis is sensitive to percussion and also to occlusal contact. The inflammatory exudates elevates the root from the alveolar socket ant the patients feels the involved tooth erupted. The mechanism of the pain sensation is that if the root is pressed into the inflamed tissue it reacts with severe pain. The pain occurring when the patient is a laid down position is the consequence of the pulpal hyperemia and acute pulpits. It is cause by the elevated blood pressure. The sudden swelling around tooth is a characteristic sing of periodontal inflammation

Which can be a permanent trism ? 1) myogen 2) ankylosis 3) spastic 4) osteogen A) if 1., 2. and 3. are correct B) if 1. and 3. are correct C) if 2. and 4. are correct D) only the 4th answer is correct E) if all the answers are correct

C) if 2. and 4. are correct The myogenic and spastic trismus (lockjaw) might suddenly develops and usually transitory and reversible. The ankylosis and osteogenic trismus slowly develops . The most common causes: The juvenile injuries can cause hematoma in the joint that can later maturate and finally ossifys.. In extreme cases a typical bird like face develops. In adulthood the chronic rheumatoid arthritis or other degenerative diseases like Marie—Stümpfell-spondylitis can lead to irreversible trismus .

How can fetal alcohol syndrome be prevented? A) if the mother ignores alcohol consumption during the first trimester of pregnancy B) if the mother ignores alcohol consumption during the first two trimesters of pregnancy C) if the mother ignores alcohol consumption throughout the pregnancy D) it can not be prevented as is due to genetic predisposition

C) if the mother ignores alcohol consumption throughout the pregnancy The best way to prevent fetal alcohol syndrome is to avoid alcohol consumption throughout the pregnancy, as the quantity of alcohol that would not cause harm to the fetus during pregnancy is not known.

Urodynamic examination is recommended: A) in case of hematuria B) in case of pyuria and fever C) imperative urinary stimuli and involuntary discharge of urine from the bladder when coughing D) in case of abnormal bleeding

C) imperative urinary stimuli and involuntary discharge of urine from the bladder when coughing The combined occurrence of imperative urinary stimuli and physical exertion leads to mixed incontinence, which indicates urodynamic examination. In case of haematuria ultrasound and cystoscopy is the recommended diagnostic method. Gynecological examination is indicated if abnormal bleeding occures. In case pyuria, urine sediment controll, or if necessary bacteriological urine test is indicated.

In case of malfunction of the rectus superior muscle in which direction of the looking is affected? A) inward and downward B) outward and downward C) inward and upward D) upward E) outward and upward

C) inward and upward The examiner should ask the patient to look in- and upward in order to control the function of the superior rectus muscle, because this muscle pulls the eye upward and inward.

Verruca vulgaris occurs commonly in the following conditions, except: A) pes valgus B) immunodeficiency C) psoriasis D) HIV positivity E) hyperhidrosis

C) psoriasis

What is the most important factor of healthcare in the market-oriented countries? A) accessibility B) equality C) quality D) the above factors are not being investigated in market-oriented systems

C) quality Market-oriented systems do not focus on democratic theorems but on the quality of service which comes closest to the mechanisms of the free market. The more restrictive regulation of market mechanisms is characteristic of socially sensitive systems which, however, are unable to provide a high quality level due to their limited resources.

The characteristic sign of pulpal hyperemia : A) cold water will alleviate pain B) the tooth is sensitive to percussion C) it is a reversibly phenomenon D) it is an irreversibly phenomenon E) it is caused by antihypertensive drugs

C) it is a reversibly phenomenon The inflammatory hyperemia leads to the elevation of the blood pressure in the pulp chamber, this irritates the cells. The very sudden sharp pain can spontaneously recede. If it frequently returns it sooner or later will progress to pulpitis. It is the irreversibly damage of the dental pulp. The thermal insults can aggravate the process, sensitivity to percussion is not experienced.

In case of damage of the visual pathway which of the below listed visual filed defect does not correlate with the anatomical localization? A) crossing fibers of the chiasm - bitemporal heteronym hemianopia B) left optical tract - right sided homonym hemianopsia C) left lingular gyrus - right sided lower quadrant anopia D) left temporal lobe - right sided upper quadrant anopia

C) left lingular gyrus - right sided lower quadrant anopia The damage of the left lingular gyrus causing right sided temporal peripheral defect in the visual field.

In adult patients with unilateral, recurring catarrh of the Eustachian tube you should exclude... A) peritonsillar abscess. B) adenoid hypertrophy. C) malignant tumor in the epipharynx. D) deviated septum.

C) malignant tumor in the epipharynx. Adult patients with unilateral, recurring catarrhal inflammation of the Eustachian tube should be examined for epipharyngeal tumors that obstruct the opening of the tube.

The anatomic structure(s) opening into the middle nose cavity is/are: A) nasofrontal and nasolacrimal ducts B) sphenoidal sinus and Eustachian tube C) maxillary sinus, frontal sinus and anterior and middle ethmoid cells D) posterior ethmoid cells E) only the maxillary sinus

C) maxillary sinus, frontal sinus and anterior and middle ethmoid cells The middle nasal cavity includes the openings for the maxillary sinuses, frontal sinus and anterior and middle ethmoid cells.

The diagnosis of proliferative diabetic retinopathy can be established by the following signs: A) cotton-wool type degenerative spots B) microaneurysms C) neovascularization D) point-like bleedings E) retinal edema

C) neovascularization In the proliferative stadium of diabetic retinopathy neovascularization is the most characteristic sign, other signs may occur in the non-proliferative stadium as well.

Which of the below listed nerves is damaged least of all by a tumour localized in the posterior part of the skull (posterior scala)? A) hypoglossal nerve B) facial nerve C) optic nerve D) vagal nerve

C) optic nerve The tumor affecting the posterior scala affects least (if at all) the optic nerve.

About the function of the outer hair cells we can get objective information on A) tympanometry B) BERA C) otoacoustic emission D) investigation of stapedial reflex

C) otoacoustic emission The otoacoustic emission objectively registers the low intensity sounds at the external ear canal,that have been produced by the outer hair cells.

Characteristics of senile macular degeneration, except: A) slow progression B) does not cause full blindness C) peripheral vision is affected D) it can be confused with systemic disease

C) peripheral vision is affected Senile macular degeneration never affects the retinal periphery, the changes are in the macula.

A patient living with tracheostomy cannula suddenly begins to suffocate. The first thing you do is to A) suction the trachea through the cannula B) refer the patient to a specialist C) remove the cannula immediately and dilate the stoma with a nasal speculum D) force the patient to cough

C) remove the cannula immediately and dilate the stoma with a nasal speculum When a patient with tracheostomy cannula starts to suffocate, immediate removal of the cannula is recommended and dilation of the stoma with a nasal speculum

Which plays the main role in the development of testicular tumor? A) injury B) testicular torsion C) retentio testis (cryptorchidism) D) genetic predisposition

C) retentio testis (cryptorchidism) The only proven risk factor in the development of testicular cancer is cryptorchism. (In this case, the testicular cancer is 10 to 20 times more common). The role of injury, testicular torsion, genetic predisposition has not been proven.

It is not characteristic of ostitis alveolaris (dry socket) : A) oral malodor B) inflammation around the alveolar socket C) severe pain at the day of extraction D) general malaise E) fever

C) severe pain at the day of extraction The dry socket (alveolar ostitis) develops a couple of days after the extraction. Its main cause is the early degradation of the blood clot or the insufficient blood clot formation in the socket.

Genetic subtypes predominant to rheumatoid arthritis: 1) HLA DRB1*01 2) HLA DRB1*04 3) HLA DRB1*10 4) HLA B27 5) HLA B35 A) the 1st, 2nd and 4th answers are correct B) the 1st, 3rd and 5th answers are correct C) the 1st, 2nd and 3rd answers are correct D) the 3rd, 4th and 5th answers are correct E) the 2nd, 3rd and 5th answers are correct

C) the 1st, 2nd and 3rd answers are correct The mentioned subtypes of HLA DRB1are predisposing factors for RA while HLA B27 and B35 are associated with seronegative spondyloarthritis.

Underlying cause of epistaxis may be 1) trombocytopenia 2) morbus Osler 3) vasomotor rhinitis 4) lacrimal gland obstruction 5) hypertension A) the 1st, 2nd and 3rd answers are correct B) the 2nd, 3rd and 4th answers are correct C) the 1st, 2nd and 5th answers are correct D) the 3rd, 4th and 5th answers are correct E) all of the answers are correct

C) the 1st, 2nd and 5th answers are correct Underlying cause of epistaxis are thrombocytopenia, morbus Osler, and hypertension.

Conservative therapy of paranasal sinusitis involves 1) decongestion nosedropp usement 2) tamponade of the middle meatus 3) punction and rinsing of the paranasal sinus 4) X-ray radiation therapy 5) adminstration of antibiotics A) the 1st, 2nd and 3rd answers are correct B) the 2nd, 4th and 5th answers are correct C) the 1st, 3rd and 5th answers are correct D) the 3rd, 4th and 5th answers are correct E) all of the answers are correct

C) the 1st, 3rd and 5th answers are correct Conservative therapy of paranasal sinusitis includes decongestion nosedropp usement, punction and rinsing of the sinus, administration of antibiotics.

Sensorineural hearing loss may develop after the following viral infections 1) mumps 2) chickenpox 3) rubeola 4) measles 5) serous meningitis A) the 1st, 2nd and 3rd answers are correct B) the 1st and 3rd answers are correct C) the 1st, 4th and 5th answers are correct D) the 2nd, 3rd and 4th answers are correct E) the 4th and 5th answers are correct

C) the 1st, 4th and 5th answers are correct Mumps, measles and serous meningitis can cause sensorineural hearing loss.

Which of the following drugs are used to improve incontinence in females? 1) Doxycycline 2) Ditropan (oxybutynin) 3) Minipress (prazosin) 4) Ephedrine A) the 1st, 2nd and 3rd answers are correct B) the 1st and 3rd answers are correct C) the 2nd and 4th answers are correct D) only the 4th answer is correct E) all of the answers are correct

C) the 2nd and 4th answers are correct Ditropan is an anticholinergic drug, which causes smooth muscle relaxation. It is used in "urge" type incontinence. Ephedrine is an α- and β-adrenoreceptor-excitatory, enhances the tone of the internal sphincter of the bladder. The drug that can be given in stress incontinence. Doxycyclin is an antibiotic. The Minipress α1 adrenoreceptor blocker, which leads to relaxation of smooth muscle elements on the vesicles, and thus increases urine flow.

Continent urinary diversions are: 1) ureterocutaneostomy 2) orthotopic ileal bladder 3) cutaneous uretero-ileostomy (Bricker bladder) 4) Sigma rectum pouch (Mainz pouch II) A) the 1st, 2nd and 3rd answers are correct B) the 1st and 3rd answers are correct C) the 2nd and 4th answers are correct D) only the 4th answer is correct E) all of the answers are correct

C) the 2nd and 4th answers are correct In an ureterocutaneostomy the urethra is terminating on the abdominal wall. In the Bricker bladder, ureter is implanted in a short ileum segment, the other end of the intestinal tract is determined to the abdominal wall. These two methods are not continental urinary tract creating methods, because the urine is constantly flowing away, patients should wear urostomy bags. Orthotopic bladder formation is a bladder replacement from an isolated intestinal tract, in which case the bladder is sewn with the original urethra. The Mainz II. pouch is actually a low pressure, continental urine reservoir, in which case the ureter is implanted into the detubularized intestinal segment of the sigma. *Continent urinary diversions - ways to pass urine after having a surgery to remove the bladder (cystectomy).

The testicular ultrasound examination is good to detect: 1) infertility 2) testicular tumor 3) azoospermia 4) orchidoepididymitis A) the 1st, 2nd and 3rd answers are correct B) the 1st and 3rd answers are correct C) the 2nd and 4th answers are correct D) only the 4th answer is correct E) all of the answers are correct

C) the 2nd and 4th answers are correct Infertility and azoospermia is a complex andrological diagnosis, which is based on physical examination; US alone is not enough. It is absolutely necessary to perform a microscopic examination of the ejaculate. Other procedures like microbiological, diagnostic, surgical, radiological, biochemical, genetic, endocrinological, examinations are necessary in order to find the cause. In case of orchiepididymitis, an inhomogeneous echotexture can be found in the testicles. In a testicular tumor an inhomogenous echotexture can be observed in part or in the whole testicle.

Not a transurethral procedure: 1) internal urethrotomy 2) ESWL (Extracorporeal Shock Wave Lithotripsy) 3) inserting urethral catheter with cystoscopy 4) epicystostomy A) the 1st, 2nd and 3rd answers are correct B) the 1st and 3rd answers are correct C) the 2nd and 4th answers are correct D) only the 4th answer is correct E) all of the answers are correct

C) the 2nd and 4th answers are correct Internal uretrotomy is the incision of the urethral stenoses with the optic control. An ureteral catheter inserted using a special ureterocystoscope, also through the urethra. ESWL: see question n.o.: URO-61. Epicystostomy means percutaneous bladder punction.

Invasive bladder tumor treatment options: 1) Mitomycin, Epirubicin 2) radical cystectomy 3) local chemotherapy 4) resection of bladder wall A) the 1st, 2nd and 3rd answers are correct B) the 1st and 3rd answers are correct C) the 2nd and 4th answers are correct D) only the 4th answer is correct E) all of the answers are correct

C) the 2nd and 4th answers are correct Local chemotherapy is only suitable for the prevention of tumor recurrences after surgical treatment of superficial bladder tumors. The treatment of invasive bladder tumors involves radical cystectomy, bladder wall resection and TUR, possibly with radiotherapy or chemotherapy (adjuvant, neoadjuvant therapy). For palliative use, TUR, radiation or chemotherapy is most commonly used, although in some cases cystectomy can be performed.

Which are the testicular tumor markers? 1) PSA (Prostate-Specific Antigen) 2) β-hCG 3) testosterone 4) AFP (Alpha-fetoprotein) A) the 1st, 2nd and 3rd answers are correct B) the 1st and 3rd answers are correct C) the 2nd and 4th answers are correct D) only the 4th answer is correct E) all of the answers are correct

C) the 2nd and 4th answers are correct Serum level of β-hCG increase mainly in choriocarcinoma and AFP is higher in nonseminoma type testicular tumors. PSA is a tumor marker used in prostate cancer. Testosterone levels are determined for the differential diagnosis of different types of hypogonadism.

The chloride level can be high : 1) in kidney failure 2) after ureterosigmoideostomy 3) because of increased intake of sodium hydrogencarbonate from soda pops 4) in hyperchloremic metabolic acidosis A) the 1st, 2nd and 3rd answers are correct B) the 1st and 3rd answers are correct C) the 2nd and 4th answers are correct D) only the 4th answer is correct E) all of the answers are correct

C) the 2nd and 4th answers are correct Ureterosigmoideostomy is followed by hyperchloraemic acidosis. Laboratory abnormalities in renal failure: hyponatremia, hypochloremia, hypocalcemia, hyperkalemia, hypermagnesemia, acidosis.

Cause of pollakisuria: 1) Ormond's disease 2) urine retention in the bladder 3) stress incontinence 4) urethritis A) the 1st, 2nd and 3rd answers are correct B) the 1st and 3rd answers are correct C) the 2nd and 4th answers are correct D) only the 4th answer is correct E) all of the answers are correct

C) the 2nd and 4th answers are correct Urine remaining after urination forms a constant urge to urinate. In case of urethritis infected urine causes irritative symptoms such as pollakisuria.. *Pollakisuria - abnormally frequent urination.

Symptoms of acute pyelonephritis: 1) Both side kidney pain 2) kidney sensitivity to touch 3) fever 4) pyuria 5) macroscopic haematuria A) the 1st, 3rd and 5th answers are correct B) the 1st, 4th and 5th answers are correct C) the 2nd, 3rd and 4th answers are correct D) the 2nd, 4th and 5th answers are correct

C) the 2nd, 3rd and 4th answers are correct Acute pyelonephritis usually develops due to the the onset infections targeting the lower section of the urinary tract. Typical symptomes are high fever with chills, the enlarged kidneys stretch the kidney capsules and this causes pain. Pyuria (excluding the blocked kidney) refers to urinary infection. Macroscopic haematuria is not characteristic. Bilateral kidney pain refers to a locomotor disorder or an internal medicine related kidney disease.

What are the contraindications of antirheumatic biological therapy? 1) Strong joint arthritis 2) Active tuberculosis 3) NYHA class 3-4 heart failure 4) The existence of malignant disease 5) Rheumatoid factor positivity A) the 1st, 3rd and 4th answers are correct B) the 2nd, 3rd and 5th answers are correct C) the 2nd, 3rd and 4th answers are correct D) the 3rd, 4th and 5th answers are correct E) the 1st, 2nd and 3rd answers are correct

C) the 2nd, 3rd and 4th answers are correct Strong rheumatoid factor positive inflammatory arthritis is rather an indication than a contraindication opposite to the other three contraindications.

The patient reports a foreign body in the esophagus. In addition to laryngoscopy we 1) make him/her eat bread immediately 2) give him/her antibiotics and spasmolytics 3) perform radiography of the esophagus with absorbable contrast nedia 4) oesophagoscopy 5) scintigraphy A) the 1st, 4th and 5th answers are correct B) the 2nd and 5th answers are correct C) the 3rd and 4th answers are correct D) the 2nd, 4th and 5th answers are correct E) all of the answers are correct

C) the 3rd and 4th answers are correct In the case of foreign body in the esophagus, we must perform radiography of the esophagus with an absorbable contrast media, and an oesophagoscopy along with laryngoscopy.

From the below listed questions, choose the incorrect sentence A) In the cornea and sclera collagen fibres can be founded B) the physiologic function of the Schlemm's channels to drain the aqueous from the anterior chamber C) the anterior and posterior surface of the iris is covered by multilayer squamous epithelium D) parts of the uvea: iris, ciliary body, chorioid

C) the anterior and posterior surface of the iris is covered by multilayer squamous epithelium The statement about to cover the anterior and posterior surface of the iris by multilayer squamous epithelium is incorrect. Anterior surface is covered by endothelial cells, the posterior surface is covered by pigment epithelium.

What phenomenon urged the /rethinking of the healthcare system of the developed countries in the second half of the 20th century? A) The rapid growth of death rates related to chronic non-communicable diseases B) Reduction in the payments related to health insurance due to increasing unemployment rate C) the gap between GDP's slow and the health care costs rapid growth D) Significant development in the field of health informatics

C) the gap between GDP's slow and the health care costs rapid growth Mortality of chronic noncommunicable diseases did not grow intensively in the most industrialized countries in the second half of the 20th century, even a declining trend was observed related to cardiovascular mortality. The increasing trend in unemployment rate was not so intensive that it could affect the health care system itself. Informatics is not a crucial factor in the healthcare systems, on the contrary, it must adapt to the healthcare system as well as must serve its needs. The true reason was the very rapid growth of health care costs, which the GDP increase could not keep pace with. It was necessary to restructure the healthcare system, besides organizational changes rethinking its content was also necessary.

The juvenile periodontal disease can be managed BECAUSE it is always caused only by local etiologic factors . A) Both the statement and the reason are true, and the reason verifies the statement. B) both the statement and the reason are true ,but there is no relation between them C) the statement is true the reason is false D) the statement is false the reason is true E) both the statement and the reason are false

C) the statement is true the reason is false In childhood the juvenile periodontitis is mostly associated with severe systemic diseases but the local causative factors (dental plaque bacteria) are also important factors in the aetiology. Depending of the systemic background disease its management and therapy can be successful

After the TUR surgery of superficial bladder tumors local BCG bladder installation should be performed, because this is the only drug for this procedure. A) both the statement and the explanation are true and a causal relationship exists between them; B) both the statement and the explanation are true but there is no causal relationship between them; C) the statement is true, but the explanation is false; D) the statement is false, but the explanation itself is true E) both the statement and the explanation are false

C) the statement is true, but the explanation is false; In the case of superficial bladder tumors, the recurrences are very common, as prevention immunomodulatory agents can be locally used such as BCG, Interferon or Interleukin instillation.

Usually the nearest tooth to the basis of the maxillary sinus is: A) the upper wisdom tooth B) the palatal root of the upper first premolar C) the upper second molar D) the distal root of the upper second premolar E) the upper first molar

C) the upper second molar Usually in 45% of the cases the upper second molars' roots are in the nearest position to the alveolar recess of the maxillary sinus. Sometimes the sinus floor protrudes into the interradicular area between the mesial and palatal roots. The next is the first molar (30%) and followed by the wisdom tooth (27%) according to the approximation. It is very rare that the sinus protrudes until the apex of the first premolar

The cause of bilateral exophthalmos: A) orbital pseudotumor B) lymphoma C) thyroid induced exophthalmos D) dermoid E) tumor of the cavernous sinus

C) thyroid induced exophthalmos Bilateral exophthalmos is caused most frequently by thyroid dysfunction and consequently the retrobulbar tissue and the extraocular muscles at the beginning are thickened, edematous, later starts to fibrose.

A pulsating tumor palpable on one side of the neck probably indicates a A) thyreioditis B) mediastinitis C) tumor adjoining to the carotid artery D) foreign body in the hypopharynx

C) tumor adjoining to the carotid artery A pulsating tumor palpable on one side of the neck probably indicates a tumor adjoining to the carotid artery.

Inspiratory dyspnea is a typical symptom of A) cardiac insufficiency B) bronchial asthma C) upper airway dyspnea D) psychogenic dyspnea

C) upper airway dyspnea Inspiratory dyspnea is a typical symptom of upper airway system. In this case intubation or tracheotomy must be considered.

The following diagnoses are synonyms. The only exception is: A) hypernephroma B) Grawitz- tumor C) renal cell carcinoma D) renal adenoma

D) renal adenoma The renal adenoma is a benign, epithelial tumor, while hypernephroma, Grawitz tumor and renal cell carcinoma are synonyms for malignant kidney cancer.

The lidocain cartridge/ampoule commercially delivered for dental anesthesia as A) Inj. Lidocain 1% - Adrenalin 0,01% B) Inj. Lidocain 2% - Adrenalin 0,1% C) Inj. Lidocain 2% - Adrenalin 0,01% D) Inj. Lidocain 2% - Adrenalin 0,001% E) Inj. Lidocain 1% - Adrenalin 0,02%

D) Inj. Lidocain 2% - Adrenalin 0,001% For dental infiltration and block anesthesia -unless the administration of adrenalin (epinephrine) contraindicated the proper anesthesia can be achieved with injections containing 2% lidocain (xilocain) +0,001% adrenalin. The commercial unit dose is 2 ml in brown ampoule. In general surgery 1 and 2% lidocain is used without epinephrine., but this has no adequate analgesic effect is dentistry . It can only be used for superficial soft tissue anesthesia for a short period of time. The reason is the special vascularisation of the oral tissues and the compact bony structure .

The patient suffers from vertigo and unilateral hearing loss and tinnitus for 1 hour. The probable diagnosis is: A) acoustic neurinoma B) neuronitis vestibularis C) vertebrobasilar insufficiency D) Ménière's disease

D) Ménière's disease The Ménière disease is characterized by a continues spinning vertigo, lasting for at least 1-2 hours, accompanied by tinnitus and unilateral hearing loss. In later phase the tinnitus and hearing loss can be permanent.

In which disease(s) may nail clubbing occur? 1) Psoriasis 2) SLE 3) Onychomycosis 4) Pulmonary fibrosis A) Answers 1, 2 and 3 are correct. B) Answers 1 and 3 are correct. C) Answers 2 and 4 are correct. D) Only answer 4 is correct. E) All answers are correct.

D) Only answer 4 is correct.

Which symptom is NOT characteristic of Felty syndrome? A) Inflammatory arthritis B) High titer rheumatoid factor positivity C) Splenomegaly D) Pneumoconiosis E) Granulocytopenia

D) Pneumoconiosis Pneumoconiosis is part of Caplan's syndrome, the other four symptoms are characteristic of Felty's syndrome.

It does NOT belong to seronegative spondyloarthritis: A) Ankylosing spondylitis B) Reactive arthritis C) Enteropathic arthritis D) Polymyalgia rheumatica E) Psoriatic arthritis

D) Polymyalgia rheumatica Polymyalgia rheumatica does not belong to this group due to having different characteristics.

Hospitalization of the patient is essential if the cause of food poisoning is: A) Staphylococcus B) Salmonella C) Campylobacter D) Toxic mushroom

D) Toxic mushroom Hospital admission is required if there is a suspicion of mushroom poisoning. In the other cases hospitalization is required only if the symptoms indicate it.

Which vitamins are extremely important for the elderly? A) Vitamin B12 B) Vitamin B12, folic acid C) Vitamin B12, Vitamin D D) Vitamin B12, vitamin D, folic acid

D) Vitamin B12, vitamin D, folic acid Sufficient intake of calcium and vitamin D in combination with exercise is a key factor in the prevention of osteoporosis and resulting bone fractures. Intake of adequate amounts of vitamin B12 and folic acid is necessary not only to prevent anorexia, but also to prevent the symptoms of neuropathy. The higher concentration of folic acid in plasma correlates with the improved cognitive performance observed among older people.

The term "acoustic impedance" refers to the A) pressure in the middle ear. B) resonancy of the external ear. C) hearing loss of inner ear origin. D) acoustic resistance. E) spatial hearing.

D) acoustic resistance. The part of the middle ear (tympanic membrane and ossicular chain) create an acoustic resistance against the sound wave pressure.

Persistance of a broncial foreign body may lead to A) pneumonia B) bronchiectasia C) pulmonary abscess D) all the above conditions

D) all the above conditions Krónikus hörgőidegentest pneumoniát, bronchiectasiát, tüdőtályogot okozhat, ezért eltávolítása indokolt. (A chronic bronchial foreign body can cause pneumonia, bronchiectasis, and lung abscess, so its removal is justified)

Most relevant therapy of allergic rhinitis consists of: A) vasoconstrictor agents B) antibiotics C) NSAIDs D) elimination of the allergen

D) elimination of the allergen Most relevant therapy of allergic rhinitis is the elimination of the allergen.

Food allergy: A) symptoms are localized to the gastrointestinal tract B) every fourth person is allergic to a food ingredient C) usually means lactose sensitivity D) an immune system-mediated pathological response to one (or sometimes multiple) component of food

D) an immune system-mediated pathological response to one (or sometimes multiple) component of food The food allergy is an abnormal immune response to the food which is often IgE-mediated (Type I hypersensitivity) or delayed cell-mediated reaction (IV type .). Mostly, specific antibodies can be detected in the serum of patients. Food allergy symptoms can not only affect the gastrointestinal tract but could cause skin symptoms, asthma or anaphylactic shock. The prevalence among the adult population is about 2%, while in children its more frequent, approx.. 8%. Lactose sensitivity or intolerance is a non-allergic pathological state, caused by the decreased functioning of the lactase enzyme, due to which the main carbohydrate component of the cow's milk, lactose can not be digested by the patients and this causes the gastrointestinal symtpoms.

Most common location of nasal bleeding: A) nasopharynx B) middle turbinate C) superior meatus D) anterior part of nasal septum E) posterior part of the nasal septum

D) anterior part of nasal septum Most common location of nasal bleeding is the anterior part of the nasal cavity, most specifically at the Kisselbach area, whereas the mucosa is thinner and has a rich vascularity.

The most frequent complication of severe myopia A) posterior uveitis B) cataract C) angioid streaks D) atrophic chorioideal area in the posterior pole E) iris rubeosis

D) atrophic chorioideal area in the posterior pole In severe myopia in the posterior pole of the eye, staphylomata may occur, which causing a circulatory deficiency and leading to atrophic area in the choroid. The stretching of the sclera is not followed by the retina and choroid. With the rupture of the blood vessels bleeding might occur in the atrophic areas (Fuchs spots).

What is the definition of preterm birth? A) birth before the termination of the 40th week during pregnancy B) birth weight below 2500 g C) birth weight below 3000 g D) birth before the termination of the 37th week during pregnancy

D) birth before the termination of the 37th week during pregnancy According to an earlier definition of the World Health Organization (WHO), premature birth was defined as the birth before the termination of the 37th pregnancy week or when the birth weight is below 2.500 g. However, the current WHO definition refers to premature birth when the birth take place before the end of the 37th week.

Aflatoxins produced by the Aspergillus mold genotypes that contaminate food: A) have estrogenic effects B) have an antimicrobial effect C) cause vomiting and diarrhea D) can cause liver cancer

D) can cause liver cancer The oldest known mycotoxins are aflatoxins produced by Aspergillus flavus and Aspergillus parasiticus fungi. Aflatoxins are teratogenic, mutagenic and carcinogenic compounds and represent one of the major etiological factors of liver cancer in Africa. Cereals, oilseeds, beans and dried fruits can be infected with aflatoxin-producing molds, most commonly the contamination is affecting peanut, therefore peanuts can only be marketed after aflatoxin control.

Surgical treatment of T1a vocal chord cancer is A) laryngectomy B) horizontal resection of the larynx C) vertical resection of the larynx D) chordectomy E) arytenoidectomy

D) chordectomy Surgical treatment of T1a vocal chord cancer is chordectomy.

The consequence of the use of pacifier soaked in candy : A) caries at the upper and lower central incisors B) gingivitis C) periodontitis D) circular caries on the four upper deciduous incisors E) all of them

D) circular caries on the four upper deciduous incisors The pacifier soak in candy or the very sweet tea given from a bottle thru a nipple can cause very characteristic lesion- the so called circular caries. It is located on the upper deciduous incisors. A dark brown lesion develops around the cervical area of the crown, later a real decay is formed and can lead to crown fracture.

From the below mentioned symptoms which might be found both in keratitis, iritis, and in acute glaucomatous attack? A) corneal precipitates B) corneal blur C) dilated pupil D) circumcorneal or ciliary injection E) subconjunctival bleeding

D) circumcorneal or ciliary injection Circumcorneal or ciliary injection. This sign labels inflammations, congestion within the iris, ciliary body and this can occur in glaucoma as well.

Which is not a cause of the glaucoma from the below listed entities? A) closed chamber angle B) pupillary block C) obstruction of Schlemm channels D) damage of the secretory epithelium of the ciliary body E) obstruction of the trabecular meshwork around the Fontana spaces

D) damage of the secretory epithelium of the ciliary body The damage of the epithelium of the ciliary body does not cause glaucoma.

The most likely diagnose in case of multiple, small, light diffuse retinal changes, if no other retinal pathology can be observed: A) inveterated chorioretinitis B) hypertensive retinopathy C) diabetic retinopathy D) drusen E) sclerosis of the retinal arterioles

D) drusen It is drusen, in any other cases other changes can also be observed in the fundus. *Drusen - tiny yellow or white accumulations of extracellular material that build up between Bruch's membrane and the retinal pigment epithelium of the eye. The presence of a few small ("hard") drusen is normal with advancing age, and most people over 40 have some hard drusen.[1] However, the presence of larger and more numerous drusen in the macula is a common early sign of age-related macular degeneration (AMD).

Ectropium often presents together with: A) pterygium B) entropium C) pinguecula D) epiphora E) ptosis

D) epiphora If the lower eyelid is ectropic, usually the lower punctual point is everted outward also. Therefore tearing (epiphora) is the consequence, because the drainage of the tear is impaired, the lower punctual point is not reaching the tear level.

Basic pediatric care is accomplished by A) home paediatricians, school doctors B) home paediatricians, school doctors, school dentists C) home pediatricians, health visitors, school doctors, school dentists, D) family doctors, home pediatricians, health visitors, school doctors, school dentists

D) family doctors, home pediatricians, health visitors, school doctors, school dentists Basic medical care for children between the ages of 0 and 18 includes school health care in Hungary; the control of growth and the screening tests must be performed by family doctors, home pediatricians, school doctors, school dentists, and health visitors.

The risk of stomach cancer is increased by: A) excessive alcohol consumption B) insufficient intake of dietary fibers C) consumption of very hot drinks D) frequent consumption of salt-preserved foods

D) frequent consumption of salt-preserved foods Frequent consumption of salted foods is associated with an increased risk of developing gastric cancer. The significant decrease in the incidence of gastric cancer in recent decades is due to changes seen in the preservation methods (traditional preservation methods was replaced by refrigeration) in the industrialized countries. Frequent consumption of very hot drinks causes an increased risk of developing esophageal cancer. Inadequate intake of dietary fibers and excessive alcohol consumption correlates with the development of colorectal cancer, not with gastric cancer.

The condition in which the cause of dysphagia cannot be detected on swallow radiography is A) esophageal diverticulum B) cervical spondylarthrosis C) esophageal carcinoma D) globus hystericus E) esophageal spasm

D) globus hystericus If dysphagia is not detected by swallow radiography. Patients most of the times suffers out of globus hystericus.

Esophageal varicosity is a concomitant disease of A) cardiac decompensation B) stricture of the cardia C) esophageal tumor D) hepatic cirrhosis E) malignant struma

D) hepatic cirrhosis Esophageal varicosity is a concomitant disease of hepatic cirrhosis. The varicose veins are firstly found in this territory.

In which disease can occur the so called "band keratopathy"? A) syphilis B) hyperlipidemia C) older age D) hypercalcemia E) Wilson disease

D) hypercalcemia Calcium is deposited in the cornea due to the hypercalcemia.

In a teenager boy due to eye trauma the vision decreases suddenly. The physical examination reveals blood in the anterior chamber. This is called: A) hypopyon B) conjunctival hemorrhage C) corneal abrasion D) hyphema E) vitreous hemorrhage

D) hyphema If blood appears in the anterior chamber, it is called hyphema.

The management of pregnancy gingivitis : A) gingivoplasty B) topical metronidazole application C) broad spectrum antibiotics D) improved oral hygiene E) neither of them

D) improved oral hygiene The gingivitis was considered as one of the physiological symptoms of the pregnancy hormonal changes. After delivery and the finish of lactation it well recede but only the prefect oral hygiene can cure the disease. Medication or surgery is contraindicated. The best remedy is the perfect oral hygiene even during pregnancy.

The location of the oral orifice of the Stenon's duct (parotid gland's duct): A) in the vestible at the upper premolars' region B) in the cavum oris proprium( oral cavity) at the second upper molars C) in the vestible at the upper canines' region D) in the vestible at the upper second molars E) in the vestible at the upper first molars' region

D) in the vestible at the upper second molars The Stenon's duct is the duct of the parotid gland it enters into the vestibule of the oral cavity at the upper second molars, creating a small nodule named as Caruncula

The main symptom of stress incontinence is: A) urination urgency B) incontinence during nights C) low abdominal pain D) incontinence after increased abdominal pressure E) burning in the vagina

D) incontinence after increased abdominal pressure In stress incontinence the patietnt complaints of involuntary urine leakage when the intravesical pressure associated with the elevated abdominal pressure exceeds the maximal urethral pressure without the contraction of the detrusor muscle.

Which illegal drug is consumed the most among adolescents in Hungary? A) heroin B) cocaine C) extasy D) marijuana

D) marijuana According to data obtained from the National Drug Collection and Contact Center, the most commonly used illegal drug among adolescents is the marijuana. According to the European Monitoring Center for Drugs and Drug Addiction in Europe and according to the United Nations Office on Drugs and Crime (UNODC) worldwide marijuana is the most commonly used drug among teenagers.

Which health monitoring method (s) can be used when the information required to operate the monitor goes beyond the routine information requirements and which is not required by law and requires additional work and commitment of the data providers? 1) Polls 2) Linking custom data from other databases 3) Disease (group)-specific registries 4) Voluntary data providers A) Answers 1, 2 and 3 are correct B) Answers 1 and 3 are correct C) Answers 2 and 4 are correct D) only 4 is right E) all 4 answers are correct

D) only 4 is right Based on the information needs and data sources the following types of surveillance can be defined: 1. Mandatory reporting system (E.g. infectious diseases, occupational diseases).They are needed when the report requires some sort of intervention. Consequently, the content of the report is defined in detail according to the conditions of the intervention. 2. Monitoring based on laboratory results. (E.g.: Monitoring of antibiotic resistance) By regularly evaluating the results of microbiological laboratories, changes can be monitored relatively simply and effectively even at institutional level.3. Disease (group) specific registry (E.g .: tumor register, register of developmental malformations) Tools that register every diesease of a certain population, which use exact definition for the cases, collects detailed informations and often enable follow up of patienst. These tools require relatively large amounts of resources and rarely contain information regarding determinants of health. 4. Voluntary reporting (E.g.: Family doctor monitoring systems in regard to certain illnesses ,prevalency and quality of care; the General Practitioners Sentinal Station Programme operated by the Faculty of Public Health of the University of Debrecen). Ocassionally for operating the monitoring system such information is needed that goes beyond the routine information requirements and which is not required by law and requires additional work and commitment of the data providers.. In these cases it is advisable to generate a longer list from voluntary data providers, from whom a representative survey group can be set up. 5. Surveys (eg: National Health Interview Survey in 2000 and 2003, the European Health Interview Survey, 2009) These can be interview based, where the knowledge related to health, attitude and actions towards health as well as and experiences with the healthcare system and the health status based on self-assessment can be evaluated. It may require more resources, but the final data will be more reliable if this is combined with objective measurements as well. 6, Utilising databases of other information systems. (E.g.: Database of the National Health Insurance Fund, IT databases of healthcare providers.) These systems are primarily not for monitoring morbidity; the data content is generally insufficient for the monitoring, their data quality is often insufficient, its reliability varies from one provider to another, as institutional diagnostic and coding methods vary. We can get misleading results if we use these databases without taking into account these limiting factors. 7. Monitoring Warning Events (eg diagnosis of Mesothelioma) This method can be used in the case of rare diseases that are connected to a risk factor in a very specific way and are automatically linked to a certain intervention..8. Combining custom data from databases (individual IDs can be used to connect different databases. In a pooled database of linked exposure and health data, little effort is required to detect specific connections. It is a very potent method in regard to data utilization, however this possibility is very limited in certain countries due to legisislation. It is also useful to determine the reporting discipline of some registries.) 9. Combined Monitor (Eg .: Northeastern Hungarian Health Observatory) We can create a more complete picture of a disease by evaluating the results of multiple monitors together, for example if combined evaluation of mortality, morbidity and the quality of care data is possible.

The cause of retinal cotton-wool spots: 1) lipid deposition 2) drusen 3) chorioiditis 4) infarct of the retinal nerve fibers A) 1st, 2nd and 3rd answers are correct B) 1st and 3rd answers are correct C) 2nd and 4th answers are correct D) only 4th answer is correct E) all of the answers are correct

D) only 4th answer is correct The microinfarct of the retinal nerve fibers causing the cotton-wool spots.

Admission to the given health care facility should be suspended if the disease(s) below take(s) place in the institution: 1) scarlet fever 2) meningitis caused by H. influenzae 3) invasive disease caused by Pneumococcus pneumoniae 4) meningitis epidemica A) answers 1, 2 and 3 are correct B) answers 1 and 3 are correct C) answers 2 and 4 are correct D) only answer 4 is correct E) all of the answers are correct

D) only answer 4 is correct Blocking the hospital admissions in the case of meningitis epidemica is necessary, because the probability of transmission of the pathogen is 50. For this reason, persons entering the area (hospital ward) are exposed to an increased risk of being infected. Moreover, admisson blocking is justified by the high lethatility of the disease which is 70% without antibiotic therapy, 15-20% with antibiotic therapy. For the other three diseases admission blocking is not necessary.

Which of the listed substances could cause hematopoietic cancer in humans due to their chronic exposure? 1) vinyl chloride 2) benzidine 3) nitrosamine 4) benzene A) answers 1, 2 and 3 are correct B) answers 1 and 3 are correct C) answers 2 and 4 are correct D) only answer 4 is correct E) all of the answers are correct

D) only answer 4 is correct Chronic effects of benzene could cause hematopoietic tumors, leukemia. Vinyl chloride could cause hemangiosarcomas, benzidine could cause bladder cancer, while nitrosamine exposure could cause stomach cancer.

What are the stochastic effects of ionizing radiation? 1) teratogenic effects 2) cataract 3) dermatitis caused by radiation 4) carcinogenic effect A) answers 1, 2 and 3 are correct B) answers 1 and 3 are correct C) answers 2 and 4 are correct D) only answer 4 is correct E) all of the answers are correct

D) only answer 4 is correct Stochastic effects are those effects the frequency and probability of which increses on elevating the doses, more precisely the dose equivalent value. There is no threshold dose in the case of these effects, and the dose-response curve runs through the origo (zero point). A typical example of the stochastic effects is the carcinogenic effect.

Drugs causing gingival hyperplasia 1) major tranquilizer drugs 2) penicillin group 3) minor tranquilizer drugs 4) dilantin -hydantoin group A) if 1., 2. and 3. are correct B) if 1. and 3. are correct C) if 2. and 4. are correct D) only the 4th answer is correct E) if all the answers are correct

D) only the 4th answer is correct Gingival hyperplasia can be side effects of many different drugs. One of them the dilantine (hydantoin) groups given to patients with epilepsy. After one or two months administration might cause gingival enlargement, The fibrotic tissues with minimal tendency to bleed sometime might cover the whole crows interfering with chewing. The proper oral hygiene in this case is very important.

Pyelonephritis gravidarum: 1) is mostly on the left side 2) therapy = pregnancy interruption 3) frequent in the first trimester 4) hydronephrosis can be detected with ultrasound A) the 1st, 2nd and 3rd answers are correct B) the 1st and 3rd answers are correct C) the 2nd and 4th answers are correct D) only the 4th answer is correct E) all of the answers are correct

D) only the 4th answer is correct Pregnancy pyelonephritis usually occurs in the second half of pregnancy, affects the right side more often. With the double J catheter or nephrostomy placed for the time of pregnancy, the urine can be led away, and the patient becomes asymptomatic.

The term "blow-out fracture" refers to a(n) A) frontobasal fracture. B) zygomatic fracture. C) cheekbone fracture. D) orbital floor fracture. E) frontal bone fracture.

D) orbital floor fracture. Blow-out fracture is a traumatic deformity of the orbital floor.

The patients has no ear pain in: A) perichondritis of the ear B) herpes zoster oticus C) angina D) otosclerosis E) lymphadenitis of the neck

D) otosclerosis In the case of otosclerosis, there is no pain but only hearing loss. *Otosclerosis - when a tiny bone inside the ear, called the stapes, joins (fuses) with other parts of the ear and stops the ability to hear properly.

Among the below listed structure-funtion pairs which one is incorrect? A) ciliary processes - aqueous production B) sclera - insertion site of the extraocular muscles C) lacrimal gland - to produce serous liquid D) papilla (optic nerve head) - place of sharp vision

D) papilla (optic nerve head) - place of sharp vision The papilla is not the site of the sharp vision, nerve fibers leaving the eyeball through the papilla (optic nerve head). The site of sharp vision is the central fovea in the central part of the macula.

The most frequently injured nerve during thyroid surgery is A) hypoglossal nerve B) accessory nerve C) superior laryngeal nerve D) recurrent laryngeal nerve E) brachial plexus

D) recurrent laryngeal nerve The recurrent laryngeal nerve is the nerve that most frequently gets injured during thyroid surgery, due to its localization.

Nasal congestion (obstructed nadal breathing) can be caused by 1) nasal polyposis 2) deviated septum 3) perforated septum 4) choanal atresia 5) laryngeal recurrent nerve palsy A) the 1st, 2nd and 3rd answers are correct B) the 2nd, 4th and 5th answers are correct C) the 3rd, 4th and 5th answers are correct D) the 1st, 2nd and 4th answers are correct E) all of the answers are correct

D) the 1st, 2nd and 4th answers are correct Nasal congestion can be caused by nasal polyposis, deviated septum, and chia all atresia.

What does it mean „ the completion of the dental arch" ? A) the eruption of the wisdom tooth B) the eruption of a mesiodens C) the eruption of a supernumeraly tooth D) the eruption of the lower first permanent molar E) all of them

D) the eruption of the lower first permanent molar During the eruption of the first permanent molars the dental arch built up of 5 teeth by quadrants will be completed, in this way those teeth will not be exploited or changed and any damage on those teeth will be irreversible. Therefore those teeth need more attention and care from the dentist and parents.

The dental caries is a fluoride deficiency disease BECAUSE the fluoride integrated into dental enamel can increase the acid resistance . A) both the statement and the reason are true, and the reason verifies the statement. B) both the statement and the reason are true ,but there is no relation between them C) the statement is true the reason is false D) the statement is false the reason is true E) both the statement and the reason are false

D) the statement is false the reason is true The caries is definitely not the consequence of the fluoride ion deficiency . Although the fluoride ions incorporated into the hydroxyapatite can facilitate the acid resistance by decreasing the acid solubility of the crystals (this is the so called dispositional prophylaxis).

The small arteriole on the dental pulp do not form an anastomosis THEREFOR they are functionally terminal arteriole . A) both the statement and the reason are true, and the reason verifies the statement. B) both the statement and the reason are true ,but there is no relation between them C) the statement is true the reason is false D) the statement is false the reason is true E) both the statement and the reason are false

D) the statement is false the reason is true The pulpal blood vessels of multi-rooted teeth can anastamose in the pulp chamber. Because functional collateral circulation does not exist in the pup if one of the arteriole gets blocked none of the other can substitute and restore the circulation. In this way the pulpal arteriole are considered as terminal arteriole.

If the urine flows through the small intestine constant bacteruria will be detected, so patients will have hyperchloremic acidosis. A) both the statement and the explanation are true and a causal relationship exists between them; B) both the statement and the explanation are true but there is no causal relationship between them; C) the statement is true, but the explanation is false; D) the statement is false, but the explanation itself is true E) both the statement and the explanation are false

D) the statement is false, but the explanation itself is true In the case of small bowel bladder replacement - if secondary superficial infection has not occurred - urine is not bacteriuretic. After bladder replacement with intestinal segregation, hyperchloraemic acidosis is always present. This is due to the fact that chloride and ammonium ion are absorbed from the intestine and due to the protons that are removed from the ammonium ion in the liver, the pH level of the blood is shifted in the acidic direction.

Adequate therapy of chronic tonsillitis includes A) suction of the tonsils B) squeezing the detritus out of the tonsils C) high doses of antibiotics D) tonsillectomy

D) tonsillectomy The treatment for chronic tonsillitis is tonsillectomy.

Which therapeutic option is appropriate for the treatment of allergic contact dermatitis? A) marigold ointment B) salicylic acid C) 5-fluorouracyl D) topical glucocorticoids E) none of the above

D) topical glucocorticoids

What type of exercise would you primarily recommend to an elderly patient with arthritis? A) cycling B) running C) gardening D) walking

D) walking The recommended exercise should be the walking, which duration and speed can be adjusted to the capacity of the patient and could be increased slightly with time. It wont put a great burden on the joints, it improves the patients' endurance, no tools are required and can be easily integrated into everyday activities.

Normal intraocular pressure does not exceed: A) 7 mmHg B) 11 mmHg C) 14 mmHg D) 18 mmHg E) 22 mmHg

E) 22 mmHg The normal intraocular pressure is below 22 mmHg.

Typical symptoms of lumbar discus herniation, EXCEPT A) Antalgic posture B) Strong lumbar back pain C) Motor weakness of the lower extremities D) Loss of tendon reflexes on the lower limbs E) Acute abdominal complaints

E) Acute abdominal complaints Lumbar discus herniation almost never causes acute abdominal complaints, however the other four symptoms are typical.

Which statement(s) is (are) true for gonorrheal urethritis? 1) Purulent urethral discharge, intracellular, Gram negative diplococci in the smear 2) Incubation time in men approx. 3 days; in women usually no symptoms 3) Therapy of first choice is ceftriaxone 4) Co-infections are relatively common, thus HIV, syphilis and chlamydial co-infections have to be excluded. A) Answers 1, 2 and 3 are correct. B) Answers 1 and 3 are correct. C) Answers 2 and 4 are correct. D) Only answer 4 is correct. E) All answers are correct

E) All answers are correct

In which disease is tick bite typical for the patients' history? 1) Lymphadenosis cutis benigna 2) Erythema chronicum migrans 3) Acrodermatitis chronica atrophicans 4) Lyme disease A) Answers 1, 2 and 3 are correct. B) Answers 1 and 3 are correct. C) Answers 2 and 4 are correct. D) Only answer 4 is correct. E) All answers are correct.

E) All answers are correct.

Which cutaneous manifestation(s) is (are) typical for a vasculitis? 1) Purpura 2) Persisting urtica for over 24 hours 3) Hemorrhagic papule 4) Skin necrosis A) Answers 1, 2 and 3 are correct. B) Answers 1 and 3 are correct. C) Answers 2 and 4 are correct. D) Only answer 4 is correct. E) All answers are correct.

E) All answers are correct.

Which factors influence the increasing prevalence of onychomycosis? 1) ageing of population 2) increased number of immunosuppressed patients 3) frequent use of broad spectrum antibiotics 4) changed lifestyle (occlusive shoes, sport and leisure activities) A) Answers 1, 2 and 3 are correct. B) Answers 1 and 3 are correct. C) Answers 2 and 4 are correct. D) Only answer 4 is correct. E) All answers are correct.

E) All answers are correct.

Which side effects of a therapy with isotretinoin are most common? 1) Dry mouth 2) Conjunctivitis 3) Increased serum cholesterol levels 4) Cheilitis A) Answers 1, 2 and 3 are correct. B) Answers 1 and 3 are correct. C) Answers 2 and 4 are correct. D) Only answer 4 is correct. E) All answers are correct.

E) All answers are correct.

Which therapeutic option(s) is (are) appropriate for the treatment of cutaneous T cell lymphomas? 1) PUVA 2) Bone marrow transplantation 3) α-interferon 4) X-ray irradiation A) Answers 1, 2 and 3 are correct. B) Answers 1 and 3 are correct. C) Answers 2 and 4 are correct. D) Only answer 4 is correct. E) All answers are correct.

E) All answers are correct.

Which therapeutic option(s) is (are) appropriate for the treatment of alopecia areata? A) Topical glucocorticoid treatment B) PUVA (psoralen + UVA) C) Provocation of a local contact dermatitis D) Systemic treatment of acive bacterial foci E) All of the above

E) All of the above

NOT a symptom of Scheuermann's disease: A) Dorsal hyperkyphosis B) Forwardly wedge-shaped vertebrae formation C) Uneven vertebrae closing plates D) Moderate back pain of teenagers E) Association of avascular necrosis of femoral head

E) Association of avascular necrosis of femoral head Avascular necrosis of femoral head is characteristic to another juvenile osteochondritis, Perthes' disease.

What medicine is NOT used to treat rheumatoid arthritis? A) Methotrexate B) Leflunomide C) Methylprednisolone D) Diclofenac E) Colchicine

E) Colchicine Colchicine is used to treat gout attack; the listed base (first-line) therapeutic agents, corticosteroids and non-steroidal anti-inflammatory agents are all used for treating rheumatoid arthritis.

It does NOT play an important role in the pathogenesis of rheumatoid arthritis: A) Activation of Th1 line B) Activation of the Th17 line C) Activation of the Th2 line D) Decreased operation of Treg cells E) Decreased formation of cathepsins

E) Decreased formation of cathepsins In the pathogenesis of RA, the activation of the Th1, Th2, Th17 lines and the decreased function of Treg cells are both involved. Cathepsins are involved in later bone- and cartilage degradation, but rather by increased formation not decreased.

Which clinical symptom is NOT characteristic of rheumatoid arthritis? A) Early morning stiffness in the hands B) Symmetrical small joint inflammation of the hands C) Small joint erosions on X-ray D) Anti-CCP positivity E) Early low back pain with sacroiliitis

E) Early low back pain with sacroiliitis The first four symptoms are specific to RA, early low back pain with sacroiliitis is a typical symptom of seronegative spondyloarthritis.

Characteristics of joint processes associated with inflammatory bowel diseases (IBD), EXCEPT A) Especially lower extremities oligoarthritis B) Spondarthritis of the spine C) Effectiveness of infliximab therapy D) Effectiveness of sulfasalazine therapy E) Effectiveness of methotrexate therapy

E) Effectiveness of methotrexate therapy Infliximab and sulfasalazine are effective in IBD however methotrexate isn't.

Cadmium of the soil is accumulated in the following plant(s): 1) tobacco plant 2) leafy vegetables 3) potato 4) cereals A) answers 1, 2 and 3 are correct B) answers 1 and 3 are correct C) answers 2 and 4 are correct D) only answer 4 is correct E) all 4 answers are correct

E) all 4 answers are correct All listed plants are able to accumulate cadmium absorbed from the soil, their consuming (smoking, eating) leads to chronic metal exposition. This represents a particular danger, especially when the Cd content level is high in the soil mainly due to industrial activity.

What condition does NOT lead to increased formation of osteoarthritis? A) Hip joint dysplasia B) Significant weight gain C) Varus / valgus deformity D) Excessive arthritis E) HLA B-27 positivity

E) HLA B-27 positivity HLA B-27 positivity does not predispose to osteoarthritis, but the other factors listed are risk factors.

What is the most important laboratory symptom of primary osteoporosis? A) Elevated alkaline phosphatase B) Elevated erythrocyte sedimentation rate C) Elevated parathyroid hormone levels D) Decreased serum calcium levels E) None of the above

E) None of the above In case of primary osteoporosis the routine laboratory parameters are negative.

Which symptom is NOT typical to lower leg nerve root compression? A) Positive Lasegue-sign B) Tenderness at the Valleix points C) Positive femoral sign D) Loss of the Achilles tendon reflex E) Patellar ballottement

E) Patellar ballottement Patellar ballottement is typical to knee joint fluid, the four other symptoms could be the sign of nerve route compression.

Symptoms of sarcoidosis, EXCEPT: A) Granulomatous systemic process B) Bilateral hilaris lymphadenopathy C) Erythema nodosum D) Arthritis E) Polychondritis

E) Polychondritis Polychondritis does not occur in sarcoidosis.

What symptom group is NOT characteristic of Lyme disease? A) Erythema migrans B) Arthralgia - Arthritis C) Neurological lesions D) Cardiac changes E) Severe gastrointestinal symptoms

E) Severe gastrointestinal symptoms Severe gastrointestinal symptoms are extremely rare in Lyme disease, oppositely to the others.

Which feature is not characteristic for dysplastic nevi? A) Their diameter is >5 mm B) They often have more than one colors (e.g., red, blue, light and dark brown, black) C) They are almost always asymmetric D) Their edges are usually irregular, gyrated E) They are inborn skin lesions

E) They are inborn skin lesions

The management of an abscess developed at deciduous teeth A) immediate extraction B) access cavity preparation with open therapy C) access cavity preparation and putting a filling afterwards D) the abscess is eliminated with a Volkmann curette E) access cavity preparation and the abscess is drained and excavated

E) access cavity preparation and the abscess is drained and excavated It is no use to extract the deciduous molars - because of their space maintaining rolls. The filling is an inadequate therapy . The access cavity preparation or the drainage of the parulis will not solve the case. A comprehensive therapy should be performed.

Which of the followings is/are the main fields of health policies? 1) Supporting health oriented decisions to be made 2) Ensuring equal access to care 3) Health protection and health promotion 4) Preventing special health problems A) Answers 1, 2 and 3 are correct B) Answers 1 and 3 are correct C) Answers 2 and 4 are correct D) only 4 is correct answer E) all 4 answers are correct

E) all 4 answers are correct All 4 areas listed in the question (support for health-oriented decision-making in all public areas, ensuring equal access to health care for all, health protection ad promotion, prevention, detection and treatment of specific health problems) are the main areas of health policies in most countries of the world. It should be noted that the term policy in health policy is used as the way to achieve certain goals.

Which of the following wastes is/are considered as hazardous waste(s)? 1) manure of livestock farms 2) unused pesticides 3) medical waste 4) household chemical products A) Answers 1, 2 and 3 are correct B) Answers 1 and 3 are correct C) Answers 2 and 4 are correct D) only answer 4 is correct E) all 4 answers are correct

E) all 4 answers are correct All four types of waste listed - such as livestock manure, pesticides that are not used for plant protection, wastes generated by health care activities and household chemical products - are considered as hazardous wastes due to the infectivity and / or toxicological effects.

Pregnant women are not allowed to be employed at a given workplace if it means exposition to the following metal(s): 1) Nickel 2) Lead 3) Mercury 4) Arsenic A) answers 1, 2 and 3 are correct B) answers 1 and 3 are correct C) answers 2 and 4 are correct D) only answer 4 is correct E) all 4 answers are correct

E) all 4 answers are correct Employing pregnant women exposed to any metal listed is contraindicated.

The following quality criteria can be specified for user protection at healthcare websites: 1) the website should contain the exact name, postal and electronic address of the service provider 2) authors of the website should be easily identified 3) the last update must be indicated clearly 4) If the site recommends the use of medications, then it must indicate that the recommendation does not replace the personal advice of professionals. A) Answers 1, 2 and 3 are correct B) Answers 1 and 3 are correct C) Answers 2 and 4 are correct D) only answer 4 is correct E) all 4 answers are correct

E) all 4 answers are correct For healthcare websites, the following quality criterias can be formulated regarding user protection: 1. Transparency (It is generally expected that the website must contain the exact name, the postal and electronic address of the service provider, the target group of the publication , the aim of the site and a listing of the a participants in the development of the webpage, as well as its supporters.) 2. Authenticity of the data source (The website must display the data that enables user to identify the authors of the site,it must also display the sources of the published information together with the date of publication. If the use of any methods, medicine or devices is recommended by the website, then it must be indicated that the recommendation does not replace the personal advice of professionals. Date of the last update must be also shown on the website.) 3. Responsibility (It must provide options for user feedback, it also has to indicate who will be responsible for answering these user inputs.).

In case of a suspected mushroom poisoning: 1) the patient should be vomited or gastric lavage must be performed 2) persons suspected of mushroom poisoning should be observed for 48 hours 3) stomach lavage liquid, food leftovers, kitchen wastes should be sent for laboratory testing 4) the Public Health Authority should be notified in a written reporting form A) answers 1, 2 and 3 are correct B) answers 1 and 3 are correct C) answers 2 and 4 are correct D) only answer 4 is correct E) all 4 answers are correct

E) all 4 answers are correct In case of mushroom poisoning the main components of the first aid are the induction of vomiting and / or performing gastric lavage as soon as possible after the consumption of mushrooms. People suspected of having mushroom poisoning are also advised to be hospitalized even in case of having only mild symptoms because the person who picked the mushrooms could collect other species too, including not only moderately harmful ones with short latency and early symptoms but also lethal ones with longer latency (eg death-cup). If poisoning is suspected, further consumption of the food containing mushroom components must be prohibited, the rest of the food, the kitchen cleaning waste, a portion of the gastric lavage fluid and the vomit, respectively should be sent for laboratory testing. For the prevention of mushroom-induced poisoning the most important task is the examination of the collected mushrooms by a mushroom expert.

Which of the following activities is/are the most popular method(s) of quality improvement (PDCA)? 1) Planning 2) Action 3) Check (control) 4) Feedback A) answers 1, 2 and 3 are correct B) answers 1 and 3 are correct C) answers 2 and 4 are correct D) only answer 4 is correct E) all 4 answers are correct

E) all 4 answers are correct The PDCA acronym used in quality improvement consists of the initials of the English words plan, do, check, act. It refers to the quality improvement cycle that consists of planning (necessary changes, preparation and planning of interventions), action (implementation of changes), control (impact assessment) and feedback.

What are the most frequent sensory changes affecting the elderly? 1) Cataract 2) Presbyacusis 3) Presbyopia 4) Deterioration of the equilibrium A) answers 1, 2 and 3 are correct B) answers 1 and 3 are correct C) answers 2 and 4 are correct D) only answer 4 is correct E) all 4 answers are correct

E) all 4 answers are correct The incidence of those diseases is high in the elderly, which are not so important as the direct cause of death but severely impair the life quality. The most common sensory changes are the ones effecting the vision (refraction disorders - presbyopia, cataract), hardness of hearing (presbyacusis), and loss of equilibrium (vestibular disorders).

Strategy for the prevention of accidental injuries and accidental deaths: 1) its aim is the accident control instead of accident prevention 2) the best known strategy is Haddon's strategy 3) is mainly not related to public health 4) focuses on the safe designing of vehicles and trafic areas A) answers 1, 2 and 3 are correct B) answers 1 and 3 are correct C) answers 2 and 4 are correct D) only answer 4 is correct E) all 4 answers are correct

E) all 4 answers are correct The strategy of preventing accidental injuries and accidental deaths is basically not a designed public health field. Haddon's technical accident control strategy is consisted of ten, logically coherent strategic steps, which describe safe designing of the vehicles, traffic areas, etc. Literature is using the term injury controll mostly instead of using the term accident prevetnion (eg the safety belt is for accident prevention, but it can also cause injuries)

To prevent the development of type 2 diabetes, the followings is/are recommended: 1) Weight loss 2) Regular physical exercise 3) Consumption of dietary fibers 4) Limiting the intake of saturated fats A) Answers 1, 2 and 3 are correct B) Answer 1 and 3 are correct C) Answers 2 and 4 are correct D) only 4 is correct answer E) all 4 answers are correct

E) all 4 answers are correct Type 2 diabetes develops through the interaction of genetic and environmental factors. A typical diabetinogenic diet has high energy density, contains high amount of saturated fat and low amounts of dietary fibers. In epidemiological studies strong correlation was found between the increase of body mass index (especially central obesity) and the development of type 2 diabetes. Worldwide, 44% of diabetes risk is attributed to overweight and obesity. Daily intense physical activity or sports activity increases insulin sensitivity and thus reduces the risk of diabetes.

Symptoms of vitamin A deficiency: 1) blindness 2) xerophtalmia 3) keratomalacia, complete blindness 4) increased susceptibility to infectious diseases A) answers 1, 2 and 3 are correct B) answers 1 and 3 are correct C) answers 2 and 4 are correct D) only answer 4 is correct E) all 4 answers are correct

E) all 4 answers are correct Vitamin A is essential for the following functions: vision, intactness of cornea, skin and mucous membranes, reproduction and growth. In the absence of vitamin A hemeralopia, xerophthalmia, keratomalacia, complete blindness as well as skin and mucosal lesions could occur. The immune system is damaged, infections are more common and the outcomes are more severe, growth slows down.

Acute kidney failure laboratory values: 1) hyperkalemia 2) hypermagnesemia 3) acidosis 4) hyponatremia A) the 1st, 2nd and 3rd answers are correct B) the 1st and 3rd answers are correct C) the 2nd and 4th answers are correct D) only the 4th answer is correct E) all of the answers are correct

E) all of the answers are correct

Obesity increases the risk of: 1) hypertension 2) Type 2 diabetes 3) breast cancer 4) colorectal cancer A) answers 1, 2 and 3 are correct B) answers 1 and 3 are correct C) answers 2 and 4 are correct D) only answer 4 is correct E) all of the answers are correct

E) all of the answers are correct According to the World Health Organization, overweight and obesity are the second most important preventable risk factors after smoking. They play an important causal role in the development of hypertension, cardiovascular diseases, cerebral vascular diseases (stroke), type 2 diabetes. It's a risk factor for many cancers (breast, endometrium, kidney, colorectal carcinoma, pancreas, esophagus).

In which secretion of the patient can the Lyssavirus be detected? 1) saliva 2) tears 3) urine 4) milk A) answers 1, 2 and 3 are correct B) answers 1 and 3 are correct C) answers 2 and 4 are correct D) only answer 4 is correct E) all of the answers are correct

E) all of the answers are correct After the virus has entered the central nervous system, it begins to grow intensely and then travels along the efferent nerves to all organs. As a result of this, it will appear in all tissues and - listed - excretions.

Which of the listed wastes can not be destroyed by incineration? 1) radioactive waste 2) waste containing more than 40% chlorine 3) which causes the formation of carcinogenic substances during incineration 4) which have heavy metal content of more than a few percent A) answers 1, 2 and 3 are correct B) answers 1 and 3 are correct C) answers 2 and 4 are correct D) only answer 4 is correct E) all of the answers are correct

E) all of the answers are correct All waste types listed - radioactive waste, waste containing more than 40% chlorine, those which cause the formation of carcinogenic substances during their burning, which have a heavy metal content of more than a few percent are not permitted to deal with in waste incinerators. These wastes must be preserved/settled in special waste cemeteries.

An indicator is considered as well defined when 1) its calculation is based on a standardized method. 2) it is connected to interventions. 3) its results can be compared in different populations. 4) it is easy to interpret. A) answers 1, 2 and 3 are correct B) answers 1 and 3 are correct C) answers 2 and 4 are correct D) only answer 4 is correct E) all of the answers are correct

E) all of the answers are correct An indicator can be considered as well defined, when it can be easly interpreted, it is connected to intervention possibilites, correlates with other indicators, is timely, can be produced in all study populations, its raw data originate from reliable data sources, does not need too many resources, its calculation is based on standardized methodology, it reflects reality in a valid manner, its values are comparable in different populations, and is related to a real public health issue.

Cause(s) of erectile dysfunction: 1) antihypertensive drugs 2) induratio penis plastica 3) prostatovesiculitis 4) alcoholism A) the 1st, 2nd and 3rd answers are correct B) the 1st and 3rd answers are correct C) the 2nd and 4th answers are correct D) only the 4th answer is correct E) all of the answers are correct

E) all of the answers are correct Antihypertensive drugs can cause erectile dysfunction through reduced arterial inflow. In plastic penile induration, the penile curvature may reach such a high level, that it leads to erectile dysfunction. Prostatovesiculitis usually causes premature ejaculation. Chronic alcoholism can also lead to erectile dysfunction.

Liver diseases and cirrhosis in Hungary 1) accounts for approximately 80% of premature death cases caused by gastrointestinal diseases 2) about 80% of these diseases has an alcoholi-related origin 3) their high prevalence can be explained by the amount of alcohol consumed 4) their high prevalence can be explained by the quality of alcohol consumed A) answers 1, 2 and 3 are correct B) answers 1 and 3 are correct C) answers 2 and 4 are correct D) only answer 4 is correct E) all of the answers are correct

E) all of the answers are correct Diseases of the liver and cirrhosis are responsible for 80% of premature death caused by gastrointestinal disorders in Hungary. Approximately 80% of these diseases has an alcohol-related origin, their high prevalence can be explained by the amount and quality of alcohol consumed (consumption of alcoholic beverages contaminated by aliphatic alcohols and obtained from illegal sources)

Dysuria can be a symptom of: 1) bladder stone 2) prostate cancer 3) acute prostatitis 4) phimosis A) the 1st, 2nd and 3rd answers are correct B) the 1st and 3rd answers are correct C) the 2nd and 4th answers are correct D) only the 4th answer is correct E) all of the answers are correct

E) all of the answers are correct Dysuria may be caused by urethral compression of the BPH, prostate cancer obstructing the pars prostatica, inflammation and enlargement of prostate. And so does the punctually narrowed form of phimosis too, where the patient urinates under the foreskin, around glans.

Ecological studies: 1) usually produce correlation indicators 2) create and analyse scatter diagrams 3) are based on data collection at population level 4) evaluate relations of parameters of various groups A) answers 1, 2 and 3 are correct B) answers 1 and 3 are correct C) answers 2 and 4 are correct D) only answer 4 is correct E) all of the answers are correct

E) all of the answers are correct Ecological studies are a specific type of descriptive epidemiological studies which examine population level connections of characeristics on the basis on aggregated data by evaluating correlation indicators. Consequently, ecological studies are also called correlational studies. The two names emphasize different features of the same method. The term "ecological study" indicates that the examination is based not on individual data but on the average parameters of populations (or their subgroups). The term "correlation study" indicates that the direction and the strength of the correlation is described by correlation coefficients or regression equations. Results of the ecological studies are usually presented on scatter diagrams.

Epidemiology: 1) uses population level methods 2) examines the distribution of health status/phenomena 3) examines the distribution of factors that affect health status 4) examines the distribution of changes in health status/phenomena A) answers 1, 2 and 3 are correct B) answers 1 and 3 are correct C) answers 2 and 4 are correct D) only answer 4 is correct E) all of the answers are correct

E) all of the answers are correct Epidemiology is a scientific field that observs the distribution and changes of those determinants that influence the health-related states or events in a given population in order to use its findings in the field of health promotion, disease prevention, medical care and rehabilitation.

Which disease(s) is (are) spread by cats? 1) Q fever 2) Lyssa (rabies) 3) Tularemia 4) Toxoplasmosis A) answers 1, 2 and 3 are correct B) answers 1 and 3 are correct C) answers 2 and 4 are correct D) only answer 4 is correct E) all of the answers are correct

E) all of the answers are correct In all listed diseases cats may be the sources of infection although the frequency may vary. The role of cats is less important in the transmisson of Q fever and tularemia than the role of other animals. Cats belong, however, to the most important sources of Rabies; the most recent lethal Rabies cases were due to cats because of the late diagnosis of the infections The role of the cats in the transmission of toxoplasmosis should be especially emphasized. Toxoplasmosis is a teratogenic infection. Since 10-15% of cats - and not only those living free but also the domestic ones - are constantly infected with Toxoplasma and shed the pathogens with their feces, pregnant women should avoid direct contact with them.

Indications of tonsillectomy are 1) chronic tonsillitis 2) tonsillar hypertrophy in sleep apnoe syndrome 3) peritonsillar abscess 4) tonsillogenic sepsis 5) suspected focus of tonsillogenic origin A) the 1st and 5th answers are correct B) the 1st, 3rd and 5th answers are correct C) the 2nd, 3rd and 5th answers are correct D) the 3rd, 4th and 5th answers are correct E) all of the answers are correct

E) all of the answers are correct Indications for tonsillectomy includes chronic tonsillitis, peritonsillar abscess, tonsillar hypertrophy in sleep apnoe syndrome, tonsillogenic sepsis, suspected focus of tonsillogenic origin.

In which case(s) should chemoprophylaxis be used on the contact persons? 1) Diphtheria 2) Meningitis epidemica 3) H. influenzae meningitis 4) Pertussis A) answers 1, 2 and 3 are correct B) answers 1 and 3 are correct C) answers 2 and 4 are correct D) only answer 4 is correct E) all of the answers are correct

E) all of the answers are correct People who were in direct contact with the patient infected with any of the four listed diseases should receive antibiotic prophylaxis.

What are the risk factors of preterm birth? 1) young age of the mother (under 19 years) 2) lower education level of the mother 3) previous abortion 4) diabetic mother A) answers 1, 2 and 3 are correct B) answers 1 and 3 are correct C) answers 2 and 4 are correct D) only answer 4 is correct E) all of the answers are correct

E) all of the answers are correct Preterm birth is more than five times higher among women with elementary school qualification than in mothers with a university degree. It is more common among adolescent mothers. Risk factors of preterm birth are the followings: previous artificial abortion, multiple earlier pregnancies, infections, diabetes and hypertension as chronic non-communicable diseases.

Cause of urinary total retention: 1) bladder tumor 2) prostatitis acuta 3) bladder stone 4) bladder paralysis A) the 1st, 2nd and 3rd answers are correct B) the 1st and 3rd answers are correct C) the 2nd and 4th answers are correct D) only the 4th answer is correct E) all of the answers are correct

E) all of the answers are correct The bladder tumor located around the internal orifice can block the orifice or infiltrate the back tract of urethra. Bladder stones embedded in the orifice or in the back track of urethra may also cause complete retention. During acute prostatitis, the inflamed, enlarged prostate may compress the prostatic urethra completely. In case of bladder paralysis, the detrusor muscle does not work, so it can not empty the urine.

Final disinfection is mandatorial in the following infectious diseases: 1) Cholera 2) Infectious hepatitis 3) Diphtheria 4) Salmonellosis A) answers 1, 2 and 3 are correct B) answers 1 and 3 are correct C) answers 2 and 4 are correct D) only answer 4 is correct E) all of the answers are correct

E) all of the answers are correct The listed diseases may be transmitted by the faeco-oral route and by body fluids, consequently, continuous and final disinfection is needed in order to prevent the infection from spreading. Final disinfection with more strict regulations has to be carried out in case of cholera infection.

Epidemiological studies refer to populations which 1) can be defined geographically 2) can be defined in time 3) members have one or more common characteristics 4) members do not have one or more charateristics A) answers 1, 2 and 3 are correct B) answers 1 and 3 are correct C) answers 2 and 4 are correct D) only answer 4 is correct E) all of the answers are correct

E) all of the answers are correct The study of health related states and determinants is always carried out in a given - well-defined - population. Such a population may be a group of individuals belonging together or merged on the basis of their unique/specific characteristics (eg pupils of a particular grade of a given school, workers or other employees of a particular factory or other the institution, people who have been treated with a certain surgical procedure). Alternatively, a population may be formed by the residents of a given administrative unit (eg the inhabitants of a country/region/county/settlement at a given time or interval.

The oral signs and symptoms of AIDS : A) Kaposi-sarcoma B) severe progressing periodontitis C) leukoplakia D) candidiasis E) all of them

E) all of them All the listed conditions might associate with AIDS, their cause is the severely compromised immune status of the patient.

The characteristic sing of periodontal abscess : A) develops after the obturation of the orifice of the pocket B) strong pulsating pain C) the involved tooth has vital sign D) a putrid charge from the sulcus E) all of them

E) all of them All the symptoms are characteristic of periodontal abscesses. The pyogenic exudates in the closed pocket will increase the pressure and this leads to acute pain and many ties can mimic the symptoms of the periapical abscess. On the other hand the tooth is vital . With pressing the pocket wall puss can be discharged from the sulcus. The location of the periodontal abscess helps the differential diagnosis, to differentiate from the acute periapical abscess. It is closer to the gingival margin than to the apex. It is usually smaller and can be drained thru the sulcus.

In the context of health education A) We use purposeful learning opportunities to transfer knowledge on health-related information B) we can create an internet portal C) we can use personalized advice D) we can organize an advertising campaign E) all of them could be applied

E) all of them could be applied Health education's aim is to increase the individuals' health-related knowledge and skills by creating learning opportunities with adequate methods (individual and group meetings, live and recorded lectures, printed and electronic communication, websites).

Average life expectancy at birth in Hungary between 1988 and 1993 1) for men, it has been gradually dropping behind the Austrian reference level 2) For women, it has been increasing steadily 3) For women, it has been gradually dropping behind the Austrian reference level 4) for men, it has been decreasing steadily A) statements 1, 2 and 3 are corrrect B) statements 1 and 3 are correct C) statements 2 and 4 are correct D) only statement 4 is correct E) all statement are correct

E) all statement are correct As for men, the average life expectancy at birth calculated on the basis of annual mortality indicators decreased slightly between 1988-1993 - which was a unique phenomenon in Europe. On the contrary, slightly but consequently increasing trend has been observable in case of women since 1980. However, in both genders this indicator is far behind the same indicators of not only the most developed countries, but also of Czechia and Poland, the previous sister states showing similar historical fate. When comparing the Hungarian indicator regarding the average life expectancy to that of the male and female population of Austria (a country that has similar geografical location and - apart from the second half of the 20th century similar history) the lagging behind was showing a constantly increasing trend from 1980 to the millenium and has been stable since that. This lag was 7.9 years in case of men and 5.1 years in case of women in 2008.

Which examination is not suitable for making diagnosis of a cutaneous vasculitis? A) dermatohistopathological examination of lesional skin B) direct immunofluorescent microscopy of lesional skin C) hepatitis C serology D) measurement of serum levels of cryoglobulin and cryofibrinogen E) angiography

E) angiography

Funduscopy of the right eye should be made in the following order, except: A) the examiner sits or stands on the right side of the patient B) the retinoscope is held in the right hand of the examiner and he/she examines with the right eye C) possibly performs the examination in a dark room D) the left hand is positioned on the forehead of the patient and with her/his thumb lifts the upper eyelid E) asks the patient to look directly into the light

E) asks the patient to look directly into the light During direct funduscopy the right eye should be examined by the right eye, the left eye with the left one, the approach is easier that way. The examination should be carried out in a dark room, eyelid might be lifted. Patients should not look directly into the light, because in that case the pupil narrows and the peripheral part of the fundus cannot be observed.

Which diagnosis cannot be the underlying cause of a secondary Raynaud syndrome? A) SLE B) systemic scleroderma C) cervical rib D) cryoglobulinaemia E) atopic dermatitis

E) atopic dermatitis

In case of acute renal colic the first choice should be narcotic pain medication, so patients having renal colic should be directed to urology department immediately. A) both the statement and the explanation are true and a causal relationship exists between them; B) both the statement and the explanation are true but there is no causal relationship between them; C) the statement is true, but the explanation is false; D) the statement is false, but the explanation itself is true E) both the statement and the explanation are false

E) both the statement and the explanation are false Renal colic with fever, chills and without vomiting can be treated in the patients home with iv. or im. analgesics and antispasmodics (eg Algopyrin, No-Spa). However, in the case of no response of medication, an institute referral is required. Here, with the assured diagnosis, we can add stronger painkillers. However, in order to avoid masking the symptoms of any other acute abdominal complaints on the spot, narcotic analgesics are prohibited.

Red eye can be observed in the followings, except: A) keratitis cum hypopyo B) acute conjunctivitis C) acute glaucomatous attack D) acute iridocyclitis E) corneal sign of the Wilson disease

E) corneal sign of the Wilson disease In Wilson disease a greenish-yellowish ring can be observed at the limbus (Kayser-Fleischer ring). Red eye is not observed in Wilson disease.

The malignant melanoma of the eye can be differentiated most accurately from the benign nevus by which of the following signs? A) colour B) shape C) size D) localization E) does it lift (grows) above the plane of the retina

E) does it lift (grows) above the plane of the retina The tumour is heighted largely above the plane of the retina. Nevi are usually in the plane of the retina or very flat. In case of uncertain diagnosis fluorescein angiography is necessary.

Adenotomy is indicated in A) chronic sinusitis B) otitis media C) sleep apnoe D) adenoid hypertrophy E) each of the above conditions

E) each of the above conditions Adenotomy is indicated in chronic sinusitis, frequent otitis media, sleep apnoe, adenoid hypertrophy. In these cases, the nose breathing and the Eustachian tube function improve.

The following clinical features are characteristic for porphyria cutanea tarda except A) blisters in sun-exposed skin areas B) hyperpigmentation C) hypertrichosis D) elevated serum liver transaminases E) green discoloration of urine

E) green discoloration of urine

A 25-year-old male patient complains in the emergency room of having an inflamed red eye. Fluorescein staining shows branch like corneal ulcer. The most likely diagnosis: A) corneal abrasion B) foreign body in the cornea C) heat induced corneal damage D) bacterial origin of corneal inflammation (keratitis) E) herpetic infection

E) herpetic infection In younger age the one-sided corneal inflammation, which shows branch like ulceration means a herpetic infection.

In case of a closed pulpitis the anatomical situation are very disadvantageous from the inflammation point of view 1) The volume of the closed pulp cannot be expanded and one of the leading sign of any inflammations is the swelling 2) In inflammation in a closed system the inflammatory swelling leads to a pressure increase in the tissues 3) the active hyperemia is followed by a passive venous hyperemia that leads to increased pressure and finally stasis in the pulp 4) the stasis arrest the circulation in the pulp that finally results in tissue necrosis A) if 1., 2. and 3. are correct B) if 1. and 3. are correct C) if 2. and 4. are correct D) only the 4th answer is correct E) if all the answers are correct

E) if all the answers are correct An inflammation process in a closed space by the concomitant swelling will evidently lead to tissue pressure increase. The listed signs are all the causative factors or the consequences of this mechanism.

It is characteristic of the forceps used for the removal of an upper fractured root 1) the beaks closely approximate each other 2) the beaks are a straight continuation of the hinge 3) the beaks set in a wide angle to the hinge 4) the width of the beaks are divided and pointed A) if 1., 2. and 3. are correct B) if 1. and 3. are correct C) if 2. and 4. are correct D) only the 4th answer is correct E) if all the answers are correct

E) if all the answers are correct The beaks of the root tip forceps meet the tip. For upper front teeth the beaks are symmetrical and placed in the same line as the handle. For molars the beaks are at the wide angle to the handle. The size of the beaks are variable according to the diameter of the root.

Oezena A) is a form of atrophic rhinitis B) involves foul-smelling lesion of the mucosa C) has unknown etiology D) becomes better with aging E) is characterized by all the above statements

E) is characterized by all the above statements Atrophic oezema is characterized by a foul smelling fluid, where it's etiology is unknown and is shown to be improving with aging.

Which is a precancerous lesion? A) fibromatosis gingivae B) lingua pilosa nigra C) Fordyce's granules D) geographic tongue E) leukoplakia

E) leukoplakia The leukoplakia (white plaque) is a premalignant lesion and the prevalence of malignant transformation is ranging between 2-6% (see also FOG.93) The other entities are not premalignant conditions

Tongue burn is not present in A) pernicious anemia B) vitamin deficiency C) glossitis D) Plummer-Vinson syndrome E) lingua nigra pilosa (black hairy tongue)

E) lingua nigra pilosa (black hairy tongue) Tongue burn is not present in lingua nigra pilosa.

A 67 year-old man suddenly loses his vision in the right eye. Funduscopy reveals a pale retina, in the macular area a cherry-red spot can be detected. What is the most likely diagnosis? A) occlusion of the central retinal vein B) multiple sclerosis C) carotidal aneurysm D) diabetic retinopathy E) occlusion of the central retinal artery

E) occlusion of the central retinal artery The sudden loss of central vision means the quick deterioration of the central fovea, which is a consequence of the occlusion of the central retinal artery. The retina getting edematous, within 24 hrs it is going to have a milky white color. This bleaching effect does not occur in the area of the central fovea, therefore its colour stays red (cherry red spot) after the occlusion of the central retinal artery. In this area the four inner layer of the retina are missing, therefor the red colour of the choroid can be detected.

Possible complications of mastoiditis do not include: A) meningitis B) subperiosteal abscess C) cavernous sinus trombosis D) Bezold-abscess E) otogenic hidrocephalus

E) otogenic hidrocephalus Mastoiditis can cause a variety of different complication, but otogenic hydrocephalus is not one of them. *Otogenic hidrocephalus - a syndrome of raised intracranial pressure associated with middle ear infection with normal CSF.

Peripheral facial palsy cannot be caused by: A) polyomyelitis B) herpes zoster oticus C) mastoiditis D) chronic otitis media with cholesteatoma E) otosclerosis

E) otosclerosis Peripheral facial palsy cannot be caused by otosclerosis. Though it can be caused by poliomyelitis, Herpes Zoster oticus, mastoiditis, chronic otitis media with cholesteatoma.

A patient perceives a sudden opacity in front of one eye. Some weeks later a progressive peripheral visual field defect is started in that particular eye. The most likely diagnosis: A) retinitis pigmentosa B) luxation of the crystalline lens C) iritis D) macular degeneration E) retinal detachment

E) retinal detachment The sudden peripheral visual filed defect - especially following vitreous floaters - sign of retinal detachment. In macular degeneration central visual filed defect is the consequence. In pigmentary retinitis the peripheral visual filed narrowing lasts longer, to years or to decades.

As a result of the diagnostic steps a prostate cancer of stage T3cN1M1 is found, the treatment can be: A 65-year-old man comes to the clinic with a complaint of frequent urination. He also complains about low back and leg pain. There is no blood in the urine, urination is not painful, comes in weak yellow stream. 1) radical prostatectomy 2) bilateral orchiectomy 3) hormone therapy with LHRH agonists 4) Anti-androgen therapy 5) estrogen hormone therapy A) the 1st, 2nd and 3rd answers are correct B) the 1st, 3rd and 4th answers are correct C) the 1st, 4th and 5th answers are correct D) the 1st, 3rd and 5th answers are correct E) the 2nd, 3rd and 4th answers are correct

E) the 2nd, 3rd and 4th answers are correct A T3c, N1, M1 stage prostate cancer is a late metastatic carcinoma, so only palliative treatment is possible. This is a method of suspending androgenic activity, which can be achieved by surgical or pharmacological castration (reduction of androgenic hormone levels), antiandrogens (receptor inhibition). Radical prostatectomy can only be performed in early stage prostate cancer, as it is already late.

Ototoxic antibiotics include 1) chloramphenicol 2) streptomycin 3) tetracyclin 4) gentamycin 5) neomycin A) the 1st, 2nd and 3rd answers are correct B) the 2nd, 3rd and 4th answers are correct C) the 1st, 4th and 5th answers are correct D) the 1st, 3rd and 5th answers are correct E) the 2nd, 4th and 5th answers are correct

E) the 2nd, 4th and 5th answers are correct Ototoxic antibiotics include streptomycin, gentamycin and neomycin, and so must be administrated carefully.

The one-sided subconjunctival hemorrhage which is not associated with pain, nor visual disturbances refers to: A) acute glaucomatous attack B) usually thrombocytopenia is in the background C) usually it is caused by arteriolosclerosis D) urgent, severe ophthalmic disturbance which needs emergency care E) usually harmless idiopathic disturbance

E) usually harmless idiopathic disturbance The cause is rupture of one (or more) conjunctival vessels, there is no visual consequence, painless.

Reinke-oedema is located on/in the A) epiglottis B) uvula C) piriform sinus D) vallecules E) vocal chords

E) vocal chords Reinke oedema is located at the vocal cords, and is prominent in heavy smokers.

Join the given words marked with letters to the proper numbers below. A) enamel B) dentin C) both of them D) neither of them FOG - 51 - It continuously growth throughout the life FOG - 52 - Its major histological component is the hydroxyapatite. FOG - 53 - Its structure is highly vascularised

FOG - 51 - It continuously growth throughout the life - B) FOG - 52 - Its major histological component is the hydroxyapatite - A) FOG - 53 - Its structure is highly vascularised - D)

Join the given words marked with letters to the proper numbers below. A) periodontitis apicalis acuta B) periodontitis apicalis chronica C) all of them D) neither of them FOG - 54 - It can be symptom less for a long period of time FOG - 55 - The affected tooth is erupted from the alveolar socket FOG - 56 - Characteristic the severe pain occurring in intermittent attacks FOG - 57 - It can be attributed to the a consequence of dental caries

FOG - 54 - It can be symptom less for a long period of time - B) FOG - 55 - The affected tooth is erupted from the alveolar socket - A) FOG - 56 - Characteristic the severe pain occurring in intermittent attacks - D) FOG - 57 - It can be attributed to the a consequence of dental caries - C)

Join the given words marked with letters to the proper numbers below. A) caries media B) caries penetrans C) both of them D) neither of them FOG - 58 - It occurs only in the enamel . FOG - 59 - It affects the pulp chamber . FOG - 60 - It affects both the enamel and dentin FOG - 61 - It affects only the dentin .

FOG - 58 - It occurs only in the enamel . - D) FOG - 59 - It affects the pulp chamber . - B) FOG - 60 - It affects both the enamel and dentin - A) FOG - 61 - It affects only the dentin . - D)

Join the given words marked with letters to the proper numbers below. A) fixed dentures (restorations) B) partial removable dentures C) both of them D) neither of them FOG - 62 -The restoration occupies more space in the oral cavity than natural teeth FOG - 63 - In can be made without the damage of the natural teeth (the preparation of tooth structure) FOG - 64 - It is fixed on the remaining natural teeth . FOG - 65 - In can be made even in the case of total edentulousness . FOG - 66 - It can be cleaned easily . FOG - 67 - It can also be made in case of patient with a tendency to seizures

FOG - 62 - The restoration occupies more space in the oral cavity than natural teeth - B) FOG - 63 - In can be made without the damage of the natural teeth (the preparation of tooth structure) - B) FOG - 64 - It is fixed on the remaining natural teeth . - C) FOG - 65 - In can be made even in the case of total edentulousness . - D) FOG - 66 - It can be cleaned easily . - B) FOG - 67 - It can also be made in case of patient with a tendency to seizures - A)

Join the given words marked with letters to the proper numbers below. A) periapical abscess B) periodontal abscess C) both of them D) neither of them FOG - 68 - Increased tooth mobility. FOG - 69 - It can develop only from periodontal pockets FOG - 70 - It develops after the necrosis of the dental pulp

FOG - 68 - Increased tooth mobility - C) FOG - 69 - It can develop only from periodontal pockets - D) FOG - 70 - It develops after the necrosis of the dental pulp - A)

Join the given words marked with letters to the proper numbers below. A) It is characteristic of pregnancy gingivitis B) It is characteristic of acute ulcerative gingivitis C) both of them D) neither of them FOG - 71 - It can be cured by restoring proper oral hygiene. FOG - 72 - It theoretically can develop without the presence of dental plaque FOG - 73 - In its patomechanism the embolisation of the small blood vessels plays important role FOG - 74 - If it is not treated on time it will lead to irreversible tissue defect

FOG - 71 - It can be cured by restoring proper oral hygiene - C) FOG - 72 - It theoretically can develop without the presence of dental plaque - D) FOG - 73 - In its patomechanism the embolisation of the small blood vessels plays important role - B) FOG - 74 - If it is not treated on time it will lead to irreversible tissue defect - B)

Join the given words marked with letters to the proper numbers below. A) rhinogenic sinusitis B) odontogenic sinusitis C) both of them D) neither of them FOG - 75 - Usually both extends to the both side of the maxillary sinus FOG - 76 - It gives a positive radiological image FOG - 77 - Its characteristic syndrome is the severe facial swelling . FOG - 78 - Touching the area of the infraorbital nerves in the infraorbital foramen can provoke pain attacks

FOG - 75 - Usually both extends to the both side of the maxillary sinus - A) FOG - 76 - It gives a positive radiological image - C) FOG - 77 - Its characteristic syndrome is the severe facial swelling - D) FOG - 78 - Touching the area of the infraorbital nerves in the infraorbital foramen can provoke pain attacks - D)

Join the given words marked with letters to the proper numbers below. A) acute osteomyelitis B) chronic osteomyelitis C) both of them D) neither of them FOG - 79 - Characteristic radiological findings . FOG - 80 - Patient has a severe malaise . FOG - 81 - The shivering and septic fever are very common . FOG - 82 - Antibiotics should be administered.

FOG - 79 - Characteristic radiological findings - B) FOG - 80 - Patient has a severe malaise - A) FOG - 81 - The shivering and septic fever are very common - A) FOG - 82 - Antibiotics should be administered - C)

Join the given words marked with letters to the proper numbers below. A) gingival hyperplasia B) granuloma fissuratum C) both of them D) neither of them FOG - 83 - It belongs to the fibrous hyperplasia group of diseases FOG - 84 - It is the sign of systemic disease . FOG - 85 - Its frequent cause is the irritating effect of the ill fitting denture . FOG - 86 - It is frequently associated with hydantoin derivates used in the therapy of epilepsy

FOG - 83 - It belongs to the fibrous hyperplasia group of diseases - C) FOG - 84 - It is the sign of systemic disease - D) FOG - 85 - Its frequent cause is the irritating effect of the ill fitting denture - B) FOG - 86 - It is frequently associated with hydantoin derivates used in the therapy of epilepsy - A)

Join the given words marked with letters to the proper numbers below. A) dental implant B) natural root C) both of them D) neither of them FOG - 87 - It is anchored in the jawbones. FOG - 88 - It is an abutment of fixed restorations. FOG - 89 - it is an anchoring element. FOG - 90 - it is fixed by connective tissue.

FOG - 87 - It is anchored in the jawbones - C) FOG - 88 - It is an abutment of fixed restorations - C) FOG - 89 - it is an anchoring element - D) FOG - 90 - it is fixed by connective tissue - C)

Pair the lab marker with the most relevant number marked clinical status! A) PSA B) AFP C) β-hCG D) alkaline phosphatase E) all of them F) none of them URO - 46 - Elevated level can be a predictive sign of bone metastases in prostate cancer. URO - 47 - After the TUR surgery of bladder cancer it should be frequently checked. URO - 48 - The normal level could be higher than 6,5 ng/ml after the age of 80. URO - 49 - Elevated level can be found in the early stage prostate cancer too. URO - 50 - In the case of seminoma extending through the tunica albuginea elevated level can be found.

URO - 46 - Elevated level can be a predictive sign of bone metastases in prostate cancer. - D) URO - 47 - After the TUR surgery of bladder cancer it should be frequently checked. - F) URO - 48 - The normal level could be higher than 6,5 ng/ml after the age of 80. - A) URO - 49 - Elevated level can be found in the early stage prostate cancer too. - A) URO - 50 - In the case of seminoma extending through the tunica albuginea elevated level can be found. - F)

Pair the andrological abnormalities with the letter signed clinical references! A) erectile dysfunction B) infertility C) male menopause D) retrograde ejaculation URO - 51 - It can caused by varicocele URO - 52 - It appears as a frequent complication after TUR (transurethral resection) surgery. URO - 53 - It can caused by beta blockers. URO - 54 - One way of the treatment can be a vacuum constriction device

URO - 51 - It can caused by varicocele - B) URO - 52 - It appears as a frequent complication after TUR (transurethral resection) surgery. - D) URO - 53 - It can caused by beta blockers. - A) URO - 54 - One way of the treatment can be a vacuum constriction device - A)

Pair the the kidney stone(s) to the typical description! A) uric acid stone B) cystine kidney stone C) calcium oxalate kidney stone D) struvite kidney stone URO - 56 - A high, alkaline pH level increases the risk of developing these type of kidney stones. URO - 57 - the so called negative stone URO - 58 - Good solubility when treated with drugs. URO - 59 - To prevent the recurrence D-Penicillamin or Thiola can be used. URO - 60 - In case of presence, reduced meat diet is recommended

URO - 56 - A high, alkaline pH level increases the risk of developing these type of kidney stones. - D) URO - 57 - the so called negative stone - A) URO - 58 - Good solubility when treated with drugs. - A) URO - 59 - To prevent the recurrence D-Penicillamin or Thiola can be used. - B) URO - 60 - In case of presence, reduced meat diet is recommended - A)

Pair the diagnostic procedure name(s) to the relevant description! A) URS (ureterorenoscopy) B) PCNL (Percutaneous Nephrolithotomy) C) ESWL (Extracorporeal Shock Wave Lithotripsy) D) all of them E) none of them URO - 61 - non-endoscopic procedure URO - 62 - used in kidney stone treatment URO - 63 - Used in BPH treatment URO - 64 - The procedure is performed through the skin URO - 65 - It can be a diagnostic procedure too. URO - 66 - The procedure is only performed in intubation narcosis

URO - 61 - non-endoscopic procedure - C) URO - 62 - used in kidney stone treatment - D) URO - 63 - Used in BPH treatment - E) URO - 64 - The procedure is performed through the skin - B) URO - 65 - It can be a diagnostic procedure too. - A) URO - 66 - The procedure is only performed in intubation narcosis - E)

In which case is the hospital placement of the patient mandatory? A) Anthrax B) Leptospirosis C) Listeriosis D) Tularemia E) Q fever

A) Anthrax In order to provide an adequate treatment -adequate antibiotic therapy- for the severe, occasionally lethal disease (pulmonary anthrax), the patient should be taken to the hospital as soon as possible.

Reducing ozone concentration by 1% in the stratosphere will increase the incidence of skin cancer by: A) 1% B) 2% C) 3% D) 4% E) 5%

B) 2% A 1% reduction in ozone concentration, which plays an essential role in the uptake of harmful ultraviolet rays, increases the incidence of skin cancer by 2%. (As a result of this change, cataract incidence increases by 0.5%, too.)

Early hearing loss caused by noise is indicated on the audiogram A) a monaural recess (30 dB) seen at 8000 Hz B) binaural negative spike at 4000 Hz (30 dB) C) bilateral recess formation at any frequency D) unilateral recess formation at any frequency E) recess formation only at the 600 Hz (30 dB)

B) binaural negative spike at 4000 Hz (30 dB) An early hearing loss can only be detected by audiometrical investigations - initially in the range of 4000 to 8000 Hz. In Hungary a hearing loss at 4000 Hz that reaches/exceeds 30 dB (recess, negative spike) has to be reported as increased exposition casebut the hearing loss is irreversible in these cases.

Chromium and its compounds can cause: A) mesothelioma B) lung cancers C) myeloma D) lymphoma

B) lung cancers Inhalation of the hexavalent form of chromium could cause cancers, e.g. bronchial and nasopharyngeal cancers (which should be monitored periodically).

Infectious and suspected of being infectious patients detected in primary care should be reported to? A) the local institution of inpatient care B) the subregional public health institute competent at the regional location of the infection C) the subregional public health institute competenet at the infected patients'residence D) the National Public Health and Medical Office Service, National Center for Epidemiology

B) the subregional public health institute competent at the regional location of the infection

Which is the lowest frequency where a noise-induced permanent increase in the hearing threshold can be detected? A) 1000 Hz B) 2000 Hz C) 4000 Hz D) 8000 Hz

C) 4000 Hz An early hearing loss can only be detected by audiometrical investigations - initially in the range of 4000 to 8000 Hz. In Hungary a hearing loss at 4000 Hz that reaches/exceeds 30 dB (zajcsipke) has to be reported as increased exposition case.

DDT is: A) phenolic fungicide B) carbamate insecticide C) chlorinated hydrocarbon D) organic phosphate ester compound

C) chlorinated hydrocarbon DDT is the most commonly known representative of chlorinated hydrocarbons.

The chronic effect of which of the listed metals causes acrocyanosis? A) lead B) mercury C) cadmium D) arsenic E) chromium

D) arsenic Acrocyanosis is developed due to chronic arsenic exposure, its clinical manifestation is the "Blackfoot Disease", which is a lower limb circulatory disorder.

The survival time of the geohelminths' eggs in the soil is: A) 1-2 months B) 6-8 months C) 12-15 months D) 18-24 months E) 2-3 years

E) 2-3 years The resistance of geohelminths' eggs is significantly greater than the resistance of not geohelminth worms' eggs, thus they survive 2-3 years in the soil.

Limit of nitrite concentration A) 0.5 mg / l B) 1 mg / l C) 2 mg / l D) 3 mg / l E) 4 mg / l

A) 0.5 mg / l

The amount of the physiological daily sodium chloride requirement is approximately: A) 1-2 g B) 3-4 g C) 5-6 g D) 7-8 g E) 9-10 g

A) 1-2 g The physiological sodium chloride intake requirement is no more than 1-2 g per day which is exceeded even if we skip adding extra salt to our food. The WHO recommends that the daily salt intake should be kept below 5 g.

The mean age of patients with vCJB is A) 29-30 years B) 35-40 years C) 50-60 years D) over 60 years

A) 29-30 years The epidemiological feature of vCJD is that it occurs in young adults with an average age of approximately 30 years. This is one of the major differences between the new variant and the classic CJD; the latter one mainly affects the elderly generations.

Which of the two regions have the most increased risk of suicide-related deaths in Hungary? A) Northern Great Plain and Southern Great Plain B) Northern Great Plain and Northern Hungary C) Southern Great Plain and Northern Hungary D) Central Hungary and Northern Hungary E) Central Hungary and Northern Great Plain

A) Northern Great Plain and Southern Great Plain Traditional differencies can be seen related to suicide- mortality inside Hungary. In the Southern and Northern Great Plain the suicide related mortality of men aged 25-64 significantly (p<0.001; by 34% at lest) exceeded the Hungarian average suicide related mortality in the 1990s and 2000srespectively.

Which of the following procedures is not used for sterilization? A) UV light B) γ-radiation C) ethylene oxide D) autoclaving

A) UV light γ-radiation, ethylene oxide and autoclaving procedures are used for sterilization, while UV light is used in operating rooms, medical office waiting rooms and laboratories when the staff is not present. Timindimers are produced in the DNA of microorganisms during UV radiation, which penetration is weak so it can only be used for sterilizating surfaces and for disinfecting the indoor air.

Breast cancer's incidence is mainly based on various mutations in the BRCA1 gene: A) among Ashkenazi Jewish communities B) among Hungarian female population C) among Australian native population D) in the background of breast cancer in elderly population E) in the background of post-operative recidivation

A) among Ashkenazi Jewish communities Susceptibility to various neoplastic diseases at population level is reflected in the different incidence rates that could be seen among different communities (geographically determined, based on ethnicity, etc For example, high incidence rate of breast cancer can be seen among Ashkenazi Jewish communities related to various mutations of the BRCA1 gene.

Which is the organic solvent of which occupational exposure can be measured by the concentration of urine muconic acid? A) benzene B) toluene C) xylene D) ethylene glycol E) Propyl alcohol

A) benzene Determining the level of benzene exposure could be done by measuring the urine concentrations of one of its metabolites, muconic acid. Another possibility is the measurement of the dissolved product of benzene, the urinary phenol. In the case of toluene exposure, urinary hippuric acid or orthocresol concentrations can be measured, and the ortho hippuric acid content can be determined for xylene. The level of exposure to ethylene glycol and propyl alcohol can not be characterized by urinary metabolic measurement.

What is an inapparent infection? A) disease without clinical symptoms B) an infection with mild symptoms that can only be detected during an epidemic C) all infections that are cured without any harm/lesions D) any infection that does not generate laboratory changes/alterations

A) disease without clinical symptoms Inapparent infection occurs when the number of pathogens entering the organism is low or the microbe's pathogenicity is low. The immune system of the body can quickly overcome the infection, so it won't generate any clinical symptom.

In which diseases should the persons in close contact with the infected patient undergo a 7-day epidemiological surveillance? A) dysenteria B) campylobacteriosis C) typhus abdominalis D) Hepatitis A

A) dysenteria Epidemiological surveillance is used on persons who could be potential infectious sources due to their direct contact with the infected patient. Its' duration is equal to the maximum latency of the infectious disease. The duration of the epidemiological surveillance is 7 days for dysenteria, for campylobacteriosis 5 days, 21 days for typhus abdominalis and 30 days for hepatitis A infection.

Drinking water containing low level of magnesium will increase the incidence which of the following disease? A) heart attack B) cerebral stroke (dash) C) stomach cancer D) type 2 diabetes E) colon cancer

A) heart attack Based on epidemiological findings, out of the five diseases listed here only the positive relationship between myocardial infarct and the low level of magnesium in the drinking water is considered proven. (low magnesium consumption it increases the heart attack incidence)

The relative risk of premature mortality related to respiratory diseases) in Hungary compared to EU15 average A) is more than two times higher B) is more than three times higher C) is more than four times higher D) does not differ significantly () E) is 20% lower

A) is more than two times higher The relative risk of premature mortality related to respiratory diseases has been more than two times higher among both genders since 2005 in Hungary compared to EU15's mortality. (for men, 2.8, for women 2.3 in 2007)

Mortality burden of malignant tumors in Hungary: A) is the most important related to premature adult mortality B) takes the second place after cardiovascular diseases among premature adult deaths C) is the most important related to old age mortality D) has grown in the last decade E) is not remarkabla in premature mortality among women

A) is the most important related to premature adult mortality The mortality burden of malignant tumors is the most notable related to premature mortality (especially among women) and the second in total mortality (population age 0 to X) in Hungary. Cardiovascular diseases are the leading cause of death in old age mortality. The mortality of malignant tumors has decreased over the last decade.

Which statement related to odds-ratio is false? A) it is the probability ratio of exposure among cases and controls B) if the odds-ratio is exactly 1 then there is no relationship between the risk factors and the disease C) it can be calculated from contingency tables D) it can be calculated from case-control studies

A) it is the probability ratio of exposure among cases and controls Calculation of odds ratio (OR) among case and control groups is based on the probability of exposition derived from the 2x2 contingency tables. The odds of exposition in the case group: a/c ; the odds of exposition in the control group: b/d, so the ratio of the two is the odds ratio. The odds ratio is 1 if there is no relationship between the potential risk factor and the disease.

Which phase of the public health cycle is based on epidemiological analyzes? A) situation analysis B) determining the intervention options C) planning interventions D) preparing a strategic plan E) implementation of health promotion programs

A) situation analysis The first phase of the public health cycle (the phase which is repeated regularly during the active cycle) is an epidemiological analysis of the populations' health status (and the factors influencing it), which is attached to strict professional aspects, and the evaluation of which is called situation analysis. In this phase, it is often necessary to characterize the temporal changes of a given health indicator or a health determinant as well as to determine whether a change took place with respect to a reference value and if it did, to what extent and into which direction

The mechanism of action of carbamate pesticides A) the inhibition of acetylcholinesterase B) Inhibition of glutathione peroxidase C) inhibition of C-SH group enzymes D) inhibition of ATPase

A) the inhibition of acetylcholinesterase Carbamates, like organic phosphates, are acetylcholinesterase inhibitors, but their activity is reversible and short acting, so the enzyme reactivation occurs spontaneously.

Which of the following compounds has a human teratogenic effect? A) toluene B) cobalt C) acrylamide D) formaldehyde E) methanol

A) toluene Toluene has a teratogenic effect on humans; due to this special feauture, pregnant women are not allowed to be employed in case of toluene exposure.

The most common causes of premature deaths related to fatal accidents are: A) vehicle traffic accidents B) falls C) poisoning D) burns E) injuries related to firearms

A) vehicle traffic accidents Approximately 40% of accidents causing deaths under 65 were caused by vehicle traffic accidents - especially motor vehicle accidents (men: 33.7%, women: 33.0%) - and every sixth death was caused by accidental falls in 2008. Above the age of 65 years, however, the weight of the two accidental death causes changes: accidental fall is the cause in approx.. two-thirds (59%) of accidental deaths in elderly men. The corresponding number among elderly women is 70.5%.

Which monomer causes a sclerodermic change affecting the fingers of chemical plant workers? A) vinyl chloride B) acrylamide C) acrylonitrile D) vinylbenzene E) phenylethylene

A) vinyl chloride Vinyl chloride is a colorless, sweet smelling gas which causes exposition during the manufacturing and processing of PVC. Disorders of the nervous system, Raynaud's syndrome, thrombocytopenia, fibrotic liver damage, pulmonary fibrosis, acro-osteolysis, and sclerodermoid lesions have been reported among workers exposed to long (about 15 years) exposition of moderate doses of Vinly chloride.

Which formula can be used to calculate the excess mortality rate? (expected number of deaths: E, observed number of deaths: O) A) (O - E) / O × 100 B) (O - E) / E × 100 C) (E - O) / O × 100 D) (E - O) / E × 100 E) none of them are true

B) (O - E) / E × 100 The excess mortality rate is the difference between observed and expected number of deaths divided by the number of expected deaths. It can be converted to percentage form using the formula: (O - E )/E x 100; which describes how the percentage of deaths of the given population exceeds that of chosen for comparison base.

The weight of healthcare in determining the health status in developed countries is approximately: A) 5-10% B) 10-15% C) 15-20% D) 20-25% E) 25-50%

B) 10-15% The weight of the different determinants is not equal in ensuring health. Although the contribution of the individual factors is debated (and may indeed differ in different communities), it is generally accepted that the genetic factors and the quality of healthcare determine the state of health in approximately 15-30% and 10-15%, respectively, while the lifestyle factors that are closely related to socio-economic status and education are responsible for the remaining 55-75% in determining health.

When was the eradication of smallpox declared by the WHO? A) 1967 B) 1979 C) 1993 D) 2003 E) only Europe is smallpox-free

B) 1979 In 1979, as a result of the smallpox eradication program organized and managed by the WHO, the organization declared that smallpox had been eradicated on a worldwide level, which served not only the overall acknowledgement of public health activity, but also boosted the development of public health.

The permissible cumulative annual equivalent dose of workers exposed to ionizing radiation is: A) 5-10 mSv / year B) 20 mSv / year C) 50 mSv / year D) 100 mSv / year E) 500 mSv / year

B) 20 mSv / year Occupational exposure level should be regulated so that the effective annual dose should not o-exceed 20 mSv (averaged over five consecutive years).

What is the minimum dose responsible for causing irreversible blindness in acute methanol intoxication? A) 1-3 g B) 8-10 g C) 20-22 g D) 25-30 g E) 40-50g

B) 8-10 g Consumption of 8 to 10 g of methyl alcohol causes irreversible blindness due to its direct effect on the optic nerve.

Which organization developed the Public Health Code of Ethics describing the valid ethical principles of public health activity? A) World Health Organization (WHO) B) American Public Health Association C) United Nations Organization D) European Public Health Association E) An ad hoc committee established by the WHO for this purpose

B) American Public Health Association The American Public Health Association (APHA) defined the Public Health Code of Ethics in 12 points in 2001

Whose activity can be related to the establishment of occupational health as a subdiscipline of public health? A) Max Pettenkofer B) Bernardino Ramazzini C) Fodor József D) Robert Koch E) Rachel Carlson

B) Bernardino Ramazzini The first professional book related to occupational health, Ramazzini's work entitled " De morbis artificum diatribe " was published in 1700, and it has been considered to be a milestone in establishing the subdisciplines of public health

What does "phenotype prevention" mean? A) Interrupting pregnancy (in the case of parental consent) to prevent giving birth to genetically impaired fetuses B) Preventing the onset of a disease in individuals with increased risk genotype mainly by abolishing harmful environmental exposures C) genetic counseling for couples with increased genotype risk who want to have children D) gene therapy interventions among individuals to correct defective genes among individuals with increased genotype risk E) estimation of population frequency of susceptible alleles related to diseases

B) Preventing the onset of a disease in individuals with increased risk genotype mainly by abolishing harmful environmental exposures Detection of genetic predisposition can not be considered as a primary preventive intervention because the risk factor can not be excluded. It can not be considered as secondary preventive intervention either because it is not intended to help early detection of the disease but to detect the susceptibility. Detection of genetic predisposition of chronic non-communicable diseases is called phenotype prevention (as suggested and determined by Juengst in 1995) which falls between primary and secondary prevention. Phenotypic prevention's main aim is to prevent the disease's onset among endangered genotype individuals (high risk groups).

What is the most likely disease in the case of "food poisoning" when the symptoms appear in a 6-48 hours period after the consumption of homemade ham and mayonnaise French salad, and the symptoms are: abdominal pain, diarrhoea, vomiting, fever? A) Staphylococcal food poisoning B) Salmonellosis C) Botulism D) Yersiniosis E) Trichinellosis

B) Salmonellosis The difference between the diseases with similar clinical symptoms is in their latency period. In a staphylococcal foodborne disease, the incubation period is usually between 2 to 6 hours, while 6 to 48 hours for salmonellosis and 12 to 36 hours for botulism. The latency period of Yersiniosis is 3-7 days. The incubation period is usually 7 to 10 days in trichinellosis.

In which disease is not the swine the infectious source? (Which disease is not spread by the swine?) A) Brucellosis B) Tularemia C) Listeriosis D) Taeniasis E) Trichinellosis

B) Tularemia Swine is not infectious source of the Tularemia; its reservoirs could be certain rodents (mice, rats), cats, sheep, etc. Reservoirs as well as infectious source of Brucellosis (Brucella suis), listeriosis, Taenia (Taenia solium) and trichinellosis are the swine.

Utilization of the findings of genomic research in public health: A) is free of charge and widely available B) is limited C) genetic screening can only be used after payment D) a complete lack of legislation is missing to ensure their implementation E) screening tests which could prove genetic predisposition to cancer are unreliable

B) is limited The application of knowledge related to genetic / genomic science in public health is still limited. The obstacles hindering utilization are as follows. i) Genetic epidemiological studies were rarely conducted in potentially affected populations. ii) The applied methods are usually cost-intensive. iii) Preventive interventions are available only in a small minority of the identified risk conditions and the acceptance of these interventions for the individuals is also questionable. iv) Human resources are not granted either for genetic-based screening in the field of public health or for dissemination of goals and results. v) . The legal background for the regulation of genomic researches is restricted.

At which heavy metal exposure are people with hyperthyroidism not allowed to be employed/to work? A) arsenic B) mercury C) lead D) cadmium E) chrome

B) mercury People with hyperthyroidism should not be exposed to mercury. Non-serious thyroid gland disease does not exclude the employability with/of other heavy metals.

Which indicator is not part of the vital statistics based on civil registries? A) fertility B) migration C) mortality D) natality E) reproduction

B) migration Population dynamics - ongoing population processes - are characterized by vital events and migration over a specified period (usually a one year span). The basic elements of natural population dynamics are natality/birth rate, fertility, mortality and reproduction. Civil registries (vital statistics) are the main information source for vital events/population dynamics

Which disease does not require that people who are in direct contact with the infected patient be treated with chemoprophylaxis? A) pertussis B) scarlatina C) diphteria D) meningitis epidemica E) H. influenzae meningitis

B) scarlatina Due to the fact that the cause of scarlet fever (Streptococcus pyogenes) occurs in 5-10% of the population in general and in 20-30% of the population during an epidemic, the possibility of an infection is not restricted only to the patient, but may happen from others, too. Consequently, chemoprophylaxis of the contacts has no significant preventive effects and does not need to be used. In other diseases, as circulating bacteria are limited in the population, the chemoprophylaxis of the contacts - by blocking the development of the infection originating from the patient- is effectively preventive and should be used.

The main aim of the North Karelia Project was: A) to reduce cancer-related premature mortality B) to reduce cardiovascular premature mortality C) to reduce the mortality of mental illnesses D) to improve the care of hypertensive patients E) the early detection of cardiovascular diseases

B) to reduce cardiovascular premature mortality The North Karelia Project was implemented by the Finnish Government in the early 1970s which main aim was to reduce the mortality rate of cardiovascular diseases. The project mainly focused on healthy eating and the reduction of smoking rate which resulted in a reduction of cancer-related mortality. Premature cardiovascular mortality decreased by 60%, and premature cancer-related mortality decreased by 40% within 25 years after implementing the Project.

In the case of acute poisoning which of the substances below can cause - after a few weeks latency - a reversible paralysis of the lower limbs? A) formaldehyde B) tri-ortho-cresyl phosphate C) acrylonitrile D) acrylamide E) vinyl chloride

B) tri-ortho-cresyl phosphate In the case of acute tri-ortho-cresyl phosphate intoxication after 2-4 weeks of the first symptoms (vomiting, diarrhea) paralysis of the lower limbs may occur that is usually reversible. The other compounds will not cause acute damage to the nerves of the lower limbs.

Which of the following infections should be reported urgently? A) dysenteria B) typhus abdominalis C) campylobacteriosis D) amoebiasis

B) typhus abdominalis Highly infectious diseases, e.g. typhus abdominalis, cholera, poliomyelitis, lyssa, pestis, etc. have to be reported immediately by phone or by e-mail to the competent county public health service and to the National Public Health and Medical Office Service, National Center for Epidemiology.

The ICD-10 classification... A) has been valid since the 18th century. B) uses an alphanumeric code system. C) varies country by country. D) the individual classes show overlaps. E) quantifies mortality data.

B) uses an alphanumeric code system. ICD-10 classifies diseases into standardized classes based on international criteria which were implemented in Hungary on January 1, 1996. The aim of the standardized unification is to ensure that the morbidity and mortality data derived from different sources and for different diseases be comparable. It uses an alphanumeric code system.

What is the effective temperature? A) various combinations of air temperature and humidity which elicit the same heat sensation B) various combinations of air temperature, humidity, and air movement which elicit the same heat sensation C) combinations of air temperature and radiation that elicit the same heat sensation D) combinations of air temperature and air flow that elicit the same heat sensation E) none of them

B) various combinations of air temperature, humidity, and air movement which elicit the same heat sensation Effective temperature is based on the relative humidity of the air as well as on the airflow rate, while the corrected effective temperature takes into account the heat radiation also. The effective temperature can be determined by a nomogram.

Is the child (infant?) mortality differerent among the highest and lowest socioeconomic groups, respectively? A) Yes, its the highest among infants with the highest status B) yes, its the highest among infants with the lowest status C) No D) No general conclusion could be drawn, it varies from country to country.

B) yes, its the highest among infants with the lowest status The socio-economic situation is a factor that has a significant impact on the health status and on its various indicators, including infant mortality. This factor tends to be more favorable in children with better socioeconomic status in all countries (resulting in smaller infant mortality rate).

When is it recommended to remove the worker spraying with organich phosphoric acid ester from her/his job? A) When the worker's red blood cell AChE activity has decreased by 10% compared to the exposure-free self-activity. B) When the worker's plasma AChE activity decreased by 50% compared to its own basic activity. C) When the worker's red blood cell AChE activity has decreased by more than 50% compared to own baseline activity. D) If the worker's red blood cell AChE activity decreased by 20-50% from own basic activity. E) Urgent action should be taken even in the case of the smallest decrease of the red blood cell AChE activity

C) When the worker's red blood cell AChE activity has decreased by more than 50% compared to own baseline activity. If the individual's red blood cell cholinesterase activity is less than 50% compared to its own basic activity, then the person must be (permanently) removed from the position and this is also recommended when the enzyme activitiy decreased by 25%

What vaccine should be given to newborns of HbsAg-positive mothers? A) passive immunization against hepatitis B B) active immunization against hepatitis B C) both D) none of them

C) both Newborns of HbsAg positive mothers should receive both active and passive immunization against hepatitis B. Protective antigen of HBV is HbsAg. HBs antibody provides protection against it. HBV can not be produced without hosting human body (in vitro). The protective antigen was produced by plasmapheresis from the plasma of healthy carriers in the past. Nowadays, it is produced using standard genetic engineering techniques, the virus's HbsAg-encoding S gene has been implanted into Saccharomyces cerevisiae and can be produced under large-scale circumstances. Component of human origin is not present in the vaccine. Protective antibodies are produced from sera with high titers for passive immunization. Newborns should be given hyperimmun γ globulin within 12 hours as well as the iniation of active immunization should start. The latter must be followed by revaccination at month 1 and and 6.

For which infection must be the patient quarantined at an inpatient hospital unit? A) typhus abdominalis B) cholera C) both of them D) none of them

C) both of them Limiting and separating the movement of the infectious source (infected patient) can prevent the infectious pathogens from spreading. Separation could be done in an inpatient hospital unit's quarantine room especially when it is indicated by the patient's health status, too.

In which sterilization method are carcinogenic compounds used? A) dry heat sterilization B) autoclaving C) chemical sterilization D) plasma sterilization

C) chemical sterilization Liquids (formaldehyde) and gas (ethylene oxide) are used for chemical sterilization of heat-sensitive materials to destroy microorganisms. Their application allows lowering the sterilization temperature.but both compounds are known to have carcinogenic effects. During dry heat sterilization dry air with temperature between 160 and 200 oC circulates in a sealed space in which microorganisms will be inactivated, then destroyed due to high temperature. Autoclaves operate with saturated, pressurized water vapor. The microorganisms are destroyed by the heat energy released from the vapor condensed on the surface of the deposited materials. During plasma steriliztaion, the sterilization will be done in the hydrogen peroxide plasma which will split into water and oxygen

What kind of radiation cause the cataracts of smeltery workers? A) microwave radiation B) visible light with high brightness C) infrared radiation D) stroboscopic effects

C) infrared radiation Infrared radiation spans between the visible and the radiofrequency range of the electromagnetic spectrum. Besides the sunlight it is produced by processes and activities that use the thermal energy of infrared radiation.These activities include heating and dewatering processes, welding, glass manufacturing, drying and burning of coatings for public goods. Waves with wavelengths shorter than 2000 nm can damage the cornea, iris and lens.

Industry or activity with a high risk of causing cancer is A) rag-picking B) pepper (paprika) splitting C) iron and steel production D) electrical installation

C) iron and steel production High cancer-risk industries are those industrial areas, where most of the people employed are exposed to one or more carcinogenic materials and within this population of workers the incidence of tumorous diseases exceeds the expected incidence of cancer in the general population.

Which statement is true related to relative risk? A) it compares the frequency of exposure among patients to the frequency of exposure among healthy subjects B) it belongs to the category of difference indicators C) it is a measure of the incidence observed among the exposed subjects to the incidence of unexposed population D) all of the statements are true E) none of the statements is true

C) it is a measure of the incidence observed among the exposed subjects to the incidence of unexposed population Risk indicators are indicators which characterize the relationship between a givendisease and the incidence of an assumed risk factor. Their estimation is based on comparison of the incidences of two (or more) groups from the population, in which the groups' risk factor's exposure levels are different. The denominator of the comparison usually is the incidence of the disease among non-exposed population. Risk indicators may be either ratio indicators (relative risk) or difference indicators (attributable risk).Relative Risk (RR) is the incidence ratio among exposed and unexposed persons (RR=events when exposed/events when not exposed).

Which statement related to poliomyelitis is not true? A) Reportable disease B) must be reported by telephone, by fax C) laboratory test is not obligatory D) Feces sample, irrigation fluid, liquor and blood samples must be sent to the Virology Department of National Public Health and Medical Officer Service

C) laboratory test is not obligatory Poliomyelitis anterior acute disease is a reportable disease, which must be reported by telephone and fax to the National Public Health and Medical Officer Service and the National Center of Epidemiology. Laboratory tests are mandatory. Feces sample, irrigation fluid, 1-2 liquor samples and native blood must be sent to the Virology Department of National Public Health and Medical Officer Service which must be followed by the resending of new blood samples after 3 weeks for laboratory retests.

Which individuals (related to their gender and family status) exhibit more complicated manifestation of reactive depression? A) married men B) divorced men C) married women D) divorced women E) promiscuous individuals regardless of their gender

C) married women Depression of married women is more severe than that of women who are not married based on the findings of cross-sectional studies.The opposite relationship has been observed in men, for example, married men are less likely to have depression and even if the depression has developed it shows milder symptoms.

Which screening type implies newborn screening? A) mandatory screening B) targeted screening C) mass screening D) screening of groups with increased risks E) diagnostic screening

C) mass screening The term "mass screening" emphasizes that the given screening test covers the entire potentially affected population. Typical examples for mass screening are, for instance, newborn screening (e.g. for phenylketonuria), the screening of the fetus for detecting developmental disorders (e.g. by ultrasound examination, etc.).

Which heavy metals chronic exposure causes allergies and contact eczema? A) manganese B) silver C) nickel D) iron

C) nickel Nickel is used in electroplating and steel alloys, in batteries (Ni-Cd and Ni-hybrid) and as a chemical catalyst. Many goods are made of nickel-containing metals. Contact with the skin often causes contact dermatitis, eczema or asthma.

The first symptom of chronic mercury poisoning is: A) paralysis of radial nerve B) paralysis of ulnar nerve C) small wave tremor D) visual impairment

C) small wave tremor Chronic exposure of mercury is most commonly caused by mercury vapor. Symptoms of the peripheral nervous system are dominating in chronic exposure of mercury, the so-called small- wave tremor mercurialis is especially characteristic, which starts in the facial and hand muscles (writing test) but eventually affects the whole body.

Which compound is not involved in the effects caused by the Los Angeles-type smog? A) aldehydes B) ozone C) sulfur dioxide D) hydrogen peroxide E) peroxyacetyl nitrate

C) sulfur dioxide Sulfur dioxide is not involved in the effects of Los Angeles-type smog because it has no oxidative effect. However, all other compounds have such effect and, therefore, they actively participate in the induction of the symptoms of oxidative-type smog.

Relative risk of lung cancer among smokers compared to non-smokers is approximately: A) two times increased risk B) five times increased risk C) ten times increased risk D) hundred times increased risk E) 90% higher

C) ten times increased risk The carcinogenic role of smoking is generally accepted based on the meta-analysis of epidemiological investigations. The relative risk of regular smokers for lung cancer is ten times higher compared to non smokers.

It is forbidden to give one of the followings in the case of carbamate intoxication: A) atropine B) diazepam C) toxogonin D) other parasympathetic compounds E) too much liquid

C) toxogonin Carbamates are insecticides classified as toxic or highly toxic compounds based on their per os LD50 values. These organic phosphates are cholinesterase inhibitors, but their effect is reversible and lasts for short time so the enzyme reactivation occurs spontaneously. In case of acute poisoning, administration of reactivators (toxogonin) is prohibited (its intake will worsening the symptoms);otherwise the treatment is the same as described for organic phosphates.

Which one of the following in not included in age-related mandatory vaccination in Hungary? A) vaccination against mumps B) vaccination against rubella C) vaccination against smallpox D) vaccination against tetanus

C) vaccination against smallpox Vaccination against mumps, rubella and tetanus belong to mandatory vaccination according to the vaccination schedule which are received in the form of combined vaccines. Vaccination against smallpox was mandatory up to 1980 (eradication of smallpox).

High dose chronic or "long-term" exposure of which monomer is responsible for the liver hemangiosarcoma in chemical factory workers ? A) acrylamide B) vinylbenzene C) vinyl chloride D) phenylethylene E) acrylonitrile

C) vinyl chloride During the manufacturing and processing of PVC, the vinyl chloride which is a colorless, sweet smelling gas could be exposed to workers. High doses of chronic or long term exposition may cause liver hemangiosarcoma.

In which organic solvent exposure are dermatological patients not allowed to work? A) benzene B) toluene C) xylene D) carbon tetrachloride E) gasoline

C) xylene In the case of work related to xylene exposure - in addition to pregnant women, breastfeeding mothers, adolescents, alcoholics, people with hemopoietic and renal disease - dermatological patients are not allowed to be employed.

The pain threshold related to sound pressure level is: A) 70 dB B) 80 dB C) 100 dB D) 130 dB

D) 130 dB During the measurement of sound pressure the hearing thereshold is taken as a reference and the pressure of the various sounds is compared to this value. The result is the logarithmic decibel scale that gives the sound pressure level.The hearing range spans from 0 dB (this corresponds to the hearing threshold of 20 μPa) to 120-140 dB (this is 2 × 107 μPa). Sound pressure greater than the pain threshold (approx.. 130 dB) causes only pain.

The nitrate concentration limit of drinking water for infants to drink or their food to be prepared with is: A) 10 mg / l B) 20 mg / l C) 30 mg / l D) 40 mg / l E) 50 mg / l

D) 40 mg / l 40 mg / l is the limit for the nitrate concentration in drinking water, above which the water is prohibited for babies to drink or to their food to be preapered with.

The prevalence of hypertension among men aged 65-74 in Hungary is approximately: A) 13-20% B) 26-40% C) 40-58% D) 48-72% E) 79-90%

D) 48-72% The prevalence of hypertension among men aged 35-44 is 13-20% and 26-40% among men aged 45-54. It was 40-58% among men aged 55-64 and 48-72% among men aged 65-74 based on the data derived from General Practitioners Morbidity Sentinel Station Program (GPMSSP).

How many days before the onset of the symptoms can infectivity persist in people infected with Lyssa? A) 1-2 days B) 3-4 days C) 5-6 days D) 7-10 days E) 12-15 days

D) 7-10 days The amount of virus required for infectivity is presented in the excretion of the infected person 7-10 days before the onset of symptoms, so those being in contact may be infected with an asymptomatic person.

Which one of the following is not a geohelmint? A) Strongyloides stercoralis B) Trichuris trichiura C) Ascaris lumbricoides D) Taenia solium

D) Taenia solium Strongyloides stercoralis, Trichuris trichiura and Ascaris lumbricoides are geohelminths, while Taenia solium is not. Infection with Taenia solium and Taenia saginata occur through the consumption of inadequately cooked, semi-cooked or raw meat. The intermediate hosts for Taenia solium are swine and for Taenia saginata are cattles. The eggs could enter the body of the animals orally with infected human feces or with infected fodder They can get everywhere in the body via blood circulation, but they usually settle in the mucles. (cystycercus).

What does "dose toxica minima" mean? A) The smallest amount of substance that is still responsible for eliciting the specific effects characteristic of the substance B) The highest amount of substance to which the organism will not respond with lesions/abnormal alterations C) Minimum amount of substance to which the body responds with non-typical changes that are not characteristic of the given substance D) The smallest amount of substance that may have toxic effects E) The smallest amount of substance to which the body responds with an effect typical of the substance

D) The smallest amount of substance that may have toxic effects The minimum toxic dose (DT) is the smallest amount of a substance that already causes toxic effects, regardless of whether these effects reflect the general toxicity or are symptoms characteristic of the substance.

The true statement on mortality of Hungarian men's aged 45-64 between 1991 and 1995 was: A) higher than in 1981-1985 B) higher than in 2001-2005 C) higher than in 1920-1921 D) all of them E) none of them

D) all of them Analyzing the premature mortality of the Hungarian population stratified by age, we can point to the fact that the mortality of men aged 45-64 between 1991 and 1995 was worse compared to 1920-21. By 2006 this parameter decreased below the value observed in 1930-31.

Which of the following pesticides are persistent in surface waters? A) organic phosphate esters B) pyrethroids C) carbamate D) chlorinated hydrocarbon E) dithiocarbamates

D) chlorinated hydrocarbon Among the listed pesticides, chlorinated hydrocarbons (eg DDT) are persistent in surface waters, they decompose only over a long time. Compounds of the other four groups are decomposed in an aqueous environment in a short time. Organic phosphate esters are hydrolyzed in the presence of a water molecule, which results in the losing of their toxic effect.

Which is not a component of metabolic syndrome? A) hypertension B) increased fasting blood glucose level C) elevated triglyceride level D) elevated HDL-C value E) visceral obesity

D) elevated HDL-C value The main components of metabolic syndrome are the hypertension, increased fasting blood glucose level, elevated triglyceride value and visceral obesity based on both the ATPIII and the IDF guidelines. Only decreased HDL-C level is considered to be abnormal.

To evaluate the effectiveness of public health programs, the following is essential: A) the knowledge of mortality B) the knowledge of morbidity C) the screening of the population D) establishing and operating health monitoring systems E) the exact knowledge of the impact of the interventions

D) establishing and operating health monitoring systems Evaluating the effectiveness of public health programs is not possible without establishing and managing a health monitoring system. Establishing and managing registries (utilizing modern information technology) of communicable and non-communicable diseases (including circulatory, carcinomatosus, digestive, respiratory, neurological, mental and musculoskeletal diseases) as well as registries of accidents are necessary, but not sufficient. This activity should be accompanied by monitoring the factors influencing health status (lifestyle, social, economic and environmental factors).

Which compounds dissolve in the body during the toxic detoxification process? A) toluene B) freon C) carbon tetrachloride D) methanol

D) methanol The process of toxic detoxification, namely a metabolism where the the degradation products are more toxic than the original molecule, is only observed related to methanol. Formaldehyde and formic acid, formed from methyl alcohol, can cause more severe symptoms (eg blindness) than the methanol itself. For the other four compounds (toluene, freon, carbon tetrachloride, benzene) the degradation products are less toxic than the original molecule.

Suicide mortality was characterized by the following over the past three decades in Hungary: A) it has drastically increased B) a temporary declining trend was followed by stagnation since the Millennium C) more women are affected than men D) mortality rates decreased significantly for both gender E) the highest suicide mortality for both gender was observed in the Northern Hungary Region

D) mortality rates decreased significantly for both gender A decreasing trend was observed in recent decades related to suicide mortality. Mortality of men aged 25-64 in 2008 was nearly 50% lower than in 1980, while mortality of women decreased by one third. Highest suicide rate can be seen in Northern Great Plain and Southern Great Plain.

Which attributable risk indicator depends on the prevalence of exposure among the population? A) incidence of exposed B) attributable risk C) attributable risk fraction D) population attributable fraction E) all of the above

D) population attributable fraction The attributable (AR) risk is the excess risk fraction of the exposured against non-exposured. AR = incidence among exposed - incidence among non-exposed. The attributable risk fraction (ARF) is the reduction in incidence that would be ovbserved in case of the exposition would cease in the entire population (ARF = incidence of the exposed - incidence of the non-exposed / incidence of the exposed). Attibutable risk indicators only indicate the excess risk caused by the risk factor itself and do not rely on prevalence (rate of exposure) of the risk factors in a given population. However, population attributable risk takes into account this feature: Population attributable risk = incidence in the entire population - incidence of non exposured

Which animal (s) is/are the infectious source (s) of Choriomeningitis lymphocytica? A) ruminants B) swine C) dog D) rodents

D) rodents LCM virus reservoirs could be rodents (domestic and field mice, hamsters, guinea pigs, etc.) as well as other animals (e.g. some monkey species) not mentioned here. Humans can be infected due to the consumption of food contaminated with urine and / or feces of these animals or due to inhaling dust containing the pathogens.

There is no active vaccination commercially available in Hungary against the following disease: A) hepatitis B B) influenza C) typhus abdominalis D) salmonellosis E) yellow fewer

D) salmonellosis Prevention of salmonellosis is not related to vaccination but it is a hygienic task. This task includes health monitoring of animals which are going to be processed that is the veterinarian's job. Regulations of production, storage and transport of food are food hygiene tasks. Providing drinking water, proper sewage treatment and disposal, fly and pest control belong to environmental and community health. Personal hygiene is also quite important related to preventive services. Strict observance of hygienic rules is important during food preparation, storage, transportation and catering to prevent salmonellosis.

The decibel means A) the strength of the noise/sound. B) the sound frequency. C) the sound pressure. D) the level of the sound pressure.

D) the level of the sound pressure. During the measurement of sound pressure the hearing thereshold is taken as a reference and the pressure of the various sounds is compared to this value. The result is the logarithmic decibel scale that gives the sound pressure level.

Which is the most vulnerable population group related to osteoporosis? A) children and adolescents B) elderly women and men C) workers exposed to permanent vibration exposure D) women at postmenopausal period E) athletes with increased physical activity

D) women at postmenopausal period Decrease in bone mass is observed as the age progresses in both men and women, but the most significant decrease in bone mass can be seen among postmenopausal women. Osteoporosis is strongly related to the decrease of estrogen production

How long must the food samples be stored in kitchens providing public catering services? A) 6 hours B) 12 hours C) 24 hours D) 48 hours E) 72 hours

E) 72 hours Food samples should be kept for 72 hours. Detecting the source of food-related infections and poisonings can be done by the investigation of food samples. Kitchens providing public catering - over 29 servings per day - have to store a 100g food sample of each meal in a clean, microbiologically and chemically acceptable container or packing bag and in a refrigerator for 72 hours. Contents, date of sampling and the name of the person performing the sampling must be attached to the sample. If bacteria was found in the patients' feces and the bacteria's antigenic structure could be found in the food sample, as well as the infection was proved by laboratory tests, then the infection developed because of the food consumption. If the symptoms of the suspected food consumers are similar and their latency time is almost the same then the relationship between food consumption and the bacteria is proved epidemiologically.

Which statement related to salmonellosis is not true? A) It is a reportable disease B) It must be reported on via a "food contamination, food poisoing notification" sheet C) The regional public health institute should be notified by telephone D) Laboratory tests are mandatory E) For the correct diagnosis blood sample should be sent to the regional laboratory before treatment

E) For the correct diagnosis blood sample should be sent to the regional laboratory before treatment Salmonellosis is a reportable disease. If the infection has been transmitted by food, apart from the usual custom notification, the onset and the suspicion of infection must be urgently reported (by telephone, fax) to the relevant sub-regional public health institute, as well as in written form by filling the "food contamination, food poisoning notification" sheet. The disease is considered as group poisioning if the number of patients is between 5-30, if more than 30 are affected, then poisoning refers to a mass poisoning. In the latter case, the regional public health institution reports to the National Public Health and Medical Officer Service which informs the Medical Office Service and the National institute of Pharmacy and Nutrition. Patients are prohibited to work with food and beverages as well as in childcare facilities until the emission of pathogens is ceased. Laboratory tests of the feces are mandatory for diagnostic purposes while release examinations have to be carry out for those whose occupation is related to a child community of 0-3 years old, or work in a "dangerous position" or give breast milk.

Which disease could cause teratogenic changes in the fetus during pregnancy? A) Ornithosis B) Tularemia C) Leptospirosis D) Q fever E) Toxoplasmosis

E) Toxoplasmosis Due to transplacental infection in pregnant women carrying toxoplasma gondi the fetus is prone to severe teratogenic changes which could affect the central nervous system. The pathogens of the other listed diseases do not pass the placenta, so they have no teratogenic effect.

Which factor does not participate in the biological activity of the soil (geocycle)? A) the filtering effect of soil particles B) soil temperature C) absorption D) biological membrane E) UV radiation

E) UV radiation Physical (pelletization, moisture, temperature), physicochemical (absorption, precipitation) and biological (degradative organisms, biological membrane) factors participate in the biological activity of the soil. As the UV radiation does not penetrate the soil, thus it does not participate in the geocycling processes.

Which compound is not a proven carcinogen in humans? A) aflatoxins B) arsenic C) benzene D) cyclophosphamide E) cyclosporine

E) cyclosporine Aflatoxin, arsenic, benzene, cyclophosphamide are proven human carcinogens.

Which acute high-dose pesticide poisoning - with other severe symptoms - causes fever above 40° C? A) carbamate B) dithiocarbamate C) phenoxycarboxylic acid derivative D) Pyrethroid E) dinitro-o-cresol

E) dinitro-o-cresol High doses of dinitro-o-cresol may cause severe acute symptoms - tachycardia, respiratory disorders, spasms, lung oedema, accompanied by high fever above 40 ° C. Fever caused by acute dinitro-o-cresol intoxication cannot be cured with drugs effectively, but direct cooling of the patient (eg cold water bath) can reduce body temperature.

Which of the following is considered as late neonatal death? A) death on the 5th day of life B) death in the 5th week of life C) death in the 5th month of life D) all of them E) none of them

E) none of them During the first year after birth, distinct periods can be distinguished related to infant mortality based on age.These are the following: early infant (neonatal) death: 0-27 days (usually divided into 0-6 days - early neonatal, 7-27 days - late neonatal), late infant (postneonatal) death: 28th day - 1 year (the first birthday does not belong to the infant age).

Standardized mortality ratio (SMR) can be calculated by indirect standardization using the following formula: A) SMR = (observed number of deaths - expected number of deaths) × 100 B) SMR = (observed number of deaths - expected number of deaths) /observed number of deaths× 100 C) SMR = (observed number of deaths - expected number of deaths) / expected number of deaths × 100 D) SMR = observed number of deaths - expected number of deaths E) none of them

E) none of them Standardized mortality ratio (SMR, usually expressed as percentage) related to the given population calculated by indirect standardization is the ratio of the observed number of deaths and the expected number of deaths..(SMR = observed number of deaths/ expected number of deaths× 100)

Which of the following compounds is not excreted into the breast milk? A) alcohol B) nicotine C) dioxin D) PCB E) phenol

E) phenol Due to their lipophilic characteristic the first four substances can be excreted into the breast milk, so the breastfed child can be exposed to them. Phenol is hydrophilic, therefore it does not appear in the breast milk.

Which substance is not asbestos? A) crocidolite B) Amosite C) antofilit D) tremolos E) stalactite

E) stalactite There are two known groups and six types of minerals related to asbestos: serpentine group - chrysotile, amphibole group -, crocidolite, amosite, antofilite, tremolite, actinolite.

What does "hidden diabetes" mean? A) no clinical symptoms are present due to treatment B) not the primary disease but the complications of the primary disease cause the symptoms C) the symptoms of the disease are masked by the characteristic symptoms of an other disease D) only the hypertension accompanying diabetes is treated while the diabetes itself remains untreated E) the existence of diabetes is unknown by both the patient and the doctor, so diabetes can be detected by a screening test only

E) the existence of diabetes is unknown by both the patient and the doctor, so diabetes can be detected by a screening test only A study related to the observation of hidden diabetes was conducted among patients aged 55-64 based on the GPMSSP. According to the study, the prevalence of hidden diabetes is quite high, because every third or fourth diabetic patient was detected only by screening tests and not by the doctor or the patient.

Which disease incidence has increased in our country (in Hungary) in the age group over 40 years? A) Scarlet B) diphtheria C) pertussis D) Haemophilus influenzae meningitis E) tuberculosis

E) tuberculosis The new cases of tuberculosis have risen in the age group over 40 year in Hungary. The morbidity of scarlet fever among the 3-5 year-olds is the highest; Diphtheria has occurred only in the form of imported cases in the recent decades; Pertussis is a disease of infants who have not received active immunization yet; Infection of H. influenzae in children under 5 years has been reported sporadically.

The most influencing social factor related to suicide behavior is the following: A) inability to loan repayment B) residential segregation C) existential uncertainty caused by economic crisis D) entrepreneurial lifestyle E) unemployment

E) unemployment The most important social factor related to suicide behavior was unemployment according to a survey conducted in Hungary in 1995. Nearly 40% of the unemployed have suicidal thoughts and over 7% of them commit a suicide attempt which requires medical intervention (within the entrepreneurial group the corresponding numbers are 13.4% and 1.2%, respectively). The rate of repeated suicide attempts was highest among the unemployed, too.

If a person has received a complete immunization within 5 years against lyssa, what is the applicable active immunization protocol in the case of a suspected new infection? A) vaccination on days 0, 7 and 21 (2-1-1 doses) B) vaccination on days 0, 7 and 21 (1-1-1 doses) C) vaccinaton on days 0, 3 and 7 (2-1-1 doses) D) vaccination on days 0, 3 and 7 (1-1-1 doses) E) vaccination on days 0 and 3 (1-1 dose)

E) vaccination on days 0 and 3 (1-1 dose) If the exposed person has received a complete vaccination sequence within 5 years, in the case of a new infection his (her) immunity is sufficient to receive a partial vaccination series only for active immunization. This scheme is described in answser E. Answer A corresponds to the full immunization for those who have not been vaccinated or who have been vaccinated for more than 5 years. Answers B, C and D are incorrect, there are no vaccinatiol protocols like these.

Which industrial activity has no risk of causing cancer? A) aluminum production B) furniture production C) shoe making D) coke production E) working with silicate

E) working with silicate High cancer-risk industries are those industrial areas, where most of the people employed are exposed to one or more carcinogenic materials and within the population of workers the incidence of tumor diseases exceeds the expected incidence that is the incidence of cancer in the general population.


Kaugnay na mga set ng pag-aaral

General Securities Industry Exam Questions

View Set

Chiropractic History & Principles

View Set

Chapter 22: Management of Patients with Upper Respiratory Tract Disorders

View Set